evolve exam 3

¡Supera tus tareas y exámenes ahora con Quizwiz!

The nurse is teaching a client about induction therapy for acute leukemia. Which client statement indicates a need for additional education? a. "After this therapy, I will not need to have any more." b. "I will need to avoid people with a cold or flu." c. "I will probably lose my hair during this therapy." d. "The goal of this therapy is to put me in remission."

a. "After this therapy, I will not need to have any more." Induction therapy is not a cure for leukemia, it is a treatment; therefore, the client needs more education to understand this. Because of infection risk, clients with leukemia should avoid people with a cold or flu. Induction therapy will most likely cause the client with leukemia to lose his or her hair. The goal of induction therapy is to force leukemia into remission.

A client, who is a mother of two, has autosomal dominant polycystic kidney disease (ADPKD). Which statement by the client indicates a need for further education about her disease? a. "By maintaining a low-salt diet in our house, I can prevent ADPKD in my children." b. "Even though my children don't have symptoms at the same age I did, they can still have ADPKD." c. "If my children have the ADPKD gene, they will have cysts by the age of 30." d. "My children have a 50% chance of inheriting the ADPKD gene that causes the disease."

a. "By maintaining a low-salt diet in our house, I can prevent ADPKD in my children." There is no way to prevent ADPKD, although early detection and management of hypertension may slow the progression of kidney damage. Limiting salt intake can help control blood pressure. Presentation of ADPKD can vary by age of onset, manifestations, and illness severity, even in one family. Almost 100% of those who inherit a polycystic kidney disease (PKD) gene will develop kidney cysts by age 30. Children of parents who have the autosomal dominant form of PKD have a 50% chance of inheriting the gene that causes the disease.

A client with an endocrine disorder says, "I can't, you know, satisfy my wife anymore." What is the nurse's best response? a. "Can you please tell me more?" b. "Don't worry. That is normal." c. "How does she feel?" d. "Should I make an appointment with a counselor?"

a. "Can you please tell me more?" Asking the client to explain his concerns in an open-ended question allows the nurse to explore his feelings more thoroughly. Telling a client that something is "normal" is dismissive; this is new to the client and is a concern for him. The focus of the nurse's response needs to be on the client, not on the wife initially. Referring the client to a counselor is not an appropriate first step; this dismisses the client's concerns and does not allow him to express his frustrations at the moment.

An older adult woman who reports a change in bladder function says, "I feel like a child who sometimes pees her pants." What is the nurse's best response? a. "Have you tried using the toilet at least every couple of hours?" b. "How does that make you feel?" c. "We can fix that." d."That happens when we get older."

a. "Have you tried using the toilet at least every couple of hours?" By emptying the bladder on a regular basis, urinary incontinence from overflow may be avoided, which may give the client some sense of control. The client has already stated how she feels; asking her again how she feels does not address her concern nor does it allow for nursing education. The nurse cannot assert that the problem can be fixed because this may be untrue. Suggesting that the problem occurs as we get older does not address the client's concern and does not provide for nursing education.

The nurse is instructing a client who will undergo a suppression test. Which statement by the client indicates that teaching was effective? a. "I am being tested to see whether my hormone glands are hyperactive." b. "I am being tested to see whether my hormone glands are hypoactive." c. "I am being tested to see whether my kidneys work at all." d. "I will be given more hormones as a trigger."

a. "I am being tested to see whether my hormone glands are hyperactive." Suppression tests are used when hormone levels are high or in the upper range of normal. Failure of suppression of hormone production during testing indicates hyperfunction. A stimulation test assesses whether hormone glands are hypoactive. The adrenal glands are endocrine glands that are located on the kidneys; a suppression test does not measure kidney function. Hormones are given as a trigger in a stimulation test.

The nurse is providing discharge teaching to a client with diabetes about injury prevention for peripheral neuropathy. Which statement by the client indicates a need for further teaching? a. "I can break in my shoes by wearing them all day." b. "I need to monitor my feet daily for blisters or skin breaks." c. "I should never go barefoot." d. "I should quit smoking."

a. "I can break in my shoes by wearing them all day." Shoes should be properly fitted and worn for a few hours a day to break them in, with frequent inspection for irritation or blistering. People with diabetes have decreased peripheral circulation, so even small injuries to the feet must be managed early. Going barefoot is contraindicated. Tobacco use further decreases peripheral circulation in a client with diabetes.

A client recently had an amputation of the right hand. Which statement by the client, who was right-handed, indicates that he or she is coping effectively? a. "I can learn to write with my left hand." b. "I'll need help with all of my personal care." c. "Clothing will cover my missing hand." d. "People will look at me differently."

a. "I can learn to write with my left hand." The client's willingness to learn to write with his or her left hand indicates that the client is coping effectively by planning to adapt to the loss of the right hand. The client can adapt to the use of assistive devices to be independent in personal care. The client's desire for help with all personal care indicates lack of willingness or information or both. Wanting to cover the missing hand with clothing indicates that the client is not adjusting to the loss of the hand. Concern over people looking at him or her differently is a realistic concern for the client, but it also indicates that the client is not coping effectively regarding the amputated limb.

A client is being discharged with propylthiouracil (PTU). Which statement by the client indicates a need for further teaching by the nurse? a. "I can return to my job at the nursing home." b. "I must call if my urine is dark." c. "I must faithfully take the drug every 8 hours." d. "I need to report weight gain."

a. "I can return to my job at the nursing home." The client should avoid large crowds and people who are ill because PTU reduces blood cell counts and the immune response, which increases the risk for infection. The client does not, however, need to remain completely at home. Dark urine may indicate liver toxicity or failure, and the client must notify the provider immediately. Taking PTU regularly at the same time each day provides better drug levels and ensures better drug action. The client must notify the provider of weight gain because this may indicate hypothyroidism; a lower drug dose may be required.

The school nurse is counseling a teenage student about how to prevent kidney trauma. Which statement by the student indicates a need for further teaching? a. "I can't play any type of contact sports because my brother had kidney cancer." b. "I avoid riding motorcycles." c. "I always wear pads when playing football." d. "I always wear a seat belt in the car."

a. "I can't play any type of contact sports because my brother had kidney cancer." Contact sports and high-risk activities should be avoided if a person has only one kidney. A family history of kidney cancer does not prohibit this type of activity. To prevent kidney and genitourinary trauma, caution should be taken when riding bicycles and motorcycles. People should wear appropriate protective clothing when participating in contact sports. Anyone riding in a car should wear a seat belt.

The nurse educates a group of women who have had frequent urinary tract infections (UTIs) about how to avoid recurrences. Which client statement shows a correct understanding of what the nurse has taught? a. "I should be drinking at least 1.5 to 2.5 liters of fluids every day." b. "It is a good idea for me to reduce germs by taking a tub bath daily." c. "Trying to get to the bathroom to urinate every 6 hours is important for me." d. "Urinating 1000 mL on a daily basis is a good amount for me."

a. "I should be drinking at least 1.5 to 2.5 liters of fluids every day." To reduce the number of UTIs, clients should be drinking a minimum of 1.5 to 2.5 liters of fluid (mostly water) each day. Showers, rather than tub baths, are recommended for women who have recurrent UTIs. Urinating every 3 to 4 hours is ideal for reducing the occurrence of UTI. This is advisable rather than waiting until the bladder is full to urinate. Urinary output should be at least 1.5 liters daily. Ensuring this amount "out" is a good indicator that the client is drinking an adequate amount of fluid.

The nurse is teaching a client with type 2 diabetes about the importance of weight control. Which comment by the client indicates a need for further teaching? a. "I should begin exercising for at least an hour a day." b. "I should monitor my diet." c. "If I lose weight, I may not need to use the insulin anymore." d. "Weight loss can be a sign of diabetic ketoacidosis."

a. "I should begin exercising for at least an hour a day." For long-term maintenance of major weight loss, large amounts of exercise (7 hr/wk) or moderate or vigorous aerobic physical activity may be helpful, but the client must start slowly. Monitoring the diet is key to type 2 diabetes management. Weight loss can minimize the need for insulin and can also be a sign of diabetic ketoacidosis.

The nurse is questioning a female client with a urinary tract infection (UTI) about her antibiotic drug regimen. Which statement by the client indicates a need for further instruction? a. "I take my medication only when I have symptoms." b. "I always wipe front to back." c. "I don't use bubble baths and other scented bath products." d. "I try to drink 3 liters of fluid a day."

a. "I take my medication only when I have symptoms." Clients with UTIs must complete all prescribed antibiotic therapy, even when symptoms of infection are absent. Wiping front to back helps prevent UTIs because it prevents infection-causing microorganisms in the stool from getting near the urethra. Limiting bubble baths and drinking 3 liters of fluid a day help prevent UTIs.

A client with bone cancer is scheduled for a right upper extremity amputation. Which statement by the client's husband indicates an effective coping strategy? a. "I'll have to find ways to help my wife focus on positive aspects of her body." b. "The family will avoid direct discussion of my wife's amputation." c. "I'll try to limit her visitors." d. "My family will use diversional methods to help her not focus on the amputation."

a. "I'll have to find ways to help my wife focus on positive aspects of her body." Planning to help the client focus on positive aspects of her body illustrates that the husband is coping with the change in his wife's body image in a positive way. Planning to have the family avoid direct discussion of the amputation does not allow the client the opportunity to discuss her feelings about the loss of a limb. Visitors could be a source of comfort and may provide a way for the client to express her feelings, so visitors should not be limited. Using diversional methods to help the client not focus on the amputation is not an effective coping strategy; it limits the chance for the client to discuss feelings about the amputation.

A client who was previously diagnosed with a urinary tract infection (UTI) and started on antibiotics returns to the clinic 3 days later with the same symptoms. When asked about the previous UTI and medication regimen, the client states, "I only took the first dose because after that, I felt better." How does the nurse respond? a. "Not completing your medication can lead to return of your infection." b. "That means your treatment will be prolonged with this new infection." c. "This means you will now have to take two drugs instead of one." d. "What you did was okay; however, let's get you started on something else."

a. "Not completing your medication can lead to return of your infection." Not completing the drug regimen can lead to recurrence of an infection and bacterial drug resistance. Needing to be re-treated does not mean that the client will have a prolonged treatment regimen. Some treatment modalities are given over a 3-day period. Given this client's history, larger doses for a shorter time span may be a wise plan. The client does not need to take two drugs, and this response is punitive rather than instructive. Saying that the client's actions were okay does not inform the client with respect to nonadherence. The client needed to take all the prescribed medication to make certain that the infection was properly treated.

The nurse is instructing a client who has been prescribed calcium citrate (Citracal). Which instruction does the nurse include? a. "Take Citracal with food." b. "For best absorption, take Citracal with a carbonated beverage." c. "One third of the daily dose is best taken during the day." d. "Milk of Magnesia (MOM) should be taken with Citracal."

a. "Take Citracal with food." Calcium supplements can cause gastric upset; taking Citracal with food can minimize gastric upset. Calcium citrate should be taken with 6 to 8 ounces of water, not carbonated beverages. One third of the daily dose is best taken at bedtime. MOM is not indicated and actually may lead to decreased absorption of calcium citrate.

A client has sustained a fracture of the left tibia. The extremity is immobilized using an external fixation device. Which postoperative instruction does the nurse include in this client's teaching plan? a. "Use pain medication as prescribed to control pain." b. "Clean the pin site when any drainage is noticed." c. "Wear the same clothing that is normally worn." d. "Apply bacitracin (Neosporin) if signs or symptoms of infection develop around pin sites."

a. "Use pain medication as prescribed to control pain." The client should be taught the correct use of prescribed pain medication to control pain adequately. Pin sites must be cleaned at least every 8 hours and as needed to reduce the risk for infection, not when any drainage is noticed. The client will have to adjust the type of clothing worn while the fixation device is in place. If signs and symptoms of infection develop around the pin sites, the client must notify the health care provider immediately. Infection at the pin sites places the client at risk for osteomyelitis.

What information will the nurse provide to a client who is scheduled for extracorporeal shock wave lithotripsy? (Select all that apply.) a. "Your urine will be strained after the procedure." b. "Be sure to finish all of your antibiotics." c. "Immediately call the health care provider if you notice bruising." d. "Remember to drink at least 3 liters of fluid a day to promote urine flow." e. "You will need to change the incisional dressing once a day."

a. "Your urine will be strained after the procedure." b. "Be sure to finish all of your antibiotics." d. "Remember to drink at least 3 liters of fluid a day to promote urine flow." After lithotripsy, urine is strained to monitor the passage of stone fragments. Clients must finish the entire antibiotic prescription to decrease the risk of developing a urinary tract infection. Drinking at least 3 L of fluid a day dilutes potential stone-forming crystals, prevents dehydration, and promotes urine flow. Bruising on the flank of the affected side is expected after lithotripsy as a result of the shock waves that break the stone into small fragments. The client must notify the health care provider if he or she develops pain, fever, chills, or difficulty with urination because these signs and symptoms may signal the beginning of an infection or the formation of another stone. There is no incision with extracorporeal shock wave lithotripsy. There may be a small incision when intracorporeal lithotripsy is performed.

Which clients with long-term urinary problems does the nurse refer to community resources and support groups? (Select all that apply.) a. A 32-year-old with a cystectomy b. A 44-year-old with a Kock pouch c. A 48-year-old with urinary calculi d. A 78-year-old with urinary incontinence e. An 80-year-old with dementia

a. A 32-year-old with a cystectomy b. A 44-year-old with a Kock pouch d. A 78-year-old with urinary incontinence Clients with a cystectomy, Kock pouch, or urinary incontinence would benefit from community resources and support groups. Others who have had their bladders removed are good sources of information and for help in establishing coping mechanisms. They can provide ideas for living with the problem or methods of curing (or minimizing) it. Urinary calculi typically are not a long-term problem that requires community resources and support groups. The older adult client with dementia would not benefit from community resources and support groups because of the client's cognitive difficulties.

Which client is at greatest risk for having a hemolytic transfusion reaction? a. A 34-year-old client with type O blood b. A 42-year-old client with allergies c. A 58-year-old immune-suppressed client d. A 78-year-old client

a. A 34-year-old client with type O blood Hemolytic transfusion reactions are caused by blood type or Rh incompatibility. When blood that contains antigens different from the client's own antigens is infused, antigen-antibody complexes are formed in the client's blood. Type O is considered the universal donor, but not the universal recipient. The client with allergies would be most susceptible to an allergic transfusion reaction. The immune-suppressed client would be most susceptible to a transfusion-associated graft-versus-host disease. The older adult client would be most susceptible to circulatory overload.

An RN from pediatrics has "floated" to the medical-surgical unit. Which client is assigned to the float nurse? a. A 42-year-old with sickle cell disease receiving a transfusion of packed red blood cells b. A 50-year-old with pancytopenia needing assessment of risk factors for aplastic anemia c. A 55-year-old with folic acid deficiency anemia caused by alcohol abuse who needs counseling d. A 60-year-old with newly diagnosed polycythemia vera who needs teaching about the disease

a. A 42-year-old with sickle cell disease receiving a transfusion of packed red blood cells Because sickle cell disease is commonly diagnosed during childhood, the pediatric nurse will be familiar with the disease and with red blood cell transfusion; therefore, he or she should be assigned to the client with sickle cell disease. Aplastic anemia, folic acid deficiency, and polycythemia vera are problems more commonly seen in adult clients who should be cared for by nurses who are more experienced in caring for adults.

Which finding in the first 24 hours after kidney transplantation requires immediate intervention? a. Abrupt decrease in urine output b. Blood-tinged urine c. Incisional pain d. Increase in urine output

a. Abrupt decrease in urine output An abrupt decrease in urine output may indicate complications such as rejection, acute kidney injury, thrombosis, or obstruction. Blood-tinged urine, incisional pain, and an increase in urine output are expected findings after kidney transplantation.

A rock climber has sustained an open fracture of the right tibia after a 20-foot fall. The nurse plans to assess the client for which potential complications? (Select all that apply.) a. Acute compartment syndrome (ACS) b. Fat embolism syndrome (FES) c. Congestive heart failure d. Urinary tract infection (UTI) e. Osteomyelitis

a. Acute compartment syndrome (ACS) b. Fat embolism syndrome (FES) e. Osteomyelitis ACS is a serious condition in which increased pressure within one or more compartments reduces circulation to the area. A fat embolus is a serious complication in which fat globules are released from yellow bone marrow into the bloodstream within 12 to 48 hours after the injury. FES usually results from long bone fracture or fracture repair, but is occasionally seen in clients who have received a total joint replacement. Bone infection, or osteomyelitis, is most common in open fractures. Congestive heart failure is not a potential complication for this client; pulmonary embolism is a potential complication of venous thromboembolism, which can occur with fracture. The client is at risk for wound infection resulting from orthopedic trauma, not a UTI.

A client with syndrome of inappropriate antidiuretic hormone is admitted with a serum sodium level of 105 mEq/L. Which request by the health care provider does the nurse address first? a. Administer infusion of 150 mL of 3% NaCl over 3 hours. b. Draw blood for hemoglobin and hematocrit. c. Insert retention catheter and monitor urine output. d. Weigh the client on admission and daily thereafter.

a. Administer infusion of 150 mL of 3% NaCl over 3 hours. The client with a sodium level of 105 mEq/L is at high risk for seizures and coma. The priority intervention is to increase the sodium level to a more normal range. Ideally, 3% NaCl should be infused through a central line or with a small needle through a large vein to prevent irritation. Monitoring laboratory values for fluid balance and monitoring urine output are important, but are not the top priority. Monitoring client weight will help in the assessment of fluid balance; however, this is also not the top priority.

A client presents to the emergency department with a history of adrenal insufficiency. The following laboratory values are obtained: Na+ 130 mEq/L, K+ 5.6 mEq/L, and glucose 72 mg/dL. Which is the first request that the nurse anticipates? a. Administer insulin and dextrose in normal saline to shift potassium into cells. b. Give spironolactone (Aldactone) 100 mg orally. c. Initiate histamine2 (H2) blocker therapy with ranitidine for ulcer prophylaxis. d. Obtain arterial blood gases to assess for peaked T waves.

a. Administer insulin and dextrose in normal saline to shift potassium into cells. This client is hyperkalemic. The nurse should anticipate a request to administer 20 to 50 units of insulin with 20 to 50 mg of dextrose in normal saline as an IV infusion to shift potassium into the cells. Spironolactone is a potassium-sparing diuretic that helps the body keep potassium, which the client does not need. Although H2 blocker therapy would be appropriate for this client, it is not the first priority. Arterial blood gases are not used to assess for peaked T waves associated with hyperkalemia; an electrocardiogram needs to be obtained instead.

A client who is admitted with urolithiasis reports "spasms of intense flank pain, nausea, and severe dizziness." Which intervention does the nurse implement first? a. Administer morphine sulfate 4 mg IV. b. Begin an infusion of metoclopramide (Reglan) 10 mg IV. c. Obtain a urine specimen for urinalysis. d. Start an infusion of 0.9% normal saline at 100 mL/hr.

a. Administer morphine sulfate 4 mg IV. Morphine administered intravenously will decrease the pain and the associated sympathetic nervous system reactions of nausea and hypotension. An infusion of metoclopramide (Reglan) 10 mg IV should be begun after the client's pain is controlled. A urine specimen for urinalysis should be obtained and an infusion of 0.9% normal saline at 100 mL/hr should be started after the client's pain is controlled.

When caring for a client with nephrotic syndrome, which intervention should be included in the plan of care? a. Administering angiotensin-converting enzyme (ACE) inhibitors to decrease protein loss b. Administering heparin to prevent deep vein thrombosis (DVT) c. Providing antibiotics to decrease infection d. Providing transfusion of clotting factors

a. Administering angiotensin-converting enzyme (ACE) inhibitors to decrease protein loss ACE inhibitors can decrease protein loss in the urine. Heparin is administered for DVT, but in nephrotic syndrome it may reduce urine protein and kidney insufficiency. Glomerulonephritis may occur secondary to an infection, but it is an inflammatory process; antibiotics are not indicated for nephrotic syndrome. Clotting factors are not indicated unless bleeding and coagulopathy are present.

Which gland releases catecholamines? a. Adrenal b. Pancreas c. Parathyroid d. Thyroid

a. Adrenal The adrenal medulla releases catecholamines in response to stimulation of the sympathetic nervous system. The principal hormones of the pancreas are insulin, glucagon, and somatostatin. Parathyroid hormone is the principal hormone of the parathyroid gland. Triiodothyronine (T3), thyroxine (T4), and calcitonin are the principal hormones of the thyroid.

Which is the best referral that the nurse can suggest to a client who has been newly diagnosed with diabetes? a. American Diabetes Association b. Centers for Disease Control and Prevention c. Health care provider office d. Pharmaceutical representative

a. American Diabetes Association The American Diabetes Association can provide national and regional support and resources to clients with diabetes and their families. The Centers for Disease Control and Prevention does not focus on diabetes. The client's health care provider's office is not the best resource for diabetes information and support. A pharmaceutical representative is not an appropriate resource for diabetes information and support.

The nurse is caring for a group of hospitalized clients. Which client is at greatest risk for infection and sepsis? a. An 18-year-old who had an emergency splenectomy b. A 22-year-old with recently diagnosed sickle cell anemia c. A 38-year-old with hemolytic anemia d. A 40-year-old alcoholic with liver disease

a. An 18-year-old who had an emergency splenectomy Removal of the spleen causes reduced immune function. Without a spleen, the client is less able to remove disease-causing organisms, and is at increased risk for infection. Sickle cell anemia causes pain and discomfort because of the changed cell morphology, so acute pain, especially at joints, is the greatest threat to this client. A low red blood cell count with hemolytic anemia can contribute to a client's risk for infection, but this client is more at risk for low oxygen levels and ensuing fatigue. The liver plays a role in blood coagulation, so this client is more at risk for coagulation problems than for infection.

A 65-year-old female client has chronic hip pain and muscle atrophy from an arthritic disorder. Which musculoskeletal assessment finding does the nurse expect to see in the client? a. Antalgic gait b. Midswing gait c. Narrow-based stance d. No lurch in gait

a. Antalgic gait The client with chronic hip pain and muscle atrophy from an arthritic disorder would likely have a lurch in the gait (antalgic gait). Midswing gait is not a term used to assess a client's gait. This client would likely have a wide-based stance because of the musculoskeletal disorder.

The nurse should encourage fluids every 2 hours for older adult clients because of a decrease in which factor? a. Antidiuretic hormone (ADH) production b. General metabolism c. Glucose tolerance d. Ovarian production of estrogen

a. Antidiuretic hormone (ADH) production A decrease in ADH production causes urine to be more dilute, so urine might not concentrate when fluid intake is low. The older adult is at greater risk for dehydration as a result of urine loss. A decrease in general metabolism causes decreased tolerance to cold, decreased appetite, and decreased heart rate and blood pressure; it is not related to fluid intake or hydration. A decrease in glucose tolerance does not affect fluid intake or hydration. A decrease in estrogen production causes a decrease in bone density and is not related to fluid intake and hydration.

A client has a bone marrow biopsy done. Which nursing intervention is the priority postprocedure? a. Applying pressure to the biopsy site b. Inspecting the site for ecchymoses c. Sending the biopsy specimens to the laboratory d. Teaching the client about avoiding vigorous activity

a. Applying pressure to the biopsy site The initial action should be to stop bleeding by applying pressure to the site. Inspecting for ecchymoses, sending specimens to the laboratory, and teaching the client about activity levels will be done after hemostasis has been achieved.

A client's left arm is placed in a plaster cast. Which assessment does the nurse perform before the client is discharged? a. Assess that the cast is dry. b. Ensure that the client has 4 × 4 gauze to take home for placement between the cast and the skin. c. Check the fit of the cast by inserting a tongue blade between the cast and the skin. d. Ensure that the capillary refill of the left fingernail beds is longer than 3 seconds.

a. Assess that the cast is dry. The cast must be dry and free of cracking and crumbling before the client is discharged. The client should not place anything between the cast and the skin. In assessing fit, one finger should easily fit between the cast and the skin. Capillary refill longer than 3 seconds indicates impairment of the circulation in the extremity and requires the health care provider's immediate attention.

A client with iatrogenic Cushing's syndrome is a resident in a long-term care facility. Which nursing action included in the client's care would be best to delegate to unlicensed assistive personnel (UAP)? a. Assist with personal hygiene and skin care. b. Develop a plan of care to minimize risk for infection. c. Instruct the client on the reasons to avoid overeating. d. Monitor for signs and symptoms of fluid retention.

a. Assist with personal hygiene and skin care. Assisting a client with bathing and skin care is included in UAP scope of practice. It is not within their scope of practice to develop a plan of care, although they will play a very important role in following the plan of care. Client teaching requires a broad education and should not be delegated to UAP. Monitoring for signs and symptoms of fluid retention is part of client assessment, which requires a higher level of education and clinical judgment.

A client with chronic kidney disease reports chest pain. The nurse notes tachycardia and low-grade fever. Which additional assessment is warranted? a. Auscultate for pericardial friction rub. b. Assess for crackles c. Monitor for decreased peripheral pulses. d. Determine if the client is able to ambulate.

a. Auscultate for pericardial friction rub. The client with uremia is prone to pericarditis; symptoms include inspiratory chest pain, low-grade fever, and ST-segment elevation. Crackles and tachycardia are symptomatic of fluid overload; fever is not present. Although the nurse will monitor pulses, and ambulation is important to prevent weakness and deep vein thrombosis, these are not pertinent to the constellation of symptoms of pericarditis that the client presents with.

A client with a recently created vascular access for hemodialysis is being discharged. In planning discharge instructions, which information does the nurse include? a. Avoiding venipuncture and blood pressure measurements in the affected arm b. Modifications to allow for complete rest of the affected arm c. How to assess for a bruit in the affected arm d. How to practice proper nutrition

a. Avoiding venipuncture and blood pressure measurements in the affected arm Compression of vascular access causes decreased blood flow and may cause occlusion; if this occurs, dialysis will not be possible. The arm should be exercised to encourage venous dilation, not rested. The client can palpate for a thrill; a stethoscope is not needed to auscultate the bruit at home. The nurse should take every opportunity to discuss nutrition, even as it relates to wound healing, but loss of the graft or fistula by compression or occlusion must take priority because lifesaving dialysis cannot be performed.

A client with pheochromocytoma is admitted for surgery. What does the nurse do for the admitting assessment? a. Avoids palpating the abdomen b. Monitors for pulmonary edema with a chest x-ray c. Obtains a 24-hour urine specimen on admission d. Places the client in a room with a roommate for distraction

a. Avoids palpating the abdomen The abdomen must not be palpated in a client with pheochromocytoma because this action could cause a sudden release of catecholamines and severe hypertension. The tumor on the adrenal gland causes sympathetic hyperactivity, increasing blood pressure and heart rate, not pulmonary edema. A 24-hour urine collection will already have been completed to determine the diagnosis of pheochromocytoma. A client diagnosed with a pheochromocytoma may feel anxious as part of the disease process; providing a roommate for distraction will not reduce the client's anxiety.

The nurse is caring for a client who has just returned to the surgical unit after a radical nephrectomy. Which assessment information alarms the nurse? a. Blood pressure is 98/56 mm Hg; heart rate is 118 beats/min. b. Urine output over the past hour was 80 mL. c. Pain is at a level 4 (on a 0-to-10 scale). d. Dressing has a 1-cm area of bleeding.

a. Blood pressure is 98/56 mm Hg; heart rate is 118 beats/min. Bleeding is a complication of radical nephrectomy; tachycardia and hypotension may indicate impending hypovolemic or hemorrhagic shock. The surgeon should be notified immediately and fluids should be administered, complete blood count should be checked, and blood administered, if necessary. A urine output of 80 mL can be considered normal. The nurse can administer pain medication, but must address hemodynamic instability and possible hemorrhage first. Administering pain medication to a client who has developed shock will exacerbate hypotension. A dressing with a 1-cm area of bleeding is expected postoperatively.

What are the risk factors for the development of leukemia? (Select all that apply.) a. Bone marrow hypoplasia b. Chemical exposure c. Down syndrome d. Ionizing radiation e. Multiple blood transfusions f. Prematurity at birth

a. Bone marrow hypoplasia b. Chemical exposure c. Down syndrome d. Ionizing radiation Reduced production of blood cells in the bone marrow is one of the risk factors for developing leukemia. Exposure to chemicals through medical need or by environmental events can also contribute. Certain genetic factors contribute to the development of leukemia; Down syndrome is one such condition. Radiation therapy for cancer or other exposure to radiation, perhaps through the environment, also contributes. There is no indication that multiple blood transfusions are connected to clients who have leukemia. Although some genetic factors may influence the incidence of leukemia, prematurity at birth is not one of them.

A 56-year-old client admitted with a diagnosis of acute myelogenous leukemia is prescribed IV cytosine arabinoside for 7 days and an infusion of daunorubicin for the first 3 days. What is the major side effect of this therapy? a. Bone marrow suppression b. Liver toxicity c. Nausea d. Stomatitis

a. Bone marrow suppression Intravenous cytosine arabinoside and daunorubicin are a commonly prescribed course of aggressive chemotherapy, and bone marrow suppression is a major side effect. The client is even more at risk for infection than before treatment began. Liver toxicity, nausea, and stomatitis are not the major problems with this therapy.

The nurse is preparing the room for the client returning from a thyroidectomy. Which items are important for the nurse to have available for this client? (Select all that apply.) a. Calcium gluconate b. Emergency tracheotomy kit c. Furosemide (Lasix) d. Hypertonic saline e. Oxygen f. Suction

a. Calcium gluconate b. Emergency tracheotomy kit e. Oxygen f. Suction Calcium gluconate should be available at the bedside to treat hypocalcemia and tetany that might occur if the parathyroid glands have been injured during the surgery. Equipment for an emergency tracheotomy must be kept at the bedside in the event that hemorrhage or edema should occlude the airway. Respiratory distress can result from swelling or damage to the laryngeal nerve leading to spasm, so it is important that the nurse work with respiratory therapy to have oxygen ready at the bedside for the client on admission. Because of the potential for increased secretions, it is important that a working suction device is present at the bedside for admission of the client from the operating room. Furosemide might be useful in the postoperative client to assist with urine output; however, this is not of added importance for this client. Hypertonic saline would not be of benefit to this client as the client is not hyponatremic.

A client with type 1 diabetes arrives in the emergency department breathing deeply and stating, "I can't catch my breath." The client's vital signs are: T 98.4° F (36.9° C), P 112 beats/min, R 38 breaths/min, BP 91/54 mm Hg, and O2 saturation 99% on room air. Which action does the nurse take first? a. Check the blood glucose. b. Administer oxygen. c. Offer reassurance. d. Attach a cardiac monitor.

a. Check the blood glucose. The client's clinical presentation is consistent with diabetic ketoacidosis, so the nurse should initially check the client's glucose level. Based on the oxygen saturation, oxygen administration is not necessary. The nurse provides support, but it is early in the course of assessment and intervention to offer reassurance without more information. Cardiac monitoring may be implemented, but the first action should be to obtain the glucose level.

A client with a compound fracture of the left femur is admitted to the emergency department after a motorcycle crash. Which action is most essential for the nurse to take first? a. Check the dorsalis pedis pulses. b. Immobilize the left leg with a splint. c. Administer the prescribed analgesic. d. Place a dressing on the affected area.

a. Check the dorsalis pedis pulses. The first action should be to assess the circulatory status of the leg because the client is at risk for acute compartment syndrome, which can begin as early as 6 to 8 hours after an injury. Severe tissue damage can also occur if neurovascular status is compromised. Immobilization will be needed, but the nurse must assess the client's condition first. Administering an analgesic and placing a dressing on the affected area should both be done after the nurse has assessed the client.

When caring for a client 24 hours after a nephrectomy, the nurse notes that the client's abdomen is distended. Which action does the nurse perform next? a. Check vital signs. b. Notify the surgeon. c. Continue to monitor. d. Insert a nasogastric (NG) tube.

a. Check vital signs. The client's abdomen may be distended from bleeding; hemorrhage or adrenal insufficiency causes hypotension, so vital signs should be taken to see if a change in blood pressure has occurred. The surgeon should be notified after vital signs are assessed. An NG tube is not indicated for this client.

The charge nurse on the medical-surgical unit is making client assignments for the shift. Which client is the most appropriate to assign to an LPN/LVN? a. Client with Cushing's syndrome who requires orthostatic vital signs assessments b. Client with diabetes mellitus who was admitted with a blood glucose of 45 mg/dL c. Client with exophthalmos who has many questions about endocrine function d. Client with possible pituitary adenoma who has just arrived on the nursing unit

a. Client with Cushing's syndrome who requires orthostatic vital signs assessments An LPN/LVN will be familiar with Cushing's syndrome and the method for assessment of orthostatic vital signs. The client with a blood glucose of 45 mg/dL, the client with questions about endocrine function, and the client with a possible pituitary adenoma all have complex needs that require the experience and scope of practice of an RN.

After receiving change-of-shift report about these four clients, which client does the nurse attend to first? a. Client with acute adrenal insufficiency who has a blood glucose of 36 mg/dL b. Client with diabetes insipidus who has a dose of desmopressin (DDAVP) due c. Client with hyperaldosteronism who has a serum potassium of 3.4 mEq/L d. Client with pituitary adenoma who is reporting a severe headache

a. Client with acute adrenal insufficiency who has a blood glucose of 36 mg/dL A glucose level of 36 mg/dL is considered an emergency; this client must be assessed and treated immediately. Although it is important to maintain medications on schedule, the client requiring a dose of desmopressin is not the first client who needs to be seen. A serum potassium of 3.4 mEq/L in the client with hyperaldosteronism may be considered normal (or slightly hypokalemic), based on specific hospital levels. The client reporting a severe headache needs to be evaluated as soon as possible after the client with acute adrenal insufficiency. As an initial measure, the RN could delegate obtaining vital signs to unlicensed assistive personnel.

For which clients scheduled for a computed tomography (CT) scan with contrast does the nurse communicate safety concerns to the health care provider? (Select all that apply.) a. Client with an allergy to shrimp b. Client with a history of asthma c. Client who requests morphine sulfate every 3 hours d. Client with a blood urea nitrogen of 62 mg/dL and a creatinine of 2.0 mg/dL e. Client who took metformin (Glucophage) 4 hours ago

a. Client with an allergy to shrimp b. Client with a history of asthma d. Client with a blood urea nitrogen of 62 mg/dL and a creatinine of 2.0 mg/dL e. Client who took metformin (Glucophage) 4 hours ago The client who will be undergoing a CT scan with contrast should be asked about known hay fever or food or drug allergies, especially to seafood, eggs, milk, or chocolate. Contrast reactions have been reported to be as high as 15% in these clients. Clients with asthma have been shown to be at greater risk for contrast reactions than the general public. When reactions do occur, they are more likely to be severe. The risk for contrast-induced nephropathy is increased in clients who have pre-existing renal insufficiency (e.g., serum creatinine levels greater than 1.5 mg/dL or estimated glomerular filtration rate less than 45 mL/min). Metformin must be discontinued at least 24 hours before and for at least 48 hours after any study using contrast media because the life-threatening complication of lactic acidosis, although rare, could occur. There are no contraindications to undergo CT with contrast while taking morphine sulfate. CT with contrast may help to identify the underlying cause of pain.

The nurse manager for the medical-surgical unit is making staff assignments. Which client will be most appropriate to assign to a newly graduated RN who has completed a 6-week unit orientation? a. Client with chronic hypothyroidism and dementia who takes levothyroxine (Synthroid) daily b. Client with follicular thyroid cancer who has vocal hoarseness and difficulty swallowing c. Client with Graves' disease who is experiencing increasing anxiety and diaphoresis d. Client with hyperparathyroidism who has just arrived on the unit after a parathyroidectomy

a. Client with chronic hypothyroidism and dementia who takes levothyroxine (Synthroid) daily The client with chronic hypothyroidism and dementia is the most stable of the clients described and would be most appropriate to assign to an inexperienced RN. A client with vocal hoarseness and difficulty swallowing is at higher risk for complications and requires close observation by a more experienced nurse. Increasing anxiety and diaphoresis in a client with Graves' disease can be an indication of impending thyroid storm, which is an emergency; this is not a situation to be managed by a newly graduated RN. A client who has just arrived on the unit after a parathyroidectomy requires close observation for bleeding and airway compromise and requires assessment by an experienced nurse.

The RN has just received change-of-shift report. Which of the assigned clients should be assessed first? a. Client with chronic kidney failure who was just admitted with shortness of breath b. Client with kidney insufficiency who is scheduled to have an arteriovenous fistula inserted c. Client with azotemia whose blood urea nitrogen and creatinine are increasing d. Client receiving peritoneal dialysis who needs help changing the dialysate bag

a. Client with chronic kidney failure who was just admitted with shortness of breath The dyspnea of the client with chronic kidney failure may indicate pulmonary edema and should be assessed immediately. The client with kidney insufficiency is stable and assessment can be performed later. The client with azotemia requires assessment and possible interventions, but is not at immediate risk for life-threatening problems. The client receiving peritoneal dialysis can be seen last because it is a slow process and the client has no urgent needs.

The nurse has just received change-of-shift report on the endocrine unit. Which client does the nurse see first? a. Client with type 1 diabetes whose insulin pump is beeping "occlusion" b. Newly diagnosed client with type 1 diabetes who is reporting thirst c. Client with type 2 diabetes who has a blood glucose of 150 mg/dL d. Client with type 2 diabetes with a blood pressure of 150/90 mm Hg

a. Client with type 1 diabetes whose insulin pump is beeping "occlusion" Because glucose levels will increase quickly in clients who use continuous insulin pumps, the nurse should assess this client and the insulin pump first to avoid diabetic ketoacidosis. Thirst is a symptom of hyperglycemia and, although important, is not a priority; the nurse could delegate a fingerstick blood glucose to unlicensed assistive personnel while assessing the client whose insulin pump is beeping. Although a blood glucose reading of 150 mg/dL is mildly elevated, this is not an emergency. Mild hypertension is also not an emergency.

To prevent pre-renal acute kidney injury, which person is encouraged to increase fluid consumption? a. Construction worker b. Office secretary c. Schoolteacher d. Taxicab driver

a. Construction worker Physical labor and working outdoors, especially in warm weather, cause diaphoresis and place the construction worker at risk for dehydration and pre-renal azotemia. The office secretary and schoolteacher work indoors and, even without air conditioning, will not lose as much fluid to diaphoresis as someone performing physical labor. The taxicab driver, even without air conditioning, will not experience diaphoresis and fluid loss like the construction worker.

The nurse is teaching a client about how to monitor therapy effectiveness for syndrome of inappropriate antidiuretic hormone. What does the nurse tell the client to look for? a. Daily weight gain of less than 2 pounds b. Dry mucous membranes c. Increasing heart rate d. Muscle spasms

a. Daily weight gain of less than 2 pounds The client must monitor daily weights because this assesses the degree of fluid restriction needed. A weight gain of 2 pounds or more daily or a gradual increase over several days is cause for concern. Dry mucous membranes are a sign of dehydration and an indication that therapy is not effective. An increased heart rate indicates increased fluid retention or dehydration and hypovolemia, and either condition is an indication that therapy is not effective. Muscle spasms are associated with hyponatremia and are an indication of a change in the client's neurologic status. Untreated hyponatremia can lead to seizures and coma.

The nurse is teaching a client with vitamin B12 deficiency anemia about dietary intake. Which type of food does the nurse encourage the client to eat? a. Dairy products b. Grains c. Leafy vegetables d. Starchy vegetables

a. Dairy products Dairy products such as milk, cheese, and eggs will provide the vitamin B12 that the client needs. Grains, leafy vegetables, and starchy vegetables are not a source of vitamin B12.

Which age-related change can cause nocturia? a. Decreased ability to concentrate urine b. Decreased production of antidiuretic hormone c. Increased production of erythropoietin d. Increased secretion of aldosterone

a. Decreased ability to concentrate urine Nocturia may result from decreased kidney-concentrating ability associated with aging. Increased production of antidiuretic hormone, decreased production of erythropoietin, and decreased secretion of aldosterone are age-related changes.

A client has been admitted to the medical intensive care unit with a diagnosis of diabetes insipidus (DI) secondary to lithium overdose. Which medication is used to treat the DI? a. Desmopressin (DDAVP) b. Dopamine hydrochloride (Intropin) c. Prednisone d. Tolvaptan (Samsca)

a. Desmopressin (DDAVP) Desmopressin is the drug of choice for treatment of severe DI. It may be administered orally, nasally, or by intramuscular or intravenous routes. Dopamine hydrochloride is a naturally occurring catecholamine and inotropic vasopressor; it would not be used to treat DI. Prednisone would not be used to treat DI. Tolvaptan is a selective competitive arginine vasopressin receptor 2 antagonist and is not used with DI.

An older adult client diagnosed with stress incontinence is prescribed the medication oxybutynin (Ditropan). Which side effects does the nurse tell the client to expect? (Select all that apply.) a. Dry mouth b. Increased blood pressure c. Increased intraocular pressure d. Constipation e. Reddish-orange urine color

a. Dry mouth c. Increased intraocular pressure d. Constipation Oxybutynin is an anticholinergic/antispasmodic. Side effects include dry mouth, urinary retention, constipation, and risk for increased intraocular pressure with the potential to make glaucoma worse. Alpha-adrenergic agonists and beta blockers, which may be prescribed for urinary incontinence, may cause an increase in blood pressure. Phenazopyridine, a bladder analgesic used to decrease urinary pain, causes the urine to be a reddish-orange color.

The nurse is teaching a group of older adult women about the signs and symptoms of urinary tract infection (UTI). Which concepts does the nurse explain in the presentation? (Select all that apply.) a. Dysuria b. Enuresis c. Frequency d. Nocturia e. Urgency f. Polyuria

a. Dysuria c. Frequency d. Nocturia e. Urgency Dysuria (painful urination), frequency, nocturia (frequent urinating at night), and urgency (having the urge to urinate quickly) are symptoms of UTI. Enuresis (bed-wetting) and polyuria (increased amounts of urine production) are not signs of a UTI.

When assisting with dietary protein needs for a client on peritoneal dialysis, the nurse recommends that the client select which food? a. Eggs b. Ham c. Eggplant d. Macaroni

a. Eggs Suggested protein-containing foods for a client on peritoneal dialysis are milk, meat, and eggs. Although a protein, ham is high in sodium, which should be avoided. Vegetables and pasta contain mostly carbohydrates. Peritoneal dialysis clients are allowed 1.2 to 1.5 g of protein/kg/day because protein is lost with each exchange.

Which assessment findings does the nurse expect in a client with kidney cancer? (Select all that apply.) a. Erythrocytosis b. Hypokalemia c. Hypercalcemia d. Hepatic dysfunction e. Increased sedimentation rate

a. Erythrocytosis c. Hypercalcemia d. Hepatic dysfunction e. Increased sedimentation rate Erythrocytosis alternating with anemia and hepatic dysfunction with elevated liver enzymes may occur with kidney cancer. Parathyroid hormone produced by tumor cells can cause hypercalcemia. An elevation in sedimentation rate may occur in paraneoplastic syndromes. Potassium levels are not altered in kidney cancer, but hypercalcemia is present.

Which clients are at risk for acute kidney injury (AKI)? (Select all that apply.) a. Football player in preseason practice b. Client who underwent contrast dye radiology c. Accident victim recovering from a severe hemorrhage d. Accountant with diabetes e. Client in the intensive care unit on high doses of antibiotics f. Client recovering from gastrointestinal influenza

a. Football player in preseason practice b. Client who underwent contrast dye radiology c. Accident victim recovering from a severe hemorrhage e. Client in the intensive care unit on high doses of antibiotics f. Client recovering from gastrointestinal influenza To prevent AKI, all people should be urged to avoid dehydration by drinking at least 2 to 3 liters of fluids daily, especially during strenuous exercise or work associated with diaphoresis, or when recovering from an illness that reduces kidney blood flow, such as influenza. Contrast media may cause acute renal failure, especially in older clients with reduced kidney function. Recent surgery or trauma, transfusions, or other factors that might lead to reduced kidney blood flow may cause AKI. Certain antibiotics may cause nephrotoxicity. Diabetes may cause acute kidney failure superimposed on chronic kidney failure.

A client with diabetes insipidus (DI) has dry lips and mucous membranes and poor skin turgor. Which intervention does the nurse provide first? a. Force fluids b. Offer lip balm c. Perform a 24-hour urine test d. Withhold desmopressin acetate (DDAVP)

a. Force fluids Dry lips and mucous membranes and poor skin turgor are indications of dehydration, which can occur with DI. This is a serious condition that must be treated rapidly. Encouraging fluids is the initial step, provided the client is able to tolerate oral intake. Lip balm may make the client more comfortable, but does not address the problem of dehydration. A 24-hour urine test will identify loss of electrolytes and adrenal androgen metabolites, but will not correct the dehydration that this client is experiencing. Desmopressin acetate is a synthetic form of antidiuretic hormone that is given to reduce urine production; it improves DI and should not be withheld.

Which risk factor is shared by clients who have osteoporosis or osteomalacia? a. High alcohol intake b. A history of smoking c. Inadequate exposure to sunlight d. Homelessness

a. High alcohol intake High alcohol intake is a risk factor for both osteoporosis and osteomalacia. A history of smoking is a risk factor for osteoporosis only. Inadequate exposure to sunlight and homelessness are risk factors for osteomalacia only.

Which urinary assessment information for a client indicates the potential need for increased fluids? a. Increased blood urea nitrogen b. Increased creatinine c. Pale-colored urine d. Decreased sodium

a. Increased blood urea nitrogen Increased blood urea nitrogen can indicate dehydration. Increased creatinine indicates kidney impairment. Pale-colored urine signifies diluted urine, which indicates adequate fluid intake. Increased, not decreased, sodium indicates dehydration.

Which clinical manifestation indicates the need for increased fluids in a client with kidney failure? a. Increased blood urea nitrogen (BUN) b. Increased creatinine level c. Pale-colored urine d. Decreased sodium level

a. Increased blood urea nitrogen (BUN) An increase in BUN can be an indication of dehydration, and an increase in fluids is needed. Increased creatinine indicates kidney impairment. Urine that is pale in color is diluted; an increase in fluids is not necessary. Sodium is increased, not decreased, with dehydration.

Which nursing intervention helps to reduce the incidence of osteomyelitis for a client receiving hemodialysis? a. Instructing the client to brush teeth after every meal b. Maintaining clean dressing change technique for long-term IV catheters c. Using clean technique d. Using Standard Precautions

a. Instructing the client to brush teeth after every meal Proper dental hygiene helps prevent periodontal infection, which can be a causative factor in osteomyelitis of the facial bone. Long-term IV catheters can be a primary source of infection, so dressing changes are done using sterile technique. All clients undergoing hemodialysis require careful sterile technique before needle cannulation. Standard Precautions should be used for all clients.

The nurse is teaching a client about maintaining a proper diet to prevent an endocrine disorder. Which food does the nurse suggest after the client indicates a dislike of fish? a. Iodized salt for cooking b. More red meat c. More green vegetables d. Salt substitute for cooking

a. Iodized salt for cooking Dietary deficiencies in iodide-containing foods may be a cause of an endocrine disorder. For clients who do not eat saltwater fish on a regular basis, teach them to use iodized salt in food preparation. The client should eat a well-balanced diet that includes less animal fat. Eating vegetables contributes to a proper diet; however, this does not prevent an endocrine disorder. Using a salt substitute does not prevent an endocrine disorder; in addition, salt substitutes may contain high levels of potassium, which may lead to electrolyte imbalances.

An older adult client has had an open reduction and internal fixation of a fractured right hip. Which intervention does the nurse implement for this client? a. Keep the client's heels off the bed at all times. b. Re-position the client every 3 to 4 hours. c. Administer preventive pain medication before deep-breathing exercises. d. Prohibit the use of antiembolic stockings.

a. Keep the client's heels off the bed at all times. Because the client is an older adult and is more at risk for skin breakdown because of impaired circulation and sensation, the client's heels must be kept off the bed at all times to avoid constant pressure on this sensitive area. Re-positioning the older adult client must be done every 2 hours, not every 3 to 4 hours, to prevent skin breakdown and to inspect the skin for any signs of breakdown. Pain medication would not be administered for deep-breathing exercises because this client typically would not experience pain upon breathing. Antiembolic stockings are not contraindicated for older adults; rather, they help prevent deep vein thrombosis.

A diabetic older adult client who had arthroscopic surgery on the right knee the previous day has a red, swollen, and painful right knee. The nurse anticipates that the health care provider will request which medication? a. Levofloxacin (Levaquin) b. Enoxaparin (Lovenox) c. Oxycodone (Roxicodone) d. Prednisone (Deltasone)

a. Levofloxacin (Levaquin) The client's symptoms indicate a possible right knee infection, so the first action will be to start antibiotic therapy, especially because the client is diabetic and is at greater risk for infection. Enoxaparin is an anticoagulant that can increase the risk for postoperative bleeding; the health care provider usually requests an opioid analgesic combination following arthroscopic surgery. Oxycodone is used for more invasive surgical procedures and is not indicated for this client. Prednisone is a glucocorticoid used to treat inflammation; it increases blood sugar and increases susceptibility to infection. Prednisone is not indicated for this client because the client is diabetic and is susceptible to infection.

When assessing a female client, the nurse learns that the client has several risk factors for osteoporosis. Which risk factor will be the priority for client teaching? a. Low calcium intake b. Postmenopausal status c. Positive family history d. Previous use of steroids

a. Low calcium intake The client's calcium intake is the only risk factor that the client can change. The nurse will discuss the other risk factors as contributing to osteoporosis, but the teaching will focus on ways to increase calcium intake. Postmenopausal status, positive family history, and previous use of steroids are not risk factors that the client can change. These risk factors should be discussed, but are not the priority for this client.

The nurse teaches a client who is recovering from acute kidney disease to avoid which type of medication? a. Nonsteroidal anti-inflammatory drugs (NSAIDs) b. Angiotensin-converting enzyme (ACE) inhibitors c. Opiates d. Calcium channel blockers

a. Nonsteroidal anti-inflammatory drugs (NSAIDs) NSAIDs may be nephrotoxic to a client with acute kidney disease, and should be avoided. ACE inhibitors are used for treatment of hypertension and to protect the kidneys, especially in the diabetic client, from progression of kidney disease. Opiates may be used by clients with kidney disease if severe pain is present; however, excretion may be delayed. Calcium channel blockers can improve the glomerular filtration rate and blood flow within the kidney.

The nurse is assessing a newly admitted client with thrombocytopenia. Which factor needs immediate intervention? a. Nosebleed b. Reports of pain c. Decreased urine output d. Increased temperature

a. Nosebleed The client with thrombocytopenia has a high risk for bleeding. The nosebleed should be attended to immediately. The client's report of pain, decreased urine output, and increased temperature are not the highest priority.

A client has undergone an elective below-the-knee amputation of the right leg as a result of severe peripheral vascular disease. In postoperative care teaching, the nurse instructs the client to notify the health care provider if which change occurs? a. Observation of a large amount of serosanguineous or bloody drainage b. Mild to moderate pain controlled with prescribed analgesics c. Absence of erythema and tenderness at the surgical site d. Ability to flex and extend the right knee

a. Observation of a large amount of serosanguineous or bloody drainage A large amount of serosanguineous or bloody drainage may indicate hemorrhage or, if an incision is present, that the incision has opened. This requires immediate attention. Mild to moderate pain controlled with prescribed analgesics would be a normal finding for this client. Absence of erythema and tenderness of the surgical site would also be normal findings for this client. The client should be able to flex and extend the right knee (limb) after surgery.

While managing care for a client with chronic kidney disease, which actions does the registered nurse (RN) plan to delegate to unlicensed assistive personnel (UAP)? (Select all that apply.) a. Obtain the client's pre-hemodialysis weight. b. Check the arteriovenous (AV) fistula for a thrill and bruit. c. Document the amount the client drinks throughout the shift. d. Auscultate the client's lung sounds every 4 hours. e. Explain the components of a low-sodium diet.

a. Obtain the client's pre-hemodialysis weight. c. Document the amount the client drinks throughout the shift. Obtaining the client's weight and documenting oral fluid intake are routine tasks that can be performed by UAP. Assessment skills (checking the AV fistula and auscultating lung sounds) and client education (explaining special diet) require more education and are in the legal scope of practice of the RN.

The nurse anticipates providing collaborative care for a client with a traumatic amputation of the right hand with which health care team members? (Select all that apply.) a. Occupational therapist b. Physical therapist c. Psychologist d. Respiratory therapist e. Speech therapist

a. Occupational therapist b. Physical therapist c. Psychologist An occupational therapist and a physical therapist will help to enable the client to become more independent in performing activities of daily living. An amputation can be traumatic to the client; loss of a body part should not be underestimated because the client may experience an altered self-concept, so counseling support with a psychologist should be made available to the client. The client does not have a respiratory condition that warrants collaborative care with a respiratory therapist. A speech therapist is not indicated because the client does not have speech impairment.

The nurse caring for four diabetic clients has all of these activities to perform. Which is appropriate to delegate to unlicensed assistive personnel (UAP)? a. Perform hourly bedside blood glucose checks for a client with hyperglycemia. b. Verify the infusion rate on a continuous infusion insulin pump. c. Monitor a client with blood glucose of 68 mg/dL for tremors and irritability. d. Check on a client who is reporting palpitations and anxiety.

a. Perform hourly bedside blood glucose checks for a client with hyperglycemia. Performing bedside glucose monitoring is an activity that may be delegated because it does not require extensive clinical judgment to perform; the nurse will follow up with the results. Intravenous therapy and medication administration are not within the scope of practice for UAP. The client with blood glucose of 68 mg/dL will need further monitoring, assessment, and intervention not within the scope of practice for UAP. The client reporting palpitations and anxiety may have hypoglycemia, requiring further intervention; this client must be assessed by licensed nursing staff.

Which is a priority problem for the older adult client diagnosed with bone cancer? a. Potential for injury related to weakness and drug therapy b. Altered self-esteem related to fear of death and dying c. Reduced mobility related to weakness and fatigue d. Pain of a chronic nature related to tumor invasion of other organs

a. Potential for injury related to weakness and drug therapy Older adult clients are more likely to fall and injure themselves because of weakness and the medications that they are prescribed, especially analgesics. Client problems of altered self-esteem, reduced mobility, and chronic pain are relevant, but are not the priority. The client's safety comes first.

When caring for a client with polycystic kidney disease, which goal is most important? a. Preventing progression of the disease b. Performing genetic testing c. Assessing for related causes d. Consulting with the dialysis unit

a. Preventing progression of the disease Preventing complications and progression of the disease is the goal. Genetic testing should be done, but this is not a priority. Assessment for related causes is an intervention, not a goal. Not all clients with polycystic kidney disease require dialysis.

When taking the health history of a client with acute glomerulonephritis (GN), the nurse questions the client about which related cause of the problem? a. Recent respiratory infection b. Hypertension c. Unexplained weight loss d. Neoplastic disease

a. Recent respiratory infection An infection often occurs before the kidney manifestations of acute GN. The onset of symptoms is about 10 days from the time of infection. Hypertension is a result of glomerulonephritis, not a cause. Weight gain, not weight loss, is symptomatic of fluid retention in GN. Cancers are not part of the cause of GN.

The nurse assists a client with acute kidney injury (AKI) to modify the diet in which ways? (Select all that apply.) a. Restricted protein b. Liberal sodium c. Restricted fluids d. Low potassium e. Low fat

a. Restricted protein c. Restricted fluids d. Low potassium Breakdown of protein leads to azotemia and increased blood urea nitrogen. Fluid is restricted during the oliguric stage. Potassium intoxication may occur, so dietary potassium is also restricted. Sodium is restricted during AKI because oliguria causes fluid retention. Fats may be used for needed calories when proteins are restricted.

Which condition may predispose a client to chronic pyelonephritis? a. Spinal cord injury b. Cardiomyopathy c. Hepatic failure d. Glomerulonephritis

a. Spinal cord injury Chronic pyelonephritis occurs with spinal cord injury, bladder tumor, prostate enlargement, or urinary tract stones. Weakness of the heart muscle may cause kidney impairment, not an infection. Pyelonephritis may damage the kidney, not the liver. Glomerulonephritis may result from infection, but may not cause infection of the kidney.

Which intervention does the nurse suggest to a client with a leg amputation to help cope with loss of the limb? a. Talking with an amputee close to the client's age who has had the same type of amputation b. Drawing a picture of how the client sees him- or herself c. Talking with a psychiatrist about the amputation d. Engaging in diversional activities to avoid focusing on the amputation

a. Talking with an amputee close to the client's age who has had the same type of amputation Meeting with someone of a comparable age who has gone through a similar experience will help the client cope better with his or her own situation. Drawing a picture is not therapeutic and may cause more harm than good. Unless the client is having serious maladjustment problems or has a coexisting psychological disorder, meeting with a psychiatrist should not be necessary. Diversional activities do not help the client deal with loss of the limb.

Which aspect of a musculoskeletal assessment will the physical therapist and the nurse plan to collaborate on? a. The need for ambulatory devices b. Medication that the client is currently taking c. Nutritional intake of the client before admission d. Current list of the client's medical conditions

a. The need for ambulatory devices The nurse and the physical therapist assess and collaborate on the need for ambulatory devices. It is the nurse's responsibility to assess which medications the client is currently taking. Nutritional assessment is performed by the nurse, but this might also involve a dietitian if special needs exist. The nurse assesses the client for all present medical conditions.

When caring for a client with uremia, the nurse assesses for which symptom? a. Tenderness at the costovertebral angle (CVA) b.Cyanosis of the skin c.Nausea and vomiting d. Insomnia

C. n/v Manifestations of uremia include anorexia, nausea, vomiting, weakness, and fatigue. CVA tenderness is a sign of inflammation or infection in the renal pelvis. Cyanosis is related to poor tissue perfusion. Insomnia is nonspecific and may be caused by psychoemotional factors, medications, or other problems.

When planning an assessment of the urethra, what does the nurse do first? a. Examine the meatus. b.Note any unusual discharge. c.Record the presence of abnormalities. d. Don gloves.

D. Don gloves Before examination begins, body fluid precautions (gloves) must be implemented first. Examining the meatus, noting unusual discharge, or recording the presence of abnormalities are things that the nurse should do after putting on gloves.

Which laboratory test is the best indicator of kidney function? a.Blood urea nitrogen (BUN) b. Creatinine c. Aspartate aminotransferase (AST) d. Alkaline phosphatase

B. creatinine Creatinine excretion, the end product of muscle metabolism, remains relatively steady and therefore is the best indicator of renal function. BUN may be affected by protein or fluid intake. AST and alkaline phosphatase are measures of hepatic function.

Which clients with an indwelling urinary catheter does the nurse reassess to determine whether the catheterization needs to be continued or can be discontinued? (Select all that apply.) a. Three-day postoperative client b. Client in the stepdown unit c. Comatose client with careful monitoring of intake and output (I&O) d. Incontinent client with perineal skin breakdown e. Incontinent older adult in long-term care

a. Three-day postoperative client b. Client in the stepdown unit e. Incontinent older adult in long-term care Three days after surgery, the postoperative client probably should be able to urinate on his or her own. This may be influenced by the type of surgery, but most clients do not need long-term catheterization after they have surgery. The incidence of complications (colonization of bacteria) begins to increase after 48 hours postinsertion. The client in the stepdown unit is definitely one who should be considered for catheter discontinuation; he or she should be somewhat ambulatory and able to get to a bedside commode. Incontinence in older adults does not necessarily mean that they have to be catheterized. The introduction of a catheter invites the possibility of infection. These clients can often be managed with adult incontinence pads with less risk for developing a urinary tract infection. These infections in the older adult population are serious and should be avoided. The comatose client who is on strict I&O must have a urinary catheter in place to keep an accurate account of fluid balance. A client who is incontinent with no breakdown areas would be considered, but perineal skin problems in this situation make a catheter necessary for this client's skin to have a clean, dry environment for healing.

A client with these assessment data is preparing to undergo a computed tomography scan with contrast: Flank pain BUN 54 mg/dL Captopril Dysuria Creatinine 2.4 mg/dL Metformin Bilateral knee pain Calcium 8.5 mg/dL Acetylcysteine Which medication does the nurse plan to administer before the procedure? a. Acetylcysteine (Mucosil) b. Metformin (Glucophage) c. Captopril (Capoten) d. Acetaminophen (Tylenol)

a. acetylcysteine (mucosal) This client has kidney impairment demonstrated by increased creatinine. Acetylcysteine (an antioxidant) may be used to prevent contrast-induced nephrotoxic effects. Metformin is held at least 24 hours before procedures using contrast. Although captopril and acetaminophen may be administered with a sip of water with permission of the provider, this is not essential before the procedure.

Which statement indicates to the nursing instructor that the nursing student understands the normal healing process of bone after a fracture? a. "A callus is quickly deposited and transformed into bone." b. "A hematoma forms at the site of the fracture." c. "Calcium and vascular proliferation surround the fracture site." d. "Granulation tissue reabsorbs the hematoma and deposits new bone."

b. "A hematoma forms at the site of the fracture." In stage 1, within 24 to 72 hours after a fracture, a hematoma forms at the site of the fracture because bone is extremely vascular. This then prompts the formation of fibrocartilage, providing the foundation for bone healing. Stage 2 of bone healing occurs within 3 days to 2 weeks after the fracture, when granulation tissue begins to invade the hematoma. Stage 3 of bone healing occurs as a result of vascular and cellular proliferation. In stage 4 of a healing fracture, callus is gradually reabsorbed and transformed into bone.

The nurse wishes to reduce the incidence of hospital-acquired acute kidney injury. Which question by the nurse to the interdisciplinary health care team will result in reducing client exposure? a. "Should we filter air circulation?" b. "Can we use less radiographic contrast dye?" c. "Should we add low-dose dobutamine?" d. "Should we decrease IV rates?"

b. "Can we use less radiographic contrast dye?" Contrast dye is severely nephrotoxic, and other options can be used in its place. Air circulation and low-dose dopamine are not associated with nephrotoxicity. Pre-renal status results from decreased blood flow to the kidney, such as fluid loss or dehydration; IV fluids can correct this.

A male client being treated for bladder cancer has a live virus compound instilled into his bladder as a treatment. What instruction does the nurse provide for postprocedure home care? a. "After 12 hours, your toilet should be cleaned with a 10% solution of bleach." b. "Do not share your toilet with family members for the next 24 hours." c. "Please be sure to stand when you are urinating." d. "Your underwear worn during the procedure and for the first 12 hours afterward should be bagged and discarded."

b. "Do not share your toilet with family members for the next 24 hours." The toilet should not be shared for 24 hours following this procedure because others using the toilet could be infected with the live virus that was instilled into the client. If only one toilet is available in the household, teach the client to flush the toilet after use and to follow this by adding 1 cup of undiluted bleach to the bowl water. The bowl is then flushed after 15 minutes, and the seat and flat surfaces of the toilet are wiped with a cloth containing a solution of 10% liquid bleach. The client must sit when he is urinating for at least 24 hours postprocedure to prevent splashing of the contaminated urine out of the commode, where it could be toxic for anyone who comes in contact with it. Underwear or other clothing that has come into contact with urine during the 24 hours after instillation should be washed separately from other clothing in a solution of 10% liquid bleach; it does not need to be discarded.

The nurse is assessing the endurance level of a client in a long-term care facility. What question does the nurse ask to get this information? a. "Are your feet or hands cold, even when you are in bed?" b. "Do you feel more tired after you get up and go to the bathroom?" c. "How much exercise do you get?" d. "What is your endurance level?"

b. "Do you feel more tired after you get up and go to the bathroom?" Asking about feeling tired after using the bathroom is pertinent to the client's activity and provides a comparison. The specific activity helps the client relate to the question and provide needed answers. Asking about cold feet or hands does not address the client's endurance. The hospitalized client typically does not get much exercise; this would be a difficult assessment for a client in long-term care facility to make. Asking the client about his or her endurance level is too vague; the client may not know how to answer this question.

The nurse is assessing a client for hematologic function risks and seeks to determine whether there is a risk that cannot be reduced or eliminated. Which clinical health history question does the nurse ask to obtain this information? a. "Do you seem to have excessive bleeding or bruising?" b. "Does anyone in your family bleed a lot?" c. "Tell me what you eat in a day." d. "Where do you work?"

b. "Does anyone in your family bleed a lot?" An accurate family history is important because many disorders that affect blood and blood clotting are inherited; genetics cannot be changed. Excessive bleeding or bruising is a symptom, not a risk. Diet can affect risk, but it is a health behavior that can be changed. Work habits can be a risk, such as working near radiation, but these are behaviors that can be changed.

The nurse is educating a group of young women who have sickle cell disease (SCD). Which comment from a class member requires correction? a. "Frequent handwashing is an important habit for me to develop." b. "Getting an annual 'flu shot' would be dangerous for me." c. "I must take my penicillin pills as prescribed, all the time." d. "The pneumonia vaccine is protection that I need."

b. "Getting an annual 'flu shot' would be dangerous for me." The client with SCD should receive annual influenza and pneumonia vaccinations; this helps prevent the development of these infections, which could cause a sickle cell crisis. Handwashing is a very important habit for the client with SCD to develop because it reduces the risk for infection. Prophylactic penicillin is given to clients with SCD orally twice a day to prevent the development of infection.

The nurse is assessing a client for endocrine dysfunction. Which comment by the client indicates a need for further assessment? a. "I am worried about losing my job because of cutbacks." b. "I don't have any patience with my kids. I lose my temper faster." c. "I don't seem to have any stressors now." d. "My weight has been stable these past few years."

b. "I don't have any patience with my kids. I lose my temper faster." Many endocrine problems can change a client's behavior, personality, and psychological responses; the client stating that he or she has become short-tempered warrants further assessment. Worrying about losing a job is a normal concern but does not give any indication of a need for further assessment. The nurse will need to assess the client's claim that he or she has no stressors at present because the client's response does not provide enough information to make this determination; however, the client's statement about losing patience is the priority. Weight gain or loss may or may not be an indication of an endocrine disorder.

The nurse is teaching a client with diabetes about proper foot care. Which statement by the client indicates that teaching was effective? a. "I should go barefoot in my house so that my feet are exposed to air." b. "I must inspect my shoes for foreign objects before putting them on." c. "I will soak my feet in warm water to soften calluses before trying to remove them." d. "I must wear canvas shoes as much as possible to decrease pressure on my feet."

b. "I must inspect my shoes for foreign objects before putting them on." To avoid injury or trauma to the feet, shoes should be inspected for foreign objects before they are put on. Diabetic clients should not go barefoot because foot injuries can occur. To avoid injury or trauma, a callus should be removed by a podiatrist, not by the client. The diabetic client must wear firm support shoes to prevent injury.

Which statement by a client with diabetic nephropathy indicates a need for further education about the disease? a. "Diabetes is the leading cause of kidney failure." b. "I need less insulin, so I am getting better." c. "My blood sugar may drop really low at times." d. "I must call my provider if the urine dipstick shows protein."

b. "I need less insulin, so I am getting better." When kidney function is reduced, insulin is available for a longer time and thus less of it is needed. Unfortunately, many clients believe this means that their diabetes is improving. It is true that diabetes mellitus is the leading cause of end-stage kidney disease among Caucasians in the United States. Clients with worsening kidney function may begin to have frequent hypoglycemic episodes. Proteinuria, which may be mild, moderate, or severe, indicates a need for follow-up.

The nurse is teaching a client about the manifestations and emergency treatment of hypoglycemia. In assessing the client's knowledge, the nurse asks the client what he or she should do if feeling hungry and shaky. Which response by the client indicates a correct understanding of hypoglycemia management? a. "I should drink a glass of water." b. "I should eat three graham crackers." c. "I should give myself 1 mg of glucagon." d. "I should sit down and rest."

b. "I should eat three graham crackers." Eating three graham crackers is a correct management strategy for mild hypoglycemia. Water or resting does not remedy hypoglycemia. Glucagon should be administered only in cases of severe hypoglycemia.

The nurse is teaching a client about the correct procedure for a 24-hour urine test for creatinine clearance. Which statement by the client indicates a need for further teaching? a. "I should keep the urine container cool in a separate refrigerator or cooler." b. "I should not eat any protein when I am collecting urine for this test." c. "I won't save the first urine sample." d. "To end the collection, I must empty my bladder, adding it to the collection."

b. "I should not eat any protein when I am collecting urine for this test." Eating protein does not interfere with collection or testing of the urine sample. Because the specimen must be kept cool, it can be placed in an inexpensive cooler with ice; the client should not keep the specimen container with food or beverages. The timing of the 24-hour collection begins after the initial void. To end a 24-hour urine specimen, emptying the bladder and adding it to the collection is the proper procedure.

Which client statement indicates that stem cell transplantation that is scheduled to take place in his home is not a viable option? a. "I don't feel strong enough, but my wife said she would help." b. "I was a nurse, so I can take care of myself." c. "I will have lots of medicine to take." d. "We live 5 miles from the hospital."

b. "I was a nurse, so I can take care of myself." Stem cell transplantation in the home setting requires support, assistance, and coordination from others. The client cannot manage this type of care on his own. The client must be emotionally stable to be a candidate for this type of care. It is acceptable for the client's spouse to support the client undergoing this procedure. It is not unexpected for the client to be taking several prescriptions. Five miles is an acceptable distance from the hospital, in case of emergency.

The nurse is teaching a client with a neurogenic bladder to use intermittent self-catheterization for bladder emptying. Which client statement indicates a need for further clarification? a. "A small-lumen catheter will help prevent injury to my urethra." b. "I will use a new, sterile catheter each time I do the procedure." c. "My family members can be taught to help me if I need it." d. "Proper handwashing before I start the procedure is very important."

b. "I will use a new, sterile catheter each time I do the procedure." Catheters are cleaned and reused. With proper handwashing and cleaning of the catheter, no increase in bacterial complications has been shown. Catheters are replaced when they show signs of deteriorating. The smallest lumen possible and the use of a lubricant help reduce urethral trauma to this sensitive mucous tissue. Research shows that family members in the home can be taught to perform straight catheterizations using a clean (rather than a sterile) catheter with good outcomes. Proper handwashing is extremely important in reducing the risk for infection in clients who use intermittent self-catheterization and is a principle that should be stressed.

The nurse is providing discharge teaching to a client with newly diagnosed diabetes. Which statement by the client indicates a correct understanding about the need to wear a MedicAlert bracelet? a. "If I become hyperglycemic, it is a medical emergency." b. "If I become hypoglycemic, I could become unconscious." c. "Medical personnel may need confirmation of my insurance." d. "I may need to be admitted to the hospital suddenly."

b. "If I become hypoglycemic, I could become unconscious." Hypoglycemia is the most common cause of medical emergency in clients with diabetes. A MedicAlert bracelet is helpful if the client becomes hypoglycemic and is unable to provide self-care. Hyperglycemia is not a medical emergency unless it is acidosis; people with diabetes tolerate mild hyperglycemia routinely. Insurance information and information needed for hospital admission do not appear on a MedicAlert bracelet.

An older adult client is discharged from the hospital for treatment of osteoporosis. What does the nurse include in client teaching related to the client's home safety? a. "Use area rugs on tile floors." b. "Keep walkways free of clutter." c. "Walk slowly on wet floor areas after mopping." d. "Keep light low to prevent glare."

b. "Keep walkways free of clutter." Walkways in the home must be clear of clutter and obstacles to help prevent falls. Clients with metabolic bone problems should not use area rugs at home because they may cause tripping or falling. Clients with metabolic bone problems must not walk on wet floors because the potential for falling is too great. Keeping the lights low would not allow the client to see adequately to walk safely or avoid an object on the floor.

A client is hesitant to talk to the nurse about genitourinary dysfunction symptoms. What is the nurse's best response? a. "Don't worry, no one else will know." b. "Take your time. What is bothering you the most?" c. "Why are you hesitant?" d. "You need to tell me so we can determine what is wrong."

b. "Take your time. What is bothering you the most?" Asking the client what is bothering him or her expresses patience and understanding when trying to identify the client's problem. Telling the client that others will not know is untrue because the client's symptoms will be in the medical record for other health care personnel to see. Asking why the client is hesitant can seem accusatory and threatening to the client. Admonishing the client to disclose his or her symptoms is too demanding; the nurse must be more understanding of the client's embarrassment.

Family members of a client diagnosed with hyperthyroidism are alarmed at the client's frequent mood swings. What is the nurse's response? a. "How does that make you feel?" b. "The mood swings should diminish with treatment." c. "The medications will make the mood swings disappear completely." d. "Your family member is sick. You must be patient."

b. "The mood swings should diminish with treatment." Telling the family that the client's mood swings should diminish over time with treatment will provide information to the family, as well as reassurance. Asking how the family feels is important; however, the response should focus on the client. Any medications or treatment may not completely remove the mood swings associated with hyperthyroidism. The family is aware that the client is sick; telling them to be patient introduces guilt and does not address the family's concerns.

A client with anemia asks the nurse, "Do most people have the same number of red blood cells?" How does the nurse respond? a. "No, they don't." b. "The number varies with gender, age, and general health." c. "Yes, they do." d. "You have fewer red blood cells because you have anemia."

b. "The number varies with gender, age, and general health." Telling the client that the number of red blood cells (RBCs) varies with gender, age, and general health is the most educational and reasonable response to the client's question. Although telling the client that people do not have the same number of RBCs is true, it is not informative, and there is a better answer. While it may be true that the client has fewer red blood cells because of anemia, it does not answer the client's general question.

The nurse is reinforcing information about genetic counseling to a client with sickle cell disease who has a healthy spouse. What information does the nurse include? a. "Sickle cell disease will be inherited by your children." b. "The sickle cell trait will be inherited by your children." c. "Your children will have the disease, but your grandchildren will not." d. "Your children will not have the disease, but your grandchildren could."

b. "The sickle cell trait will be inherited by your children." The children of the client with sickle cell disease will inherit the sickle cell trait, but may not inherit the disease. If both parents have the sickle cell trait, their children could get the disease.

The nurse is teaching a client about what to expect during a bone marrow biopsy. Which statement by the nurse accurately describes the procedure? a. "The doctor will place a small needle in your back and will withdraw some fluid." b. "You may experience a crunching sound or a scraping sensation as the needle punctures your bone." c. "You will be alone because the procedure is sterile; we cannot allow additional people to contaminate the area." d. "You will be sedated, so you will not be aware of anything."

b. "You may experience a crunching sound or a scraping sensation as the needle punctures your bone." It is accurate to describe a crunching sound or scraping sensation. Proper expectations minimize the client's fear during the procedure. A very large-bore needle is used for a bone marrow biopsy, not a small needle; the puncture is made in the hip or in the sternum, not the back. The nurse, or sometimes a family member, is available to the client for support during a bone marrow biopsy. The procedure is sterile at the site of the biopsy, but others can be present without contamination at the site. A local anesthetic agent is injected into the skin around the site. The client may also receive a mild tranquilizer or a rapid-acting sedative (such as lorazepam [Ativan]) but will not be completely sedated.

A client awaiting kidney transplantation states, "I can't stand this waiting for a kidney, I just want to give up." Which statement by the nurse is most therapeutic? a. "I'll talk to the health care provider and have your name removed from the waiting list." b. "You sound frustrated with the situation." c. "You're right, the wait is endless for some people." d. "I'm sure you'll get a phone call soon that a kidney is available."

b. "You sound frustrated with the situation." Acknowledging the client's frustration reflects the feelings the client is having and offers assistance and support. Talking to the health care provider and removing the client from the waiting list does not allow the nurse to hear more and perhaps offer therapeutic listening or a solution to the problem. Telling the client that the wait is endless for some people cuts the client off from sharing his or her concerns and accentuates the negative aspects of the situation. The waiting time for kidney matches is increasing due to a shortage of organs; the nurse should not offer false hope by suggesting that the client will get a phone call soon.

A client with type 1 diabetes mellitus received regular insulin at 7:00 a.m. The client should be monitored for hypoglycemia at which time? a. 7:30 a.m. b. 11:00 a.m. c. 2:00 p.m. d. 7:30 p.m.

b. 11:00 a.m. Onset of regular insulin is ½ to 1 hour; peak is 2 to 4 hours. Therefore, 11:00 a.m. is the anticipated peak time for regular insulin received at 7:00 a.m. For regular insulin received at 7:00 a.m., 7:30 a.m., 2:00 p.m., and 7:30 p.m. are not the anticipated peak times.

The client has sustained a traumatic amputation of the left arm after a machine accident. In what order should the following nursing actions be taken? 1. Apply direct pressure to the amputated site. 2. Elevate the extremity above the client's heart. 3. Assess the client for breathing problems. 4. Examine the amputation site. a. 2, 4, 3, 1 b. 3, 4, 1, 2 c. 1, 4, 3, 2 d. 4, 1, 2, 3

b. 3, 4, 1, 2 First, the airway must be assessed for breathing problems. Second, the nurse should examine the amputation site. Third, the nurse should apply direct pressure to the amputated site. Finally, the extremity should be elevated above the client's heart to decrease bleeding.

What is the appropriate range of urine output for the client who has just undergone a nephrectomy? a. 23 to 30 mL/hr b. 30 to 50 mL/hr c. 41 to 60 mL/hr d. 50 to 70 mL/hr

b. 30 to 50 mL/hr Urine output of 30 to 50 mL/hr or 0.5 to 1 mL/kg/hr is considered within acceptable range for the client who is post nephrectomy. Output of less than 25 to 30 mL/hr suggests decreased blood flow to the kidney and the onset or worsening of acute kidney injury. A large urine output, followed by hypotension and oliguria, is a sign of hemorrhage and adrenal insufficiency.

Which client does the nurse assign as a roommate for the client with aplastic anemia? a. A 23-year-old with sickle cell disease who has two draining leg ulcers b. A 28-year-old with glucose-6-phosphate dehydrogenase (G6PD) deficiency anemia who is receiving mannitol (Osmitrol) c. A 30-year-old with leukemia who is receiving induction chemotherapy d. A 34-year-old with idiopathic thrombocytopenia who is taking steroids

b. A 28-year-old with glucose-6-phosphate dehydrogenase (G6PD) deficiency anemia who is receiving mannitol (Osmitrol) Because clients with aplastic anemia usually have low white blood cell counts that place them at high risk for infection, roommates such as the client with G6PD deficiency anemia should be free from infection or infection risk. The client with sickle cell disease has two draining leg ulcer infections that would threaten the diminished immune system of the client with aplastic anemia. The client with leukemia who is receiving induction chemotherapy and the client with idiopathic thrombocytopenia who is taking steroids are at risk for development of infection, which places the client with aplastic anemia at risk, too.

The nurse receives the change-of-shift report on four clients. Which client does the nurse decide to assess first? a. A 26-year-old admitted 2 days ago with urosepsis with an oral temperature of 99.4° F (37.4° C) b. A 28-year-old with urolithiasis who has been receiving morphine sulfate and has not voided for 8 hours c. A 32-year-old admitted with hematuria and possible bladder cancer who is scheduled for cystoscopy d. A 40-year-old with noninfectious urethritis who is reporting "burning" and has estrogen cream prescribed

b. A 28-year-old with urolithiasis who has been receiving morphine sulfate and has not voided for 8 hours Anuria may indicate urinary obstruction at the bladder neck or urethra and is an emergency because obstruction can cause acute kidney failure. The client who has been receiving morphine sulfate may be oversedated and may not be aware of any discomfort caused by bladder distention. The 26-year-old admitted with urosepsis and slight fever, the 32-year-old scheduled for cystoscopy, and the 40-year-old with noninfectious urethritis are not at immediate risk for complications or deterioration.

After reviewing the laboratory test results, the nurse calls the health care provider about which client? a. A 44-year-old receiving warfarin (Coumadin) with an international normalized ratio (INR) of 3.0 b. A 46-year-old with a fever and a white blood cell (WBC) count of 1500/mm3 c. A 49-year-old with hemophilia and a platelet count of 150,000/mm3 d. A 52-year-old who has had a hemorrhage with a reticulocyte count of 0.8%

b. A 46-year-old with a fever and a white blood cell (WBC) count of 1500/mm3 The client with a fever is neutropenic and is at risk for sepsis unless interventions such as medications to improve the WBC level and antibiotics are prescribed. The INR of 3.0 in the 44-year-old indicates a therapeutic warfarin level. A platelet count of 150,000/mm3 in the 49-year-old is normal. An elevated reticulocyte count in the 52-year-old is expected after hemorrhage.

The nurse anticipates that a client who develops hypotension and oliguria post nephrectomy may need the addition of which element to the regimen? a. Increase in analgesics b. Addition of a corticosteroid c. Administration of a diuretic d. Course of antibiotic therapy

b. Addition of a corticosteroid Loss of water and sodium occurs in clients with adrenal insufficiency, which is followed by hypotension and oliguria; corticosteroids may be needed. The nurse should use caution when administering analgesics to a hypotensive client; no indication suggests that pain is present in this client. A diuretic would further contribute to fluid loss and hypotension, potentially worsening kidney function. A few doses of antibiotics are used prophylactically preoperatively and postoperatively; additional therapy is used when evidence of infection exists.

The nurse refers a client with an amputation and the client's family to which community resource? a. American Amputee Society (AAS) b. Amputee Coalition of America (ACA) c. Community Workers for Amputees (CWA) d. National Amputee of America Society (NAAS)

b. Amputee Coalition of America (ACA) The ACA is an available resource for clients with amputations and supports them and their families. The AAS, CWA, and NAAS do not exist

A client being treated for hyperthyroidism calls the home health nurse and mentions that his heart rate is slower than usual. What is the nurse's best response? a. Advise the client to go to a calming environment. b. Ask whether the client has increased cold sensitivity or weight gain. c. Instruct the client to see his health care provider immediately. d. Tell the client to check his pulse again and call back later.

b. Ask whether the client has increased cold sensitivity or weight gain. Increased sensitivity to cold and weight gain are symptoms of hypothyroidism, indicating an overcorrection by the medication. The client must be assessed further because he may require a lower dose of medication. A calming environment will not have any effect on the client's heart rate. The client will want to notify the health care provider about the change in heart rate. If other symptoms such as chest pain, shortness of breath, or confusion accompany the slower heart rate, then the client should see the health care provider immediately. If the client was concerned enough to call because his heart rate was slower than usual, the nurse needs to stay on the phone with the client while he re-checks his pulse. This time could also be spent providing education about normal ranges for that client.

The RN is working with unlicensed assistive personnel (UAP) in caring for a group of clients. Which action is best for the RN to delegate to UAP? a. Assessing the vital signs of a client who was just admitted with blunt flank trauma and hematuria b. Assisting a client who had a radical nephrectomy 2 days ago to turn in bed c. Helping the provider with a kidney biopsy for a client admitted with acute glomerulonephritis d. Palpating for bladder distention on a client recently admitted with a ureteral stricture

b. Assisting a client who had a radical nephrectomy 2 days ago to turn in bed UAP would be working within legal guidelines when assisting a client to turn in bed. Although assessment of vital signs is within the scope of practice for UAP, the trauma victim should be assessed by the RN because interpretation of the vital signs is needed. Assisting with procedures such as kidney biopsy and assessment for bladder distention are responsibilities of the professional nurse that should not be delegated to staff members with a limited scope of education.

When caring for a client with a left forearm arteriovenous fistula created for hemodialysis, which actions must the nurse take? (Select all that apply.) a. Check brachial pulses daily. b. Auscultate for a bruit every 8 hours. c. Teach the client to palpate for a thrill over the site. d. Elevate the arm above heart level. e. Ensure that no blood pressures are taken in that arm.

b. Auscultate for a bruit every 8 hours. c. Teach the client to palpate for a thrill over the site. e. Ensure that no blood pressures are taken in that arm. A bruit or swishing sound, and a thrill or buzzing sensation upon palpation should be present in this client, indicating patency of the fistula. No blood pressure, venipuncture, or compression, such as lying on the fistula, should occur. Distal pulses and capillary refill should be checked, and for a forearm fistula, the radial pulse is checked—the brachial pulse is proximal. Elevating the arm increases venous return, possibly collapsing the fistula.

Which instruction by the nurse will help a client with chronic kidney disease prevent renal osteodystrophy? a. Consuming a low-calcium diet b. Avoiding peas, nuts, and legumes c. Drinking cola beverages only once daily d. Increasing dairy products enriched with vitamin D

b. Avoiding peas, nuts, and legumes Kidney failure causes hyperphosphatemia; this client must restrict phosphorus-containing foods such as beans, peas, nuts (peanut butter), and legumes. Calcium should not be restricted; hyperphosphatemia results in a decrease in serum calcium and demineralization of the bone. Cola beverages and dairy products are high in phosphorus, contributing to hypocalcemia and bone breakdown.

The nurse admits an older adult client who sustained a left hip fracture and is in considerable pain. The nurse anticipates that the client will be placed in which type of traction? a. Balanced skin traction b. Buck's traction c. Overhead traction d. Plaster traction

b. Buck's traction Buck's traction may be applied before surgery to help decrease pain associated with muscle spasm. Balanced skin traction is indicated for fracture of the femur or pelvis. Overhead traction is indicated for fracture of the humerus with or without involvement of the shoulder and clavicle. Plaster traction is indicated for wrist fracture.

A client newly diagnosed with diabetes is not ready or willing to learn diabetes control during the hospital stay. Which information is the priority for the nurse to teach the client and the client's family? a. Causes and treatment of hyperglycemia b. Causes and treatment of hypoglycemia c. Dietary control d. Insulin administration

b. Causes and treatment of hypoglycemia The causes and treatment of hypoglycemia must be understood by the client and family to manage the client's diabetes effectively. The causes and treatment of hyperglycemia is a topic for secondary teaching and is not the priority for the client with diabetes. Dietary control and insulin administration are important, but are not the priority in this situation.

The nurse has just taken change-of-shift report on a group of clients on the medical-surgical unit. Which client does the nurse assess first? a. Client taking repaglinide (Prandin) who has nausea and back pain b. Client taking glyburide (Diabeta) who is dizzy and sweaty c. Client taking metformin (Glucophage) who has abdominal cramps d. Client taking pioglitazone (Actos) who has bilateral ankle swelling

b. Client taking glyburide (Diabeta) who is dizzy and sweaty The client taking glyburide (Diabeta) who is dizzy and sweaty has symptoms consistent with hypoglycemia and should be assessed first because this client displays the most serious adverse effect of antidiabetic medications. Although the client taking repaglinide who has nausea and back pain requires assessment, the client taking glyburide takes priority. Metformin may cause abdominal cramping and diarrhea, but the client taking it does not require immediate assessment. Ankle swelling is an expected side effect of pioglitazone.

The charge nurse is making client assignments for the day shift. Which client is best to assign to an LPN/LVN? a. Client who has just returned from having a kidney artery angioplasty b. Client with polycystic kidney disease who is having a kidney ultrasound c. Client who is going for a cystoscopy and cystourethroscopy d. Client with glomerulonephritis who is having a kidney biopsy

b. Client with polycystic kidney disease who is having a kidney ultrasound Kidney ultrasounds are noninvasive procedures without complications; the LPN/LVN can provide this care. A kidney artery angioplasty is an invasive procedure that requires postprocedure monitoring for complications, especially hemorrhage; a registered nurse is needed. Cystoscopy and cystourethroscopy are procedures that are associated with potentially serious complications such as bleeding and infection. These clients should be assigned to RN staff members. Kidney biopsy is associated with potentially serious complications such as bleeding, and this client should be assigned to RN staff members.

Which diagnostic test requires the nurse to know whether the client is allergic to iodine-based contrast? a. Arthroscopy b. Computed tomography (CT) c. Electromyography (EMG) d. Tomography

b. Computed tomography (CT) A CT scan creates three-dimensional images and may be done with iodine-based contrast. Arthroscopy involves inserting a fiberoptic tube into a joint for direct visualization of ligaments, menisci, and articular surfaces of the joint. An EMG evaluates diffuse or localized muscle weakness by testing nerve conduction. Tomography identifies locations, or "slices", for focus and blurs the images of other structures. Arthroscopy, EMG, and tomography do not use iodine-based contrast.

Which signs and symptoms indicate rejection of a transplanted kidney? (Select all that apply.) a. Blood urea nitrogen (BUN) 21 mg/dL, creatinine 0.9 mg/dL b. Crackles in the lung fields c. Temperature of 98.8° F (37.1° C) d. Blood pressure of 164/98 mm Hg e. 3+ edema of the lower extremities

b. Crackles in the lung fields d. Blood pressure of 164/98 mm Hg e. 3+ edema of the lower extremities Signs and symptoms of fluid retention (e.g., crackles in the lung fields and 3+ edema of the lower extremities) indicate transplant rejection. Increased blood pressure is also a symptom of transplant rejection. Increasing BUN and creatinine are symptoms of rejection; a BUN of 21 mg/dL and a creatinine of 0.9 mg/dL reflect normal values. Fever, not normothermia, is symptomatic of transplant rejection.

When assessing a client with acute glomerulonephritis, which finding causes the nurse to notify the provider? a. Purulent wound on the leg b. Crackles throughout the lung fields c. History of diabetes d. Cola-colored urine

b. Crackles throughout the lung fields Crackles indicate fluid overload resulting from kidney damage; shortness of breath and dyspnea are typically associated. The provider should be notified of this finding. Glomerulonephritis may result from infection (e.g., purulent wound); it is not an emergency about which to notify the provider. The history of diabetes would have been obtained on admission. Dark urine is expected in glomerulonephritis.

Care of the older adult may be affected by which physiologic change in the musculoskeletal system? a. Regeneration of cartilage b. Decreased range of motion (ROM) c. Increased bone density d. Narrower gait

b. Decreased range of motion (ROM) Decreased ROM occurs in older adults, and they may need assistance with self-care skills. Cartilage degeneration is an age-related change that occurs in the musculoskeletal system. Decreased bone density occurs with musculoskeletal system aging, and porous bones are more likely to fracture. The older adult experiences kyphotic posture, widened gait, and a shift in the center of gravity.

Which clinical manifestation in a client with pyelonephritis indicates that treatment has been effective? a. Decreased urine output b. Decreased white blood cells in urine c. Increased red blood cell count d. Increased urine specific gravity

b. Decreased white blood cells in urine A decreased presence of white blood cells in the urine indicates the eradication of infection. A decreased urine output, an increased red blood cell count, and increased urine specific gravity are not symptoms of pyelonephritis.

What are the typical clinical manifestations of anemia? (Select all that apply.) a. Decreased breath sounds b. Dyspnea on exertion c. Elevated temperature d. Fatigue e. Pallor f. Tachycardia

b. Dyspnea on exertion d. Fatigue e. Pallor f. Tachycardia Difficulty breathing—especially with activity—is common with anemia. Lower levels of hemoglobin carry less O2 to the cells of the body. Fatigue is a classic symptom of anemia; lowered O2 levels contribute to a faster pulse (i.e., cardiac rate) and tend to "wear out" a client's energy. Lowered O2 levels deliver less oxygen to all cells, making clients with anemia pale—especially their ears, nail beds, palms, and conjunctivae and around the mouth. Respiratory problems with anemia do not include changes in breath sounds; dyspnea and decreased oxygen saturation levels are present. Skin is cool to the touch, and an intolerance to cold is noted; elevated temperature would signify something additional, such as infection.

A client has just been diagnosed with diabetes. Which factor is most important for the nurse to assess in the client before providing instruction about the disease and its management? a. Current lifestyle b. Educational and literacy level c. Sexual orientation d. Current energy level

b. Educational and literacy level A large amount of information must be synthesized; typically written instructions are given. The client's educational and literacy level is essential information. Although lifestyle should be taken into account, it is not the priority. Sexual orientation will have no bearing on the ability of the client to provide self-care. Although energy level will influence the ability to exercise, it is not essential.

A client diagnosed with stress incontinence is started on propantheline (Pro-Banthine). What interventions does the nurse suggest to alleviate the side effects of this anticholinergic drug? (Select all that apply.) a. Take the drug at bedtime. b. Encourage increased fluids. c. Increase fiber intake. d. Limit the intake of dairy products. e. Use hard candy for dry mouth.

b. Encourage increased fluids. c. Increase fiber intake. e. Use hard candy for dry mouth. Anticholinergics cause constipation; increasing fluids and fiber intake will help with this problem. Anticholinergics also cause extreme dry mouth, which can be alleviated with using hard candy to moisten the mouth. Taking the drug at night and limiting dairy products will not have an effect on the complications encountered with propantheline.

A client admitted with hyperthyroidism is fidgeting with the bedcovers and talking extremely fast. What does the nurse do next? a. Calls the provider b. Encourages the client to rest c. Immediately assesses cardiac status d. Tells the client to slow down

b. Encourages the client to rest The client with hyperthyroidism often has wide mood swings, irritability, decreased attention span, and manic behavior. The nurse should accept the client's behavior and provide a calm, quiet, and comfortable environment. Because the client's behavior is expected, there is no need to call the provider. Monitoring the client's cardiac status is part of the nurse's routine assessment. Telling the client to slow down is unsupportive and unrealistic.

Which intervention most effectively protects a client with thrombocytopenia? a. Avoiding the use of dentures b. Encouraging the use of an electric shaver c. Taking rectal temperatures d. Using warm compresses on trauma sites

b. Encouraging the use of an electric shaver The client with thrombocytopenia should be advised to use an electric shaver instead of a razor. Any small cuts or nicks can cause problems because of the prolonged clotting time. Dentures may be used by clients with thrombocytopenia as long as they fit properly and do not rub. To prevent rectal trauma, rectal thermometers should not be used. Oral or tympanic temperatures should be taken. Ice (not heat) should be applied to areas of trauma.

The nurse is caring for an older adult client diagnosed with osteomalacia. The nurse anticipates that the health care provider will request which medication? a. Ascorbic acid (vitamin C) b. Ergocalciferol (Calciferol) c. Phenytoin (Dilantin) d. Prednisone (Deltasone)

b. Ergocalciferol (Calciferol) Osteomalacia is loss of bone related to vitamin D deficiency. The major treatment for osteomalacia is vitamin D in an active form such as ergocalciferol. Ascorbic acid (vitamin C) is not indicated for treatment of osteomalacia, which is related to vitamin D deficiency. Phenytoin interferes with the metabolism of vitamin D. Prednisone is a glucocorticoid for treatment of inflammatory disorders and is not indicated in the treatment of osteomalacia.

The nurse is caring for a client with sickle cell disease. Which action is most effective in reducing the potential for sepsis in this client? a. Administering prophylactic drug therapy b. Frequent and thorough handwashing c. Monitoring laboratory values to look for abnormalities d. Taking vital signs every 4 hours, day and night

b. Frequent and thorough hand washing Prevention and early detection strategies are used to protect the client in sickle cell crisis from infection. Frequent and thorough handwashing is of the utmost importance. Drug therapy is a major defense against infections that develop in the client with sickle cell disease, but is not the most effective way that the nurse can reduce the potential for sepsis. Continually assessing the client for infection and monitoring the daily complete blood count with differential white blood cell count is early detection, not prevention. Taking vital signs every 4 hours will help with early detection of infection, but is not prevention.

To best determine how well a client with diabetes mellitus is controlling blood glucose, which test does the nurse monitor? a. Fasting blood glucose b. Glycosylated hemoglobin (HbA1c) c. Oral glucose tolerance test d. Urine glucose level

b. Glycosylated hemoglobin (HbA1c) Glycosylated hemoglobin indicates the average blood glucose over several months and is the best indicator of overall blood glucose control. Fasting blood glucose can be used to monitor glucose control, but it is not the best method (although this may be the method that clients are most familiar with). Oral glucose testing and urine glucose levels look at one period of time and are not the best methods to look at overall effectiveness of treatment.

The nurse plans to use which tool to measure joint range of motion (ROM)? a. Doppler device b. Goniometer c. Reflex hammer d. Tonometer

b. Goniometer A goniometer provides an exact measurement of flexion and extension or joint ROM. A Doppler device is used to check and find pulses. A reflex hammer is used to test and elicit reflexes and is used in neurologic examinations. A tonometer is used to measure tension or pressure in the eye.

A client is receiving immune-suppressive therapy after kidney transplantation. Which measure for infection control is most important for the nurse to implement? a. Adherence to therapy b. Handwashing c. Monitoring for low-grade fever d. Strict clean technique

b. Handwashing The most important infection control measure for the client receiving immune-suppressive therapy is handwashing. Adherence to therapy and monitoring for low-grade fever are important, but are not infection control measures. The nurse should practice aseptic technique for this client, not simply clean technique.

The client is taking fludrocortisone (Florinef) for adrenal hypofunction. The nurse instructs the client to report which symptom while taking this drug? a. Anxiety b. Headache c. Nausea d. Weight loss

b. Headache A side effect of fludrocortisone is hypertension. New onset of headache should be reported, and the client's blood pressure should be monitored. Anxiety is not a side effect of fludrocortisone and is not associated with adrenal hypofunction. Nausea is associated with adrenal hypofunction; it is not a side effect of fludrocortisone. Sodium-related fluid retention and weight gain, not loss, are possible with fludrocortisone therapy.

A newly admitted client has an elevated reticulocyte count. Which disorder does the nurse suspect in this client? a. Aplastic anemia b. Hemolytic anemia c. Infectious process d. Leukemia

b. Hemolytic anemia An elevated reticulocyte count in an anemic client indicates that the bone marrow is responding appropriately to a decrease in the total red blood cell (RBC) mass and is prematurely destroying RBCs. Therefore, more immature RBCs are in circulation. Aplastic anemia is associated with a low reticulocyte count. A high white blood cell count is expected in clients with infection. A low white blood cell count is expected in clients with leukemia.

Which factor is an indicator for a diagnosis of hydronephrosis? a. History of nocturne b. History of urinary stones c. Recent weight loss d. Urinary incontinence

b. History of urinary stones Causes of hydronephrosis or hydroureter include tumors, stones, trauma, structural defects, and fibrosis. Nocturia is a key feature of polycystic kidney disease and pyelonephritis, but it is not associated with hydronephrosis. Recent weight loss and urinary incontinence may be factors in renal cell carcinoma, but are not associated with hydronephrosis.

A client had a parathyroidectomy 18 hours ago. Which finding requires immediate attention? a. Edema at the surgical site b. Hoarseness c. Pain on moving the head d. Sore throat

b. Hoarseness Hoarseness or stridor is an indication of respiratory distress and requires immediate attention. Edema at the surgical site of any surgery is an expected finding. Pain when the client moves the head or attempts to lift the head off the bed is an expected finding after a parathyroidectomy. Any time a client has been intubated for surgery, a sore throat is a common occurrence in the postoperative period. This is especially true for clients who have had surgery involving the neck.

In type 1 diabetes, insulin injections are necessary to maintain which action between insulin and glucose? a. Glucose intolerance b. Homeostasis c. Insulin intolerance d. Negative feedback

b. Homeostasis Insulin injections maintain homeostasis, or normal balance, between insulin and glucose in the client with type 1 diabetes. Type 1 diabetes is a lack of insulin production, not glucose intolerance, and requires frequent doses of insulin. Negative feedback does not occur in type 1 diabetes because of lack of insulin.

Which statement is true about hormones and their receptor sites? a. Hormone activity is dependent only on the function of the receptor site. b. Hormones need a specific receptor site to work. c. Hormones need to be plasma-bound to activate the receptor site. d. Hormone stores are available for activation until needed.

b. Hormones need a specific receptor site to work. In general, each receptor site type is specific for only one hormone. Hormone receptor actions work in a "lock and key" manner, in that only the correct hormone (key) can bind to and activate the receptor site (lock). Hormones travel through the blood to all body areas, but exert their actions only on target tissues. Not all hormones are plasma-bound; for example, thyroid hormones are plasma protein-bound, whereas posterior pituitary hormones are transported by axons. Only certain cells manufacture specific hormones and store the hormones in vesicles.

A 32-year-old client is recovering from a sickle cell crisis. His discomfort is controlled with pain medications and he is to be discharged. What medication does the nurse expect to be prescribed for him before his discharge? a. Heparin (Heparin) b. Hydroxyurea (Droxia) c. Tissue plasminogen activator (t-PA) d. Warfarin (Coumadin)

b. Hydroxyurea (Droxia) Hydroxyurea (Droxia) has been used successfully to reduce sickling of cells and pain episodes associated with sickle cell disease (SCD). Clients with SCD are not prescribed anticoagulants such as heparin or warfarin (Coumadin). t-PA is used as a "clot buster" in clients who have had ischemic strokes.

The nurse is transfusing 2 units of packed red blood cells to a postoperative client. What post-transfusion electrolyte imbalance does the nurse want to rule out? a. Hypercalcemia b. Hyperkalemia c. Hypomagnesemia d. Hyponatremia

b. Hyperkalemia During transfusion, some cells are damaged. These cells release potassium, thus raising the client's serum potassium level (hyperkalemia). This complication is especially common with packed cells and whole-blood products. High serum calcium levels, low magnesium levels, or low sodium levels are not expected with blood transfusions.

A client is hospitalized for pituitary function testing. Which nursing action included in the client's plan of care will be most appropriate for the RN to delegate to the LPN/LVN? a. Assess the client for clinical manifestations of hypopituitarism. b. Inject regular insulin for the growth hormone stimulation test. c. Palpate the thyroid gland for size and firmness. d. Teach the client about the adrenocorticotropic hormone stimulation test.

b. Inject regular insulin for the growth hormone stimulation test. Injection of insulin is within the LPN/LVN scope of practice. Client assessment for clinical manifestations of hypopituitarism, palpating the thyroid gland, and client education are complex skills requiring training and expertise, and are best performed by an RN.

A client has undergone a transsphenoidal hypophysectomy. Which intervention does the nurse implement to avoid increasing intracranial pressure (ICP) in the client? a. Encourages the client to cough and deep-breathe b. Instructs the client not to strain during a bowel movement c. Instructs the client to blow the nose for postnasal drip d. Places the client in the Trendelenburg position

b. Instructs the client not to strain during a bowel movement Straining during a bowel movement increases ICP and must be avoided. Laxatives may be given and fluid intake encouraged to help with this. Although deep breathing is encouraged, the client must avoid coughing early after surgery because this increases pressure in the incision area and may lead to a cerebrospinal fluid (CSF) leak. If the client has postnasal drip, he or she must inform the nurse and not blow the nose; postnasal drip may indicate leakage of CSF. The head of the bed must be elevated after surgery.

A recently admitted client who is in sickle cell crisis requests "something for pain." What does the nurse administer? a. Intramuscular (IM) morphine sulfate b. Intravenous (IV) hydromorphone (Dilaudid) c. Oral ibuprofen (Motrin) d. Oral morphine sulfate (MS-Contin)

b. Intravenous (IV) hydromorphone (Dilaudid) The client needs IV pain relief, and it should be administered on a routine schedule (i.e., before the client has to request it). Morphine is not administered intramuscularly (IM) to clients with sickle cell disease (SCD). In fact, all IM injections are avoided because absorption is impaired by poor perfusion and sclerosed skin. Nonsteroidal anti-inflammatory drugs may be used for clients with SCD for pain relief once their pain is under control; however, in a crisis, this choice of analgesic is not strong enough. Moderate pain may be treated with oral opioids, but this client is in a sickle cell crisis; IV analgesics should be used until his or her condition stabilizes.

A client has been diagnosed with hypothyroidism. What medication is usually prescribed to treat this disorder? a. Atenolol (Tenormin) b. Levothyroxine sodium (Synthroid) c. Methimazole (Tapazole) d. Propylthiouracil

b. Levothyroxine sodium (Synthroid) Levothyroxine is a synthetic form of thyroxine (T4) that is used to treat hypothyroidism. Atenolol is a beta blocker that is used to treat cardiovascular disease. Methimazole and propylthiouracil are used to treat hyperthyroidism.

Which medication is most effective in slowing the progression of kidney failure in a client with chronic kidney disease? a. Diltiazem (Cardizem) b. Lisinopril (Zestril) c. Clonidine (Catapres) d. Doxazosin (Cardura)

b. Lisinopril (Zestril) Angiotensin-converting enzyme inhibitors such as lisinopril appear to be the most effective drugs to slow the progression of kidney failure. Calcium channel blockers such as diltiazem may indirectly prevent kidney disease by controlling hypertension, but are not specific to slowing progression of kidney disease. Vasodilators such as clonidine and doxazosin control blood pressure, but do not specifically protect from kidney disease.

A client is suspected of having muscular dystrophy (MD). Which laboratory test result does the nurse anticipate with this disease? a. Decreased serum creatine kinase (CK) level b. Moderately elevated aspartate aminotransferase (AST) c. Decreased alkaline phosphatase (ALP) d. Decreased skeletal muscle creatine kinase (CK-MM) level

b. Moderately elevated aspartate aminotransferase (AST) The AST level is moderately elevated (three to five times normal) in certain musculoskeletal diseases, such as MD. The CK level is elevated in musculoskeletal diseases such as MD. ALP is an enzyme normally present in blood, and the concentration of ALP increases with bone or liver damage; it is not associated with MD. A decreased CK-MM level is not associated with MD.

A client recently admitted with hyperparathyroidism has a very high urine output. Of these actions, what does the nurse do next? a. Calls the health care provider b. Monitors intake and output c. Performs an immediate cardiac assessment d. Slows the rate of IV fluids

b. Monitors intake and output Diuretic and hydration therapies are used most often for reducing serum calcium levels in clients with hyperparathyroidism. Usually, a diuretic that increases kidney excretion of calcium is used together with IV saline in large volumes to promote renal calcium excretion. The health care provider does not need to be notified in this situation, given the information available in the question. Cardiac assessment is part of the nurse's routine evaluation of the client. Slowing the rate of IV fluids is contraindicated because the client will become dehydrated due to the use of diuretics to increase kidney excretion of calcium.

What are serious side effects of antiviral agents prescribed for a client with acute myelogenous leukemia? (Select all that apply.) a. Cardiomyopathy b. Nephrotoxicity c. Ototoxicity d. Stroke e. Diarrhea

b. Nephrotoxicity c. Ototoxicity Antiviral agents, although helpful in combating severe infection, have serious side effects, especially nephrotoxicity and ototoxicity. Cardiomyopathy and stroke are not serious side effects of antiviral agents. Diarrhea is a mild side effect associated with antibiotic therapy. Awarded 0.5 points out of 1.0 possible points.

Which percussion technique does the nurse use to assess a client who reports flank pain? a. Place outstretched fingers over the flank area and percuss with the fingertips. b. Place one hand with the palm down flat over the flank area and use the other fisted hand to thump the hand on the flank. c. Place one hand with the palm up over the flank area and cup the other hand to percuss the hand on the flank. d. Quickly tap the flank area with cupped hands.

b. Place one hand with the palm down flat over the flank area and use the other fisted hand to thump the hand on the flank. While the client assumes a sitting, side-lying, or supine position, the nurse forms one of the hands into a clenched fist. The other hand is placed flat over the costovertebral (CVA) angle of the client. Then, a firm thump is quickly delivered to the hand over the CVA area. Percussion is not appropriate for flank pain. Placing one hand palm up is not the correct technique. Percussion therapy, not assessment, involves tapping the flank area.

The nurse is reviewing complete blood count (CBC) data for a 76-year-old client. Which decreased value causes concern because it is not age-related? a. Hemoglobin level b. Platelet (thrombocyte) count c. Red blood cell (RBC) count d. White blood cell (WBC) response

b. Platelet (thrombocyte) count Platelet counts do not generally change with age. Hemoglobin levels in men and women fall after middle age; iron-deficient diets may play a role in this reduction. Total RBC and WBC counts (especially lymphocyte counts) are lower in older adults. The WBC count does not rise as high in response to infection in older adults as it does in younger people.

The RN is caring for a client who has just had a kidney biopsy. Which action does the nurse perform first? a. Obtain blood urea nitrogen (BUN) and creatinine. b. Position the client supine. c. Administer pain medications. d. Check urine for hematuria.

b. Position the client supine. The client is positioned supine for several hours after a kidney biopsy to decrease the risk for hemorrhage. BUN and creatinine would be obtained before the procedure is performed. Only local discomfort should be noted around the procedure site; severe pain would indicate hematoma. Although pink urine may develop, the nurse should position the client to prevent bleeding first; the other actions are appropriate after this procedure, but do not need to be done immediately after the biopsy.

A client presents to the emergency department with acute adrenal insufficiency and the following vital signs: P 118 beats/min, R 18 breaths/min, BP 84/44 mm Hg, pulse oximetry 98%, and T 98.8° F oral. Which nursing intervention is the highest priority for this client? a. Administering furosemide (Lasix) b. Providing isotonic fluids c. Replacing potassium losses d. Restricting sodium

b. Providing isotonic fluids Providing isotonic fluid is the priority intervention because this client's vital signs indicate volume loss that may be caused by nausea and vomiting and may accompany acute adrenal insufficiency. Isotonic fluids will be needed to administer IV medications such as hydrocortisone. Furosemide is a loop diuretic, which this client does not need. Potassium is normally increased in acute adrenal insufficiency, but potassium may have been lost if the client has had diarrhea; laboratory work will have to be obtained. Any restrictions, including sodium, should not be started without obtaining laboratory values to establish the client's baseline.

The nurse assesses multiple clients who are receiving transfusions of blood components. Which assessment indicates the need for the nurse's immediate action? a. A partial thromboplastin time (PTT) that is 1.2 times normal in a client who received a transfusion of fresh-frozen plasma (FFP) b. Respiratory rate of 36 breaths/min in a client receiving red blood cells c. Sleepiness in a client who received diphenhydramine (Benadryl) as a premedication d. Temperature of 99.1° F (37.3° C) for a client with a platelet transfusion

b. Respiratory rate of 36 breaths/min in a client receiving red blood cells An increased respiratory rate indicates a possible hemolytic transfusion reaction; the nurse should quickly stop the transfusion and assess the client further. Because FFP is not usually given until the PTT is 1.5 times above normal, a PTT that is 1.2 times normal indicates that the FFP has had the desired response. Sleepiness is expected when Benadryl is administered. Temperature elevations are not an indication of an allergic reaction to a platelet transfusion, although the nurse may administer acetaminophen (Tylenol) to decrease the fever.

Which information about a client who was admitted with pelvic and bilateral femoral fractures after being crushed by a tractor is most important for the nurse to report to the health care provider? a. Thighs have multiple oozing abrasions. b. Serum potassium level is 7 mEq/L. c. The client is describing pain as level 4 (0-to-10 scale). d. Hemoglobin level is 12.0 g/dL.

b. Serum potassium level is 7 mEq/L. The elevated potassium level may indicate that the client has rhabdomyolysis and acute tubular necrosis caused by the crush injury. Further assessment and treatment are needed immediately to prevent further kidney damage or cardiac dysrhythmias. Thighs having multiple oozing abrasions with a pain level of 4 are not unusual for a client with this type of injury. A hemoglobin level of 12.0 g/dL is a normal finding.

Which interventions are helpful in preventing bladder cancer? (Select all that apply.) a. Drinking 2½ liters of fluid a day b. Showering after working with or around chemicals c. Stopping the use of tobacco d. Using pelvic floor muscle exercises e. Wearing a lead apron when working with chemicals f. Wearing gloves and a mask when working around chemicals and fumes

b. Showering after working with or around chemicals c. Stopping the use of tobacco f. Wearing gloves and a mask when working around chemicals and fumes Certain chemicals (e.g., those used by professional hairdressers) are known to be carcinogenic in evaluating the risk for bladder cancer. Protective gear is advised. Bathing after exposure to them is advisable. Tobacco use is one of the highest, if not the highest, risk factor in the development of bladder cancer. Increasing fluid intake is helpful for some urinary problems such as urinary tract infection, but no correlation has been noted between fluid intake and bladder cancer risk. Using pelvic floor muscle strengthening (Kegel) exercises is helpful with certain types of incontinence, but no data show that these exercises prevent bladder cancer. Precautions should be taken when working with chemicals; however, lead aprons are used to protect from radiation.

What is the primary role of the nurse when caring for an adult client with muscular dystrophy (MD)? a. Pain management b. Supportive care c. Teaching the importance of keeping appointments d. Advocating for the client and the family

b. Supportive care Management of the client with MD is supportive and involves the entire health care team. Coordinating pain management is not the nurse's primary role for the adult client with MD. The nurse's role does not focus on whether the client keeps appointments; this would be more important for clients who, for example, are receiving intermittent chemotherapy. The nurse is always an advocate for all clients and families, but this is not the nurse's primary role when caring for the client with MD.

When caring for a client with acute kidney injury and a temporary subclavian hemodialysis catheter, which assessment finding does the nurse report to the provider? a. Mild discomfort at the insertion site b. Temperature 100.8° F c. 1+ ankle edema d. Anorexia

b. Temperature 100.8° F Infection is a major complication of temporary catheters. All symptoms of infection, including fever, must be reported to the provider because the catheter may have to be removed. Mild discomfort at the insertion site is expected with a subclavian hemodialysis catheter. During acute injury, oliguria with resulting fluid retention is expected. Rising blood urea nitrogen may result in anorexia, nausea, and vomiting.

Which sign or symptom, when assessed in a client with chronic glomerulonephritis (GN), warrants a call to the health care provider? a. Mild proteinuria b. Third heart sound (S3) c. Serum potassium of 5.0 mEq/L d. Itchy skin

b. Third heart sound (S3) S3 indicates fluid overload secondary to failing kidneys; the provider should be notified and instructions obtained. Mild proteinuria is an expected finding in GN. A serum potassium of 5.0 mEq/L reflects a normal value; intervention would be needed for hyperkalemia. Although itchy skin may be present as kidney function declines, it is not a priority over fluid excess

The nurse in the urology clinic is providing teaching for a female client with cystitis. Which instructions does the nurse include in the teaching plan? (Select all that apply.) a. Cleanse the perineum from back to front after using the bathroom. b. Try to take in 64 ounces of fluid each day. c. Be sure to complete the full course of antibiotics. d. If urine remains cloudy, call the clinic. e. Expect some flank discomfort until the antibiotic has worked.

b. Try to take in 64 ounces of fluid each day. c. Be sure to complete the full course of antibiotics. d. If urine remains cloudy, call the clinic. Between 64 and 100 ounces (2 to 3 liters) of fluid should be taken daily to dilute bacteria and prevent infection. Not completing the course of antibiotics could suppress the bacteria, but would not destroy all bacteria, causing the infection to resurface. For persistent symptoms of infection, the client should contact the provider. The perineal area should be cleansed from front to back or "clean to dirty" to prevent infection. Cystitis produces suprapubic symptoms; flank pain occurs with infection or inflammation of the kidney.

A client has a grade III compound fracture of the right tibia. To prevent infection, which intervention does the nurse implement? a. Apply bacitracin (Neosporin) ointment to the site daily with a sterile cotton swab. b. Use strict aseptic technique when cleaning the site. c. Leave the site open to the air to keep it dry. d. Assist the client to shower daily and pat the wound site dry.

b. Use strict aseptic technique when cleaning the site. Using aseptic technique is the best way to prevent infection. Chlorhexidine (Hibiclens), 2 mg/mL solution, is the better cleansing solution for pin site care, not Neosporin ointment. A wound of this type should be kept covered, not left open to the air. The wound site of a compound fracture must not be exposed to a shower; this practice violates maintaining aseptic technique.

When assessing a client with acute pyelonephritis, which findings does the nurse anticipate will be present? (Select all that apply.) a. Suprapubic pain b. Vomiting c. Chills d. Dysuria e. Oliguria

b. Vomiting c. Chills d. Dysuria Nausea and vomiting are symptoms of acute pyelonephritis. Chills along with fever may also occur, as well as burning (dysuria), urgency, and frequency. Suprapubic pain is indicative of cystitis, not kidney infection (pyelonephritis). Oliguria is related to kidney impairment from severe or long-standing pyelonephritis.

The nurse is completing an admission assessment on a client scheduled for arthroscopic knee surgery. Which information will be most essential for the nurse to report to the health care provider? a. Knee pain at a level of 9 (0-to-10 scale) b. Warm, red, and swollen knee c. Allergy to shellfish and iodine d. Previous surgery on the other knee

b. Warm, red, and swollen knee Swelling, heat, and redness may indicate infection in the knee joint, which would indicate a need to cancel the procedure. Having knee pain before surgery is not unexpected but will not affect whether the client will have surgery. Allergy to shellfish and iodine will need to be reported, but also will not affect whether the client will have surgery. Having previous surgery on the other knee does not preclude the client from having this surgery.

The nurse is taking the history of an adult female client. Which factor places the client at risk for osteoporosis? a. Consuming 12 ounces of carbonated beverages daily b. Working at a desk and playing the piano for a hobby c. Having a hysterectomy and taking estrogen replacement therapy d. Consuming one alcoholic drink per week

b. Working at a desk and playing the piano for a hobby Sedentary lifestyle and prolonged immobility produce rapid bone loss. The client would have to consume large amounts of carbonated beverages daily (over 40 ounces) for this to be a risk factor for osteoporosis. Maintaining estrogen levels reduces the risk for osteoporosis. Alcohol has a direct toxic effect on bone tissue, resulting in decreased bone formation and increased bone resorption. For those who have excessive alcohol intake, alcohol calories decrease hunger and the need to take in adequate quantities of nutrients. This client's alcoholic intake is not high, so it is not a risk factor.

A hematology unit is staffed by RNs, LPN/LVNs, and unlicensed assistive personnel (UAP). When the nurse manager is reviewing documentation of staff members, which entry indicates that the staff member needs education about his or her appropriate level of responsibility and client care? a. "Abdominal pain relieved by morphine 4 mg IV; client resting comfortably and denies problems. B.C., RN" b. "Ambulated in hallway for 40 feet and denies shortness of breath at rest or with ambulation. T.Y., LPN" c. "Client reporting increased shortness of breath; oxygen increased to 4 L by nasal cannula. M.N., UAP" d. "Vital signs 37.0° C, heart rate 60, respiratory rate 20, blood pressure 110/68, and oximetry 98% on room air. L.D., UAP"

c. "Client reporting increased shortness of breath; oxygen increased to 4 L by nasal cannula. M.N., UAP" Determination of the need for oxygen and administration of oxygen should be done by licensed nurses who have the education and scope of practice required to administer it. All other documentation entries reflect appropriate delegation and assignment of care.

Which instruction does the nurse give a client who needs a clean-catch urine specimen? a. "Save all urine for 24 hours." b. "Use the sponges to cleanse the urethra, and then initiate voiding directly into the cup." c. "Do not touch the inside of the container." d. "You will receive an isotope injection, then I will collect your urine."

c. "Do not touch the inside of the container." A clean-catch specimen is used to obtain urine for culture and sensitivity of organisms present; contamination by the client's hands will render the specimen invalid and alter results. Saving urine for 24 hours is not necessary for a midstream clean-catch urine specimen. After cleaning, the client should initiate voiding into the commode, then stop and resume voiding into the container. Only 1 ounce (30 mL) is needed; the remainder of the urine may be discarded into the commode. A midstream collection further removes secretions and bacteria because urine flushes the distal portion of the internal urethra. A clean-catch specimen for culture does not require an injection of isotope, simply cleansing of the perineum.

A client experiencing kyphosis appears withdrawn and does not initiate any conversation with the nurse when medications are given each day. Which statement by the nurse is most supportive of this client? a. "It is normal to feel depressed at times about your condition. You have my support." b. "You could exercise more often to build up your strength and endurance." c. "How do you feel about the pain in your spine? I am here if you want to talk." d. "What does your family say to you? Try talking to them."

c. "How do you feel about the pain in your spine? I am here if you want to talk." Asking the client about his or her pain and offering to listen is most supportive because it allows the client to discuss his or her feelings and informs the client that the nurse is available to listen. Telling the client that it is normal to feel depressed is a leading statement and is not supportive; the client may not be depressed. Suggesting that the client exercise more often is not a supportive statement; it avoids the opportunity to support the client and diverts the subject to exercise. Asking what the client's family has to say is not supportive because it is the nurse's way of avoiding the issue.

The nurse is assessing an adult client's endurance in performing activities of daily living (ADLs). What question does the nurse ask the client? a. "Can you prepare your own meals?" b. "Has your weight changed by 5 pounds or more this year?" c. "How is your energy level compared with last year?" d. "What medications do you take daily, weekly, and monthly?"

c. "How is your energy level compared with last year?" Asking the client how his or her energy level compares with last year is an activity exercise question that correctly assesses endurance compared with self-assessment in the past. It is most likely to provide data about the client's ability and endurance for ADLs. The client may never have been able to prepare his or her own meals, and the ability to prepare meals does not really address endurance. The question about weight change addresses nutrition and metabolic needs, rather than ADL performance. The question about how often the client takes medication addresses nutrition and metabolic needs and focuses on health maintenance through the use of drugs, not on the client's ability to perform ADLs.

The nurse is instructing an older adult female client about interventions to decrease the risk for cystitis. Which client comment indicates that the teaching was effective? a. "I must avoid drinking carbonated beverages." b. "I need to douche vaginally once a week." c. "I should drink 2½ liters of fluid every day." d. "I will not drink fluids after 8 PM each evening."

c. "I should drink 2½ liters of fluid every day." Drinking 2½ liters of fluid a day flushes out the urinary system and helps reduce the risk for cystitis. Avoiding carbonated beverages is not necessary to reduce the risk for cystitis. Douching is not a healthy behavior because it removes beneficial organisms as well as the harmful ones. Avoiding fluids after 8:00 p.m. would help prevent nocturia but not cystitis. It is recommended that clients with incontinence problems limit their late-night fluid intake to 120 mL.`

The nurse is providing discharge instructions to a client on spironolactone (Aldactone) therapy. Which comment by the client indicates a need for further teaching? a. "I must call the provider if I am more tired than usual." b. "I need to increase my salt intake." c. "I should eat a banana every day." d. "This drug will not control my heart rate."

c. "I should eat a banana every day." Spironolactone increases potassium levels, so potassium supplements and foods rich in potassium, such as bananas, should be avoided to prevent hyperkalemia. While taking spironolactone, symptoms of hyponatremia such as drowsiness and lethargy must be reported; the client may need increased dietary sodium. Spironolactone will not have an effect on the client's heart rate.

The nurse is teaching a client how to provide a clean-catch urine specimen. Which statement by the client indicates that teaching was effective? a. "I must clean with the wipes and then urinate directly into the cup." b. "I will have to drink 2 liters of fluid before providing the sample." c. "I'll start to urinate in the toilet, stop, and then urinate into the cup." d. "It is best to provide the sample while I am bathing."

c. "I'll start to urinate in the toilet, stop, and then urinate into the cup." To provide a clean-catch urine sample, the client should initiate voiding, then stop, then resume voiding into the container. A midstream collection further removes secretions and bacteria because urine flushes the distal portion of the internal urethra. Although cleaning with wipes before providing a clean-catch urine sample is proper procedure, a step is missing. It is not necessary to drink 2 liters of fluid before providing a clean-catch urine sample. Providing a clean-catch urine sample does not involve bathing.

The nurse is teaching a client about thyroid replacement therapy. Which statement by the client indicates a need for further teaching? a. "I should have more energy with this medication." b. "I should take it every morning." c. "If I continue to lose weight, I may need an increased dose." d. "If I gain weight and feel tired, I may need an increased dose."

c. "If I continue to lose weight, I may need an increased dose." Weight loss indicates a need for a decreased dose, not an increased dose. One of the symptoms of hypothyroidism is lack of energy; thyroid replacement therapy should help the client have more energy. The correct time to take thyroid replacement therapy is in the morning. If the client is gaining weight and continues to feel tired, that is an indication that the dose may need to be increased.

A client is scheduled to undergo kidney transplant surgery. Which teaching point does the nurse include in the preoperative teaching session? a. "Your diseased kidneys will be removed at the same time the transplant is performed." b. "The new kidney will be placed directly below one of your old kidneys." c. "It is essential for you to wash your hands and avoid people who are ill." d. "You will receive dialysis the day before surgery and for about a week after."

c. "It is essential for you to wash your hands and avoid people who are ill." Anti-rejection medications increase the risks for infection, sepsis, and death. Strict aseptic technique and handwashing are essential. Unless severely infected, the client's kidneys are left in place and the graft is placed in the iliac fossa. Dialysis is performed the day before surgery; after the surgery, the new kidney should begin to make urine.

A client on anticoagulant therapy is being discharged. Which statement indicates that the client has a correct understanding of this therapy's purpose or action? a. "It is to dissolve blood clots." b. "It might cause me to get injured more often." c. "It should prevent my blood from clotting." d. "It will thin my blood."

c. "It should prevent my blood from clotting." Anticoagulants work by interfering with one or more steps involved in the blood clotting cascade. Thus, these agents prevent new clots from forming and limit or prevent extension of formed clots. Anticoagulants do not dissolve clots, fibrinolytics do. Anticoagulants do not cause more injuries, but may cause more bleeding and bruising when the client is injured. Anticoagulants do not cause any change in the thickness or viscosity of the blood.

A client expresses fear and anxiety over the life changes associated with diabetes, stating, "I am scared I can't do it all and I will get sick and be a burden on my family." What is the nurse's best response? a. "It is overwhelming, isn't it?" b. "Let's see how much you can learn today, so you are less nervous." c. "Let's tackle it piece by piece. What is most scary to you?" d. "Other people do it just fine."

c. "Let's tackle it piece by piece. What is most scary to you?" Suggesting the client tackle it piece by piece and asking what is most scary to him or her is the best response; this approach will allow the client to have a sense of mastery with acceptance. Referring to the illness as overwhelming is supportive, but is not therapeutic or helpful to the client. Trying to see how much the client can learn in one day may actually cause the client to become more nervous; an overload of information is overwhelming. Suggesting that other people handle the illness just fine is belittling and dismisses the client's concerns.

The nurse is caring for a client with bone cancer of the right hip who has undergone radical resection of the tumor and has received a prosthetic implant. Which client statement indicates effective coping after the procedure? a. "After I recover, I'll be just as strong as I was before the surgery." b. "I won't be able to go out in public like I did before." c. "Physical therapy and counseling will help me adjust to my prosthesis." d. "I'll be able to return to work and drive without assistance."

c. "Physical therapy and counseling will help me adjust to my prosthesis." The client stating that physical therapy and counseling will help him or her to adjust to the prosthesis illustrates effective coping and acceptance. The client expecting to be just as strong as before the surgery or expecting to return to work and drive without assistance reveals that the client is in denial of how surgery will affect his or her prognosis and activity. Avoiding going out in public suggests that the client is having difficulty coping and adjusting to his or her changed body image.

The clinic nurse is discharging a 20-year-old client who had a bone marrow aspiration performed. What does the nurse advise the client to do? a. "Avoid contact sports or activity that may traumatize the site for 24 hours." b. "Inspect the site for bleeding every 4 to 6 hours." c. "Place an ice pack over the site to reduce the bruising." d. "Take a mild analgesic, such as two aspirin, for pain or discomfort at the site."

c. "Place an ice pack over the site to reduce the bruising." Ice to the site will help limit bruising and tissue damage during the first 24 hours after the procedure. Contact sports and traumatic activity must be excluded for 48 hours, or 2 days. The client should carefully monitor the site every 2 hours for the first 24 hours after the procedure. A mild analgesic is appropriate, but it should be aspirin-free; acetaminophen (Tylenol) would be a good choice.

Discharge teaching has been provided for a client recovering from kidney transplantation. Which information indicates that the client understands the instructions? a. "I can stop my medications when my kidney function returns to normal." b. "If my urine output is decreased, I should increase my fluids." c. "The anti-rejection medications will be taken for life." d. "I will drink 8 ounces of water with my medications."

c. "The anti-rejection medications will be taken for life." Immune-suppressant therapy must be taken for life to prevent organ rejection. Adherence to immunosuppressive drugs is crucial to survival for clients with transplanted kidneys. Lack of adherence can lead to complications such as rejection, graft loss, return to dialysis, and death. Oliguria is a symptom of transplant rejection; the transplant team should be contacted immediately if this occurs. It is not necessary to take anti-rejection medication with 8 ounces of water.

A client with a low platelet count asks why platelets are important. How does the nurse answer? a. "Platelets make your blood clot." b. "Blood clotting is prevented by your platelets." c. "The clotting process begins with your platelets." d. "Your platelets finish the clotting process."

c. "The clotting process begins with your platelets." Platelets begin the blood clotting process by forming platelet plugs, but these platelet plugs are not clots and cannot provide complete hemostasis. Platelets do not clot blood; they are a part of the clotting process or cascade of coagulation. Platelets do not prevent the blood from clotting; rather they function to help blood form clots. Platelets do not finish the clotting process, they begin it.

A client with Cushing's disease begins to laugh loudly and inappropriately, causing the family in the room to be uncomfortable. What is the nurse's best response? a. "Don't mind this. The disease is causing this." b. "I need to check the client's cortisol level." c. "The disease can sometimes affect emotional responses." d. "Medication is available to help with this."

c. "The disease can sometimes affect emotional responses." The client may have neurotic or psychotic behavior as a result of high blood cortisol levels. Being honest with the family helps them to understand what is happening. Telling the family not to mind the laughter and that the disease is causing it is vague and minimizes the family's concern. This is the perfect opportunity for the nurse to educate the family about the disease. Cushing's disease is the hypersecretion of cortisol, which is abnormally elevated in this disease and, because the diagnosis has already been made, blood levels do not need to be redrawn. Telling the family that medication is available to help with inappropriate laughing does not assist them in understanding the cause of or the reason for the client's behavior.

A client with leukemia is being discharged from the hospital. After hearing the nurse's instructions to keep regularly scheduled follow-up provider appointments, the client says, "I don't have transportation." How does the nurse respond? a. "A pharmaceutical company might be able to help." b. "I might be able to take you." c. "The local American Cancer Society may be able to help." d. "You can take the bus."

c. "The local American Cancer Society may be able to help." Many local units of the American Cancer Society offer free transportation to clients with cancer, including those with leukemia. Suggesting a pharmaceutical company is not the best answer; drug companies typically do not provide this type of service. Although the nurse offering to take the client is compassionate, it is not appropriate for the nurse to offer the client transportation. Telling the client to take the bus is dismissive and does not take into consideration the client's situation (e.g., the client may live nowhere near a bus route).

A mother who is a carrier of muscular dystrophy (MD) has a daughter. The client asks the nurse what the daughter's genetic risk is for having MD. What is the nurse's best response? a. "Because you are a carrier of the MD gene, your daughter will develop MD." b. "She will not have MD nor will she be a carrier." c. "There is a 50% chance that your daughter may carry the gene." d. "Your daughter is X-linked dominant for the MD gene."

c. "There is a 50% chance that your daughter may carry the gene." MD is an X-linked recessive disorder, so the daughter of a mother who is a carrier has a 50% chance of carrying the gene. The daughter would only be a carrier of the disease; she would not have MD. Telling the client that the daughter will not have MD or will not be a carrier is only half true.

A 53-year-old postmenopausal woman reports "leaking urine" when she laughs, and is diagnosed with stress incontinence. What does the nurse tell the client about how certain drugs may be able to help with her stress incontinence? a. "They can relieve your anxiety associated with incontinence." b. "They help your bladder to empty." c. "They may be used to improve urethral resistance." d. "They decrease your bladder's tone."

c. "They may be used to improve urethral resistance." Bladder pressure is greater than urethral resistance; drugs may be used to improve urethral resistance. Relieving anxiety has not been shown to improve stress incontinence. No drugs have been shown to promote bladder emptying, and this is not usually the problem with stress incontinence. Emptying the bladder is accomplished by the individual or, if that is not possible, by using a catheter. Decreasing bladder tone would not be a desired outcome for a woman with incontinence.

The nurse is instructing a local community group about ways to reduce the risk for musculoskeletal injury. What information does the nurse include in the teaching plan? a. "Avoid contact sports." b. "Avoid rigorous exercise." c. "Wear helmets when riding a motorcycle." d. "Avoid driving in inclement weather."

c. "Wear helmets when riding a motorcycle." Those who ride motorcycles or bicycles should wear helmets to prevent head injury. Telling the general public to avoid contact sports or to avoid driving in inclement weather is not realistic. Telling the general public to avoid rigorous exercise is not only unrealistic, it is also opposed to what many health care professionals recommend to maintain health.

A diabetic client has a glycosylated hemoglobin (HbA1C) level of 9.4%. What does the nurse say to the client regarding this finding? a. "Keep up the good work." b. "This is not good at all." c. "What are you doing differently?" d. "You need more insulin."

c. "What are you doing differently?" Assessing the client's regimen or changes he or she may have made is the basis for formulating interventions to gain control of blood glucose. HbA1C levels for diabetic clients should be less than 7%; a value of 9.4% shows poor control over the past 3 months. Telling the client this is not good, although true, does not take into account problems that the client may be having with the regimen and sounds like scolding. Although it may be true that the client needs more insulin, an assessment of the client's regimen is needed before decisions are made about medications.

The nurse is educating a female client about hygiene measures to reduce her risk for urinary tract infection. What does the nurse instruct the client to do? a. "Douche—but only once a month." b. "Use only white toilet paper." c. "Wipe from front to back." d. "Wipe with the softest toilet paper available."

c. "Wipe from front to back." Wiping front to back keeps organisms in the stool from coming close to the urethra, which increases the risk for infection. Douching is an unhealthy behavior because it removes beneficial organisms as well as the harmful ones. White toilet paper helps prevent allergies, not infections. Using soft toilet paper does not prevent infection.

A client with type 2 diabetes has been admitted for surgery, and the health care provider has placed the client on insulin in addition to the current dose of metformin (Glucophage). The client wants to know the purpose of taking the insulin. What is the nurse's best response? a. "Your diabetes is worse, so you will need to take insulin." b. "You can't take your metformin while in the hospital." c. "Your body is under more stress, so you'll need insulin to support your medication." d. "You must take insulin from now on because the surgery will affect your diabetes."

c. "Your body is under more stress, so you'll need insulin to support your medication." Because of the stress of surgery and NPO status, short-term insulin therapy may be needed perioperatively for the client who uses oral antidiabetic agents. For those receiving insulin, dosage adjustments may be required until the stress of surgery subsides. No evidence suggests that the client's diabetes has worsened; however, surgery is stressful and may increase insulin requirements. Metformin may be taken in the hospital; however, not on days when the client is NPO for surgery. When the client returns to his or her previous health state, oral agents will be resumed.

The nurse is using a common scale to grade a client's muscle strength. The client is able to complete range of motion (ROM) with gravity eliminated. Which grade does the nurse document in this client's record? a. 0 b. 1 c. 2 d. 3

c. 2 This client should be given a grade of 2. Two indicates poor muscle strength; the client can complete ROM with gravity eliminated. Zero indicates no evidence of muscle contractility. One indicates trace muscle strength; the client has no joint motion and slight evidence of muscle contractility. Three indicates fair muscle strength; the client can complete ROM against gravity.

Which client does the medical unit charge nurse assign to an LPN/LVN? a. A 23-year-old scheduled for a bone marrow biopsy with conscious sedation b. A 35-year-old with a history of a splenectomy and a temperature of 100.9° F (38.3° C) c. A 48-year-old with chronic microcytic anemia associated with alcohol use d. A 62-year-old with atrial fibrillation and an international normalized ratio of 6.6

c. A 48-year-old with chronic microcytic anemia associated with alcohol use Chronic microcytic anemia is not considered life-threatening and can be assigned to an LPN/LVN. The clients with a bone marrow biopsy with conscious sedation, a history of splenectomy and a temperature, and atrial fibrillation require more complex assessment or nursing care and should be assigned to RN staff members.

Which of these clients with diabetes does the endocrine unit charge nurse assign to an RN who has floated from the labor/delivery unit? a. A 58-year-old with sensory neuropathy who needs teaching about foot care b. A 68-year-old with diabetic ketoacidosis who has an IV running at 250 mL/hr c. A 70-year-old who needs blood glucose monitoring and insulin before each meal d. A 76-year-old who was admitted with fatigue and shortness of breath

c. A 70-year-old who needs blood glucose monitoring and insulin before each meal A nurse from the labor/delivery unit would be familiar with blood glucose monitoring and insulin administration because clients with type 1 and gestational diabetes are frequently cared for in the labor/delivery unit. The 58-year-old with sensory neuropathy, the 68-year-old with diabetic ketoacidosis, and the 76-year-old with fatigue and shortness of breath all have specific teaching or assessment needs that are better handled by nurses more familiar with caring for older adults with diabetes.

The nurse receives report on a client with hydronephrosis. Which laboratory study does the nurse monitor? a. Hemoglobin and hematocrit (H&H) b. White blood cell (WBC) count c. Blood urea nitrogen (BUN) and creatinine d. Lipid levels

c. Blood urea nitrogen (BUN) and creatinine BUN and creatinine are kidney function tests. With back-pressure on the kidney, glomerular filtration is reduced or absent, resulting in permanent kidney damage. Hydronephrosis results from the backup of urine secondary to obstruction; H&H monitors for anemia and blood loss, while WBC count indicates infection. Elevated lipid levels are associated with nephrotic syndrome, not with obstruction and hydronephrosis.

A client is hospitalized with a possible disorder of the adrenal cortex. Which nursing activity is best for the charge nurse to delegate to an experienced nursing assistant? a. Ask about risk factors for adrenocortical problems. b. Assess the client's response to physiologic stressors. c. Check the client's blood glucose levels every 4 hours. d. Teach the client how to do a 24-hour urine collection.

c. Check the client's blood glucose levels every 4 hours. Blood glucose monitoring is within the nursing assistant's scope of practice if the nursing assistant has received education and evaluation in the skill. Assessing risk factors for adrenocortical problems is not part of a nursing assistant's education. Assessing the client's response to physiologic stressors requires the more complex skill set of licensed nursing staff. Teaching the proper method for a 24-hour urine collection is a multi-step process; this task should not be delegated.

A 56-year-old client admitted with a diagnosis of acute myelogenous leukemia (AML) is prescribed IV cytosine arabinoside for 7 days and an infusion of daunorubicin for the first 3 days. An infection develops. What knowledge does the nurse use to determine that the appropriate antibiotic has been prescribed for this client? a. Evaluating the client's liver function tests (LFTs) and serum creatinine levels b. Evaluating the client's white blood cell (WBC) count level c. Checking the culture and sensitivity test results to be certain that the requested antibiotic is effective against the organism causing the infection d. Recognizing that vancomycin (Vancocin) is the drug of choice used to treat all infections in clients with AML

c. Checking the culture and sensitivity test results to be certain that the requested antibiotic is effective against the organism causing the infection Checking the culture and sensitivity test results to be certain that the requested antibiotic is effective against the organism causing the infection is the best action to take. Drug therapy is the main defense against infections that develop in clients undergoing therapy for AML. Agents used depend on the client's sensitivity to various antibiotics for the organism causing the infection. Although LFTs and kidney function tests may be influenced by antibiotics, these tests do not determine the effectiveness of the antibiotic. Although the WBC count is elevated in infection, this test does not influence which antibiotic will be effective in fighting the infection. Vancomycin may not be effective in all infections; culturing of the infection site and determining the organism's sensitivity to a cohort of drugs are needed, which will provide data on drugs that are capable of eradicating the infection in this client.

These data are obtained by the RN who is assessing a client who had a transsphenoidal hypophysectomy yesterday. What information has the most immediate implications for the client's care? a. Dry lips and oral mucosa on examination b. Nasal drainage that tests negative for glucose c. Client report of a headache and stiff neck d. Urine specific gravity of 1.016

c. Client report of a headache and stiff neck Headache and stiff neck (nuchal rigidity) are symptoms of meningitis that have immediate implications for the client's care. Dry lips and mouth are not unusual after surgery. Frequent oral rinses and the use of dental floss should be encouraged because the client cannot brush the teeth. Any nasal drainage should test negative for glucose; nasal drainage that tests positive for glucose indicates the presence of a cerebrospinal fluid leak. A urine specific gravity of 1.016 is within normal limits.

After receiving change-of-shift report on the urology unit, which client does the nurse assess first? a. Client post radical nephrectomy whose temperature is 99.8° F (37.6° C) b. Client with glomerulonephritis who has cola-colored urine c. Client who was involved in a motor vehicle crash and has hematuria d. Client with nephrotic syndrome who has gained 2 kg since yesterday

c. Client who was involved in a motor vehicle crash and has hematuria The nurse should be aware of the risk for kidney trauma after a motor vehicle crash; this client needs further assessment and evaluation to determine the extent of blood loss and the reason for the hematuria because hemorrhage can be life-threatening. Although slightly elevated, the low-grade fever of the client who is post radical nephrectomy is not life-threatening in the same way as a trauma victim with bleeding. Cola-colored urine is an expected finding in glomerulonephritis. Because of loss of albumin, fluid shifts and weight gain can be anticipated in a client with nephrotic syndrome.

An RN and LPN/LVN are caring for a group of clients on the medical-surgical unit. Which client will be the best to assign to the LPN/LVN? a. Client with Graves' disease who needs discharge teaching after a total thyroidectomy b. Client with hyperparathyroidism who is just being admitted for a parathyroidectomy c. Client with infiltrative ophthalmopathy who needs administration of high-dose prednisone (Deltasone) d. Newly diagnosed client with hypothyroidism who needs education about the use of thyroid supplements

c. Client with infiltrative ophthalmopathy who needs administration of high-dose prednisone (Deltasone) Medication administration for the client with infiltrative ophthalmopathy is within the scope of practice of the LPN/LVN. Discharge teaching is a complex task that cannot be delegated to the LPN/LVN. A client being admitted for a parathyroidectomy needs preoperative teaching, which must be provided by the RN. A client who has a new diagnosis will have questions about the disease and prescribed medications; teaching is a complex task that is appropriate for the RN.

Which client does the nurse identify as being at highest risk for acute adrenal insufficiency resulting from corticosteroid use? a. Client with hematemesis, upper epigastric pain for the past 3 days not relieved with food, and melon b. Client with right upper quadrant pain unrelieved for the past 2 days, dark-brown urine, and clay-colored stools c. Client with shortness of breath and chest tightness, nasal flaring, audible wheezing, and oxygen saturation of 85% for the second time this week d. Client with three emergency department visits in the past month for edema, shortness of breath, weight gain, and jugular venous distention

c. Client with shortness of breath and chest tightness, nasal flaring, audible wheezing, and oxygen saturation of 85% for the second time this week Corticosteroids may be used to treat signs and symptoms of asthma, such as shortness of breath and chest tightness, nasal flaring, audible wheezing, and oxygen saturation of 85%. This places the client at risk for adrenal insufficiency. Corticosteroids are not used to treat signs and symptoms of GI bleeding or peptic ulcer disease (hematemesis, upper epigastric pain for the past 3 days not relieved with food, and melena), gallbladder disease (right upper quadrant pain unrelieved for the past 2 days, dark brown urine, and clay-colored stools), or congestive heart failure (edema, shortness of breath, weight gain, and jugular venous distention).

A client has suspected alterations in antidiuretic hormone (ADH) function. Which diagnostic test does the nurse anticipate will be requested for this client? a. Adrenocorticotropic hormone (ACTH) suppression test b. Chest x-ray c. Cranial computed tomography (CT) d. Renal sonography

c. Cranial computed tomography (CT) ADH is a hormone of the posterior pituitary. Brain abscess, tumor, or subarachnoid hemorrhage could cause alterations in ADH levels. These can be seen on a CT scan of the brain. ACTH triggers the release of cortisol from the adrenal cortex and is not related to ADH. A chest x-ray would not show a pituitary tumor or brain abscess. Even though ADH acts on distal convoluted tubules in the kidneys, a renal sonogram would diagnose the cause of syndrome of inappropriate antidiuretic hormone.

Which action does the nurse delegate to unlicensed assistive personnel (UAP) who are assisting with the care of a female client with anemia? a. Asking the client about the amount of blood loss with each menstrual period b. Checking for sternal tenderness while applying fingertip pressure c. Determining the respiratory rate before and after the client walks 20 feet d. Monitoring her oral mucosa for pallor, bleeding, or ulceration

c. Determining the respiratory rate before and after the client walks 20 feet Assessment of the respiratory rate before and after ambulation is within the scope of practice for UAP; UAP will report this information to the RN. Asking the client about the amount of blood loss with each menstrual period, checking for sternal tenderness, and monitoring oral mucosa require skilled assessment techniques and knowledge of normal parameters and should be done by the RN.

A client with end-stage kidney disease has been put on fluid restrictions. Which assessment finding indicates that the client has not adhered to this restriction? a. Blood pressure of 118/78 mm Hg b. Weight loss of 3 pounds during hospitalization c. Dyspnea and anxiety at rest d. Central venous pressure (CVP) of 6 mm Hg

c. Dyspnea and anxiety at rest Dyspnea is a sign of fluid overload and possible pulmonary edema; the nurse should assist the client in correlating symptoms of fluid overload with nonadherence to fluid restriction. Nonadherence to fluid restriction results in fluid volume excess and higher blood pressures; 118/78 mm Hg is a normal blood pressure. Excess fluid intake and fluid retention are manifested by an elevated CVP (>8 mm Hg) and weight gain, not weight loss.

The nurse admits a client diagnosed with Paget's disease. The nurse anticipates that the client will have which condition? a. Progressive muscle weakness b. Low body weight, thin build c. Enlarged, thick skull d. Bone infection

c. Enlarged, thick skull An enlarged thick skull is a feature of Paget's disease. Progressive muscle weakness is a feature of muscular dystrophy. Low body weight with a thin build is a feature of osteoporosis. Bone infection is a feature of osteomyelitis.

Which negative feedback response is responsible for preventing hypoglycemia during sleep in nondiabetic clients? a. Alpha cells of the pancreas b. Beta cells of the pancreas c. Glucagon release d. Insulin release

c. Glucagon release Glucagon is the hormone that binds to receptors on liver cells. This causes the liver cells to convert glycogen to glucose, which keeps blood sugar levels normal during sleep. Alpha cells are responsible for synthesizing and secreting the hormone glucagon. Beta cells are responsible for synthesizing and secreting the hormone insulin. Insulin is the hormone responsible for lowering blood glucose. Insulin improves glucose uptake by the cell.

A cognitively impaired client has urge incontinence. Which method for achieving continence does the nurse include in the client's care plan? a. Bladder training b. Credé method c. Habit training d. Kegel exercises

c. Habit training Habit training (scheduled toileting) will be most effective in reducing incontinence for a cognitively impaired client because the caregiver is responsible for helping the client to a toilet on a scheduled basis. Bladder training, the Credé method, and learning Kegel exercises require that the client be alert, cooperative, and able to assist with his or her own training.

The nurse is reviewing the medical record for a client with polycystic kidney disease who is scheduled for computed tomographic angiography with contrast: *Polycystic kidney disease *Diabetes *Hysterectomy *Abdomen distended *Negative edema *Glyburide *Metformin *Synthroid *BUN 26 mg/dL *Creatinine 1.0 mg/dL *HbA1c 6.9% *Glucose 132 mg/dL Which intervention is essential for the nurse to perform? a. Obtain a thyroid-stimulating hormone (TSH) level. b.Report the blood urea nitrogen (BUN) and creatinine. c. Hold the metformin 24 hours before and on the day of the procedure. d. Notify the provider regarding blood glucose and glycosylated hemoglobin (HbA1c) values.

c. Hold the metformin 24 hours before and on the day of the procedure. Before studies with contrast media are performed, the nurse must withhold metformin, which may cause lactic acidosis. The focus of this admission is the polycystic kidneys; a TSH level is not essential at this time. BUN and creatinine are normal. The glucose is only mildly elevated (if fasting), and the HbA1c is in an appropriate range.

The nurse is assessing a client with osteomalacia. Which findings does the nurse expect to observe? (Select all that apply.) a. Hyperparathyroidism b. Hyperuricemia c. Hypophosphatemia d. Looser's lines or zones e. Unsteady gait

c. Hypophosphatemia d. Looser's lines or zones e. Unsteady gait Osteomalacia is loss of bone related to vitamin D deficiency, which can lead to bone softening and inadequate deposits of calcium and phosphorus in the bone matrix; this may cause hypophosphatemia. Looser's lines or zones (radiolucent bands) represent stress fractures and are a classic diagnostic finding of osteomalacia. Muscle weakness in the lower extremities may cause waddling and an unsteady gait. Hyperparathyroidism and hyperuricemia may be observed in Paget's disease.

A client has sustained a rotator cuff tear while playing baseball. The nurse anticipates that the client will receive which immediate conservative treatment? a. Surgical repair of the rotator cuff b. Prescribed exercises of the affected arm c. Immobilizer for the affected arm d. Patient-controlled analgesia with morphine

c. Immobilizer for the affected arm The conservative treatment for this client is to place the injured arm in an immobilizer. Surgical intervention is not considered conservative treatment. Exercises are prohibited immediately after a rotator cuff injury. The client with a rotator cuff injury is treated primarily with nonsteroidal anti-inflammatory drugs to manage pain.

Which laboratory result indicates that fluid restrictions have been effective in treating syndrome of inappropriate antidiuretic hormone (SIADH)? a. Decreased hematocrit b. Decreased serum osmolality c. Increased serum sodium d. Increased urine specific gravity

c. Increased serum sodium Increased serum sodium due to fluid restriction indicates effective therapy. Hemoconcentration is a result of hypovolemic hyponatremia caused by SIADH and diabetes insipidus. Plasma osmolality is decreased as a result of SIADH. Urine specific gravity is decreased with diabetes insipidus and is increased with SIADH.

A client with hypothyroidism is being discharged. Which environmental change may the client experience in the home? a. Frequent home care b. Handrails in the bath c. Increased thermostat setting d. Strict infection-control measures

c. Increased thermostat setting Manifestations of hypothyroidism include cold intolerance. Increased thermostat settings or additional clothing may be necessary. A client with a diagnosis of hypothyroidism can be safely managed at home with adequate discharge teaching regarding medications and instructions on when to notify the health care provider or home health nurse. In general, hypothyroidism does not cause mobility issues. Activity intolerance and fatigue may be an issue, however. A client with hypothyroidism is not immune-compromised or contagious, so no environmental changes need to be made to the home.

When a client with diabetes returns to the medical unit after a computed tomography (CT) scan with contrast dye, all of these interventions are prescribed. Which intervention does the nurse implement first? a. Give lispro (Humalog) insulin, 12 units subcutaneously. b. Request a breakfast tray for the client. c. Infuse 0.45% normal saline at 125 mL/hr. d. Administer captopril (Capoten).

c. Infuse 0.45% normal saline at 125 mL/hr. Fluids are needed because the dye used in a CT scan with contrast has an osmotic effect, causing dehydration and potential kidney failure. Lispro is not administered until the breakfast tray arrives. A breakfast tray will be requested, but preventing complications of the procedure is done first. Because the client may be hypovolemic, the nurse should monitor blood pressure and administer IV fluids before deciding whether administration of captopril is appropriate.

The nurse is infusing platelets to a client who is scheduled for a hematopoietic stem cell transplant. What procedure does the nurse follow? a. Administer intravenous corticosteroids before starting the transfusion. b. Allow the platelets to stabilize at the client's bedside for 30 minutes. c. Infuse the transfusion over a 15- to 30-minute period. d. Set up the infusion with the standard transfusion Y tubing.

c. Infuse the transfusion over a 15- to 30-minute period. The volume of platelets—200 or 300 mL (standard amount)—needs to be infused rapidly over a 15- to 30-minute period. Administering steroids is not standard practice in administering platelets. Platelets must be administered immediately after they are received; they are considered to be quite fragile. A special transfusion set with a smaller filter and shorter tubing is used to get the platelets into the client quickly and efficiently.

A client diagnosed with hyperpituitarism resulting from a prolactin-secreting tumor has been prescribed bromocriptine mesylate (Parlodel). As a dopamine agonist, what effect does this drug have by stimulating dopamine receptors in the brain? a. Decreases the risk for cerebrovascular disease b. Increases the risk for depression c. Inhibits the release of some pituitary hormones d. Stimulates the release of some pituitary hormones

c. Inhibits the release of some pituitary hormones Bromocriptine mesylate inhibits the release of both prolactin and growth hormone. It does not decrease the risk for cerebrovascular disease leading to stroke. Increased risk for depression is not associated with the use of bromocriptine mesylate; however, hallucinations have been reported as a side effect. Bromocriptine mesylate does not stimulate the release of any hormones.

A client is in skeletal traction. Which nursing intervention ensures proper care of this client? a. Ensure that weights are attached to the bed frame or placed on the floor. b. Ensure that pins are not loose, and tighten as needed. c. Inspect the skin at least every 8 hours. d. Remove the traction weights only for bathing.

c. Inspect the skin at least every 8 hours. The client's skin should be inspected every 8 hours for signs of irritation, inflammation, or actual skin breakdown. Weights are not allowed to be placed on the floor; weights should hang freely at all times. Pin sites should be checked for signs and symptoms of infection and for security in their position to the fixation and the client's extremity. However, the nurse does not adjust the pins. Any loose pin site or alteration must be reported to the health care provider. Weights must never be removed without a request from the health care provider.

The nurse is assessing a client with Ewing's sarcoma. Which finding does the nurse expect to observe? a. Bradycardia b. High fever c. Leukocytosis d. Migraine headaches

c. Leukocytosis Ewing's sarcoma is a malignant tumor, and the client may experience systemic manifestations, including leukocytosis, anemia, and low-grade fever. Bradycardia and migraine headache are not symptoms of Ewing's sarcoma. A low-grade fever is a systemic manifestation of Ewing's sarcoma.

An older client with an elevated serum calcium level is receiving IV furosemide (Lasix) and an infusion of normal saline at 150 mL/hr. Which nursing action can the RN delegate to unlicensed assistive personnel (UAP)? a. Ask the client about any numbness or tingling. b. Check for bone deformities in the client's back. c. Measure the client's intake and output hourly. d. Monitor the client for shortness of breath.

c. Measure the client's intake and output hourly. Measuring intake and output is a commonly delegated nursing action that is within the UAP scope of practice. Numbness and tingling is part of the client assessment that needs to be completed by a licensed nurse. Bony deformities can be due to pathologic fractures; physical assessment is a complex task that cannot be delegated. An older client receiving an IV at 150 mL/hr is at risk for congestive heart failure; careful monitoring for shortness of breath is the responsibility of the RN.

Which type of thyroid cancer often occurs as part of multiple endocrine neoplasia (MEN) type II? a. Anapestic b. Follicular c. Medullary d. Papillary

c. Medullary Medullary carcinoma commonly occurs as part of MEN type II, which is a familial endocrine disorder. Anaplastic carcinoma is an aggressive tumor that invades surrounding tissue. Follicular carcinoma occurs more frequently in older clients and may metastasize to bone and lung. Papillary carcinoma is the most common type of thyroid cancer. It is slow growing and, if the tumor is confined to the thyroid gland, the outlook for a cure is good with surgical management.

Which information suggests that a client with diabetes may be in the early stages of kidney damage? a. Elevation in blood urea nitrogen (BUN) b. Oliguria c. Microalbuminuria d. Painless hematuria

c. Microalbuminuria In the early stages of diabetic nephropathy, micro-levels of albumin are first detected in the urine. Progressive kidney damage occurs before dipstick procedures can detect protein in the urine. BUN may change in response to protein and fluid intake. Oliguria is a later finding in kidney disease and may also be present in dehydration. Painless hematuria often occurs with kidney cancer.

Which action does the postanesthesia care unit (PACU) nurse perform first when caring for a client who has just arrived after a total thyroidectomy? a. Assess the wound dressing for bleeding. b. Give morphine sulfate 4 to 8 mg IV for pain. c. Monitor oxygen saturation using pulse oximetry. d. Support the head and neck with sandbags.

c. Monitor oxygen saturation using pulse oximetry. Airway assessment and management is always the first priority with every client. This is especially important for a client who has had surgery that involves potential bleeding and edema near the trachea. Assessing the wound dressing for bleeding is a high priority, although this is not the first priority. Pain control and supporting the head and neck with sandbags are important priorities, but can be addressed after airway assessment.

The nurse visualizes blood clots in a client's urinary catheter after a cystoscopy. What nursing intervention does the nurse perform first? a. Administer heparin intravenously. b. Remove the urinary catheter. c. Notify the health care provider d. Irrigate the catheter with sterile saline.

c. Notify the health care provider. Bleeding and/or blood clots are potential complications of cystoscopy and may obstruct the catheter and decrease urine output. The nurse should monitor urine output and notify the health care provider of obvious blood clots or a decreased or absent urine output. Heparin will not be administered due to bleeding. The urinary catheter is allowing close monitoring of the urinary system and should not be removed at this time. The Foley catheter may be irrigated with sterile saline, as ordered.

The nurse is caring for a client with neutropenia who has a suspected infection. Which intervention does the nurse implement first? a. Hydrate the client with 1000 mL of IV normal saline. b. Initiate the administration of prescribed antibiotics. c. Obtain requested cultures. d. Place the client on Bleeding Precautions.

c. Obtain requested cultures. Obtaining cultures to identify the infectious agent correctly is the priority for this client. Hydrating the client is not the priority. Administering antibiotics is important, but antibiotics should always be started after cultures are obtained. Placing the client on Bleeding Precautions is unnecessary.

The nurse is caring for a client with prostate cancer who has bone metastasis. The nurse anticipates that the health care provider will prescribe which medication? a. Calcitonin (Calcimar) b. Medroxyprogesterone (Prempro) c. Pamidronate (Aredia) d. Tamsulosin hydrochloride (Flomax)

c. Pamidronate (Aredia) Pamidronate is a bisphosphonate that is available intravenously and is approved for bone metastasis from the breast, lung, and prostate. Pamidronate protects bones and prevents fractures. Calcitonin is used for the treatment of postmenopausal osteoporosis, Paget's disease, and hypercalcemia associated with cancer. Medroxyprogesterone is indicated for treating menopausal symptoms and preventing osteoporosis. Tamsulosin hydrochloride is an alpha-adrenergic blocking agent used for the treatment of benign prostatic hyperplasia.

A 32-year-old client recovering from a sickle cell crisis is to be discharged. The nurse says, "You and all clients with sickle cell disease are at risk for infection because of your decreased spleen function. For this reason, you will most likely be prescribed an antibiotic before discharge." Which drug does the nurse anticipate the health care provider will request? a. Cefaclor (Ceclor) b. Gentamicin (Garamycin) c. Penicillin V (Pen-V K) d. Vancomycin (Vancocin)

c. Penicillin V (Pen-V K) Prophylactic therapy with twice-daily oral penicillin reduces the incidence of pneumonia and other streptococcal infections and is the correct drug to use. It is a standard protocol for long-term prophylactic use in clients with sickle cell disease. Cefaclor (Ceclor) and vancomycin (Vancocin) are antibiotics more specific for short-term use and would be inappropriate for this client. Gentamicin (Garamycin) is a drug that can cause liver and kidney damage with long-term use.

Which nursing activity illustrates proper aseptic technique during catheter care? a. Applying Betadine ointment to the perineal area after catheterization b. Irrigating the catheter daily c. Positioning the collection bag below the height of the bladder d. Sending a urine specimen to the laboratory for testing

c. Positioning the collection bag below the height of the bladder Urine collection bags must be kept below the level of the bladder at all times. Elevating the collection bag above the bladder causes reflux of pathogens from the bag into the urinary tract. Applying antiseptic solutions or antibiotic ointments to the perineal area of catheterized clients has not demonstrated any beneficial effect. A closed system of irrigation must be maintained by ensuring that catheter tubing connections are sealed securely; disconnections can introduce pathogens into the urinary tract, so routine catheter irrigation should be avoided. Sending a urine specimen to the laboratory is not indicated for asepsis.

An intensive care client with diabetic ketoacidosis (DKA) is receiving an insulin infusion. The cardiac monitor shows ventricular ectopy. Which assessment does the nurse make? a. Urine output b. 12-lead electrocardiogram (ECG) c. Potassium level d. Rate of IV fluids

c. Potassium level With insulin therapy, serum potassium levels fall rapidly as potassium shifts into the cells. Detecting and treating the underlying cause is essential. Insulin treats symptoms of diabetes by putting glucose into the cell as well as potassium; ectopy, indicative of cardiac irritability, is not associated with changes in urine output. A 12-lead ECG can verify the ectopy, but the priority is to detect and fix the underlying cause. Increased fluids treat the symptoms of dehydration secondary to DKA, but do not treat the cause.

Which client information is most essential for the nurse to report to the health care provider before a client with knee pain undergoes magnetic resonance imaging (MRI)? a. Daily use of aspirin b. Swollen and tender knee c. Presence of a permanent pacemaker d. History of claustrophobia

c. Presence of a permanent pacemaker Having a permanent pacemaker is a contraindication for MRI because metallic implants are present within the client. Taking a daily dose of aspirin does not affect or interact with the MRI test. A swollen and tender knee does not warrant cancellation of an MRI. A history of claustrophobia should be reported, but does not indicate that cancellation of the MRI is necessary because sedatives can be given to manage claustrophobia.

Which action is correct when drawing up a single dose of insulin? a. Wash hands thoroughly and don sterile gloves. b. Shake the bottle of insulin vigorously to mix the insulin. c. Pull back plunger to draw air into the syringe equal to the insulin dose. d. Recap the needle and save the syringe for the next dose of insulin.

c. Pull back plunger to draw air into the syringe equal to the insulin dose. The plunger is pulled back to draw an amount of air into the syringe that is equal to the insulin dose. The air is then injected into the insulin bottle before withdrawing the insulin dose. Although handwashing is important before any medication administration, sterile gloves are not required. The bottle of insulin should be rolled gently in the palms of the hands to mix the insulin, not shaken. Insulin syringes are never recapped or reused; the syringe and needle should be disposed of (without recapping) in a puncture-proof container.

A newly admitted client who is diabetic and has pyelonephritis and prescriptions for intravenous antibiotics, blood glucose monitoring every 2 hours, and insulin administration should be cared for by which staff member? a. RN whose other assignments include a client receiving chemotherapy for renal cell carcinoma b. RN who is caring for a client who just returned after having renal artery balloon angioplasty c. RN who has just completed preoperative teaching for a client who is scheduled for nephrectomy. d. RN who is currently admitting a client with acute hypertension and possible renal artery stenosis

c. RN who has just completed preoperative teaching for a client who is scheduled for nephrectomy. The client scheduled for nephrectomy is the most stable client; the RN caring for this client will have time to perform the frequent monitoring and interventions that are needed for the newly admitted client. The client receiving chemotherapy will require frequent monitoring by the RN. The client after angioplasty will require frequent vital sign assessment and observation for hemorrhage and arterial occlusion. The client with acute hypertension will need frequent monitoring and medication administration.

A client admitted to the medical unit with a history of vomiting and diarrhea and an increased blood urea nitrogen requires 1 liter of normal saline infused over 2 hours. Which staff member should be assigned to care for the client? a. RN who has floated from pediatrics for this shift b. LPN/LVN with experience working on the medical unit c. RN who usually works on the general surgical unit d. New graduate RN who just finished a 6-week orientation

c. RN who usually works on the general surgical unit The nurse with experience in taking care of surgical clients will be most capable of monitoring the client receiving rapid fluid infusions, who is at risk for complications such as pulmonary edema and acute kidney failure. The pediatric float RN and the new graduate RN will have less experience in caring for this type of client. The LPN/LVN should not be assigned to a client requiring IV therapy and who is at high risk for complications.

The nurse is to administer packed red blood cells to a client. How does the nurse ensure proper client identification? a. Asks the client's name b. Checks the client's armband c. Reviews all information with another registered nurse d. Verifies the client's room number

c. Reviews all information with another registered nurse With another registered nurse, verify the client by name and number, check blood compatibility, and note expiration time. Human error is the most common cause of ABO incompatibility reactions, even for experienced nurses. Asking the client's name and checking the client's armband are not adequate for identifying the client before transfusion therapy. Using the room number to verify client identification is never appropriate.

The nurse suspects that a client may have plantar fasciitis if the client has which assessment finding? a. Lateral deviation of the great toe; first metatarsal head becomes enlarged b. Dorsiflexion of any metatarsophalangeal (MTP) joint, with plantar flexion of the adjacent proximal interphalangeal (PIP) joint c. Severe pain in the arch of the foot, especially when getting out of bed d. A small tumor in a digital nerve of the foot

c. Severe pain in the arch of the foot, especially when getting out of bed Severe pain in the arch of the foot, especially when getting out of bed, is a description of plantar fasciitis. Lateral deviation of the great toe with an enlarged first metatarsal head describes a bunion of the foot. Dorsiflexion of any MTP joint with plantar flexion of the adjacent PIP joint is a description of a hallux valgus and hammertoe of the foot. A small tumor in a digital nerve of the foot describes Morton's neuroma of the foot.

The certified Wound, Ostomy, and Continence Nurse or enterostomal therapist teaches a client who has had a cystectomy about which care principles for the client's post-discharge activities? a. Nutritional and dietary care b. Respiratory care c. Stoma and pouch care d. Wiping from front to back (asepsis)

c. Stoma and pouch care The enterostomal therapist demonstrates external pouch application, local skin care, pouch care, methods of adhesion, and drainage mechanisms. The registered dietitian teaches the cystectomy client about nutritional care. The respiratory therapist teaches the cystectomy client about respiratory care. The client with a cystectomy does not require instruction about front-to-back wiping.

Which type of incontinence benefits from pelvic floor muscle (Kegel) exercise? a. Functional b. Overflow c. Stress d. Urge

c. Stress Pelvic floor (Kegel) exercise therapy for women with stress incontinence strengthens the muscles of the pelvic floor, thereby helping decrease the occurrence of incontinence. Functional incontinence is not caused by a weakened pelvic floor; rather, it is due to structural problems often resulting from injury or trauma. Overflow incontinence is caused by too much urine being stored in the bladder. Urge incontinence is caused by a problem (i.e., neurologic) with the client's urge to urinate.

Which assessment finding alarms the nurse immediately after a client returns from the operating room for cystoscopy performed under conscious sedation? a. Pink-tinged urine b. Urinary frequency c.Temperature of 100.8° F d. Lethargy

c. Temperature of 100.8° F Fever, chills, or an elevated white blood cell count after cystoscopy suggest infection after an invasive procedure; the provider must be notified immediately. Pink-tinged urine is expected after a cystoscopy; gross hematuria would require notification of the surgeon. Frequency may be noted as a result of irritation of the bladder. If sedation or anesthesia was used, lethargy is an expected effect. Awarded 0.0 points out of 1.0 possible points.

Which explanation best assists a client in differentiating type 1 diabetes from type 2 diabetes? a. Most clients with type 1 diabetes are born with it. b. People with type 1 diabetes are often obese. c. Those with type 2 diabetes make insulin, but in inadequate amounts. d. People with type 2 diabetes do not develop typical diabetic complications.

c. Those with type 2 diabetes make insulin, but in inadequate amounts. People with type 2 diabetes make some insulin but in inadequate amounts, or they have resistance to existing insulin. Although type 1 diabetes may occur early in life, it may be caused by immune responses. Obesity is typically associated with type 2 diabetes. People with type 2 diabetes are at risk for complications, especially cardiovascular complications.

A 32-year-old female with a urinary tract infection (UTI) reports urinary frequency, urgency, and some discomfort upon urination. Her vital signs are stable except for a temperature of 100° F. Which drug does the health care provider prescribe? a. Nitrofurantoin (Macrodantin) after intercourse b. Estrogen (Premarin) c. Trimethoprim/sulfamethoxazole (Bactrim) d. Phenazopyridine (Pyridium) with intercourse

c. Trimethoprim/sulfamethoxazole (Bactrim) Guidelines indicate that a 3-day course of trimethoprim/sulfamethoxazole is effective in treating uncomplicated, community-acquired UTI in women. Drugs from the same class as nitrofurantoin reduce bacteria in the urinary tract by inhibiting bacterial reproduction (bacteriostatic action). This client needs a drug that will kill bacteria. Estrogen cream may help prevent recurrent UTIs in postmenopausal women, which this client is not (at age 32). Use of Premarin is related to problems with incontinence. Phenazopyridine (Pyridium) is not used to treat infection, but symptoms of a UTI.

When caring for a client who had a nephrostomy tube inserted 4 hours ago, which is essential for the nurse to report to the health care provider? a. Dark pink-colored urine b. Small amount of urine leaking around the catheter c. Tube that has stopped draining d. Creatinine of 1.8 mg/dL

c. Tube that has stopped draining The provider must be notified when a nephrostomy tube does not drain; it could be obstructed or dislodged. Pink or red drainage is expected for 12 to 24 hours after insertion and should gradually clear. The nurse may reinforce the dressing around the catheter to address leaking urine; however, the provider should be notified if there is a large quantity of leaking drainage, which may indicate tube obstruction. A creatinine level of 1.8 mg/dL is expected in a client early after nephrostomy tube placement (due to the minor kidney damage that required the nephrostomy tube).

While assisting a client during peritoneal dialysis, the nurse observes the drainage stop after 200 mL of peritoneal effluent drains into the bag. What action should the nurse implement first? a. Instruct the client to deep-breathe and cough. b. Document the effluent as output. c. Turn the client to the opposite side. d. Re-position the catheter.

c. Turn the client to the opposite side. With peritoneal dialysis, usually 1 to 2 L of dialysate is infused by gravity into the peritoneal space. The fluid dwells in the peritoneal cavity for a specified time, then drains by gravity into a drainage bag. The dialyzing fluid is called peritoneal effluent on outflow. The outflow should be a continuous stream after the clamp is completely open. Potential causes of flow difficulty include constipation, kinked or clamped connection tubing, the client's position, fibrin clot formation, and catheter displacement. If inflow or outflow drainage is inadequate, re-position the client to stimulate inflow or outflow. Turning the client to the other side or ensuring that he or she is in good body alignment may help. Instructing the client to deep-breathe and cough will not promote dialysate drainage. Increased abdominal pressure from coughing contributes to leakage at the catheter site. The nurse needs to measure and record the total amount of outflow after each exchange. However, the nurse should re-position the client first to assist with complete dialysate drainage. An x-ray is needed to identify peritoneal dialysis catheter placement. Only the physician re-positions a displaced catheter.

Which finding does the nurse expect to observe in a client with suspected common chronic osteomyelitis? a. Erythema of the affected area b. Fever; temperature usually above 101° F (38° C) c. Ulceration of the skin d. Constant, localized, and pulsating bone pain

c. Ulceration of the skin Ulceration of the skin is a feature of chronic osteomyelitis. Erythema of the affected area; fever; and constant, localized, pulsating bone pain are features of acute osteomyelitis.

A client has hyperparathyroidism. Which incident witnessed by the nurse requires the nurse's intervention? a. The client eating a morning meal of cereal and fruit b. The physical therapist walking with the client in the hallway c. Unlicensed assistive personnel pulling the client up in bed by the shoulders d. Visitors talking with the client about going home

c. Unlicensed assistive personnel pulling the client up in bed by the shoulders The client with hyperparathyroidism is at risk for pathologic fracture. All members of the health care team must move the client carefully. A lift sheet should be used to re-position the client. The client with hyperparathyroidism is not restricted from eating and should maintain a balanced diet. The client can benefit from moderate exercise and physical therapy, and is not restricted from having visitors.

Which factor represents a sign or symptom of digoxin toxicity? a. Serum digoxin level of 1.2 ng/mL b. Polyphagia c. Visual changes d. Serum potassium of 5.0 mEq/L

c. Visual changes Visual changes, anorexia, nausea, and vomiting are symptoms of digoxin toxicity. A digoxin level of 1.2 ng/mL is normal (0.5 to 2.0 ng/mL). Polyphagia is a symptom of diabetes. Although hypokalemia may predispose to digoxin toxicity, this represents a normal, not low, potassium value.

When performing bladder scanning to detect residual urine in a female client, the nurse must first assess which factor? a.Abdominal girth b.Presence of urinary infection c. History of hysterectomy d. Hematuria

c. hx of hysterectomy The scanner must be in the scan mode for female clients to ensure the scanner subtracts the volume of the uterus from the measurement, or in the scan mode for male clients for women who have undergone a hysterectomy. The nurse performs this procedure in response to distention or pressure in the bladder; girth is not a factor. This procedure detects urine retained in the bladder, not infection. The presence of retained urine in the bladder is assessed, regardless of hematuria.

A client is in the emergency department for an inability to void and for bladder distention. What is most important for the nurse to provide to the client? a.Increased oral fluids b. IV fluids c. Privacy d. Health history forms

c. privacy The nurse should provide privacy, assistance, and voiding stimulants, such as warm water over the perineum, as needed, for the client with urinary problems. Increased oral fluids and IV fluids would exacerbate the client's problem. Obtaining a health history is not the priority for this client's care.

A client is scheduled to undergo closed magnetic resonance imaging (MRI) without contrast medium. Which information does the nurse give to the client before the test? a. "It will be important to lie still in a reclined position for 20 minutes." b. "Do not eat or drink for 8 hours before the test." c. "You can have the MRI if you have an internal pacemaker." d. "All jewelry and clothing with zippers or metal fasteners must be removed."

d. "All jewelry and clothing with zippers or metal fasteners must be removed." The client must remove all metal objects on clothing and all jewelry before undergoing MRI. The client having a closed MRI will lie still in a supine position for 45 to 60 minutes, not 20 minutes, and may require sedation. It is not necessary for the client to be NPO before an MRI. The client cannot undergo MRI when an internal pacemaker or any other metal object is present in the body.

A client with a fracture asks the nurse about the difference between a compound fracture and a simple fracture. Which statement by the nurse is correct? a. "Simple fracture involves a break in the bone, with skin contusions." b. "Compound fracture does not extend through the skin." c. "Simple fracture is accompanied by damage to the blood vessels." d. "Compound fracture involves a break in the bone, with damage to the skin."

d. "Compound fracture involves a break in the bone, with damage to the skin." A compound fracture involves a break in the bone with damage to the skin. A simple fracture does not extend through the skin. A compound fracture is accompanied by damage to blood vessels.

A nursing student is studying the skeletal system. Which statement indicates to the nursing instructor that the student understands a normal physiologic function of the skeletal system? a. "Volkmann's canals connect osteoblasts and osteoclasts." b. "In the deepest layer of the periosteum is the cortex, which consists of dense, compact bone tissue." c. "The matrix of the bone is where deposits of calcium and magnesium are present." d. "Hematopoiesis occurs in the red marrow, which is where blood cells are produced."

d. "Hematopoiesis occurs in the red marrow, which is where blood cells are produced." Hematopoiesis is the production of blood cells in the red marrow and is the correct statement. Volkmann's canals connect bone marrow vessels with the haversian system. In the deepest layer of the periosteum are osteogenic cells that differentiate into osteoblasts and osteoclasts. The cortex is the outer layer of the bone that consists of dense, compact bone tissue. Deposits of inorganic calcium salts (carbonate and phosphate) in the matrix of the bone are what provide the hardness of bone.

A client is admitted for extracorporeal shock wave lithotripsy (ESWL). What information obtained on admission is most critical for a nurse to report to the health care provider before the ESWL procedure begins? a. "Blood in my urine has become less noticeable, so maybe I don't need this procedure." b. "I have been taking cephalexin (Keflex) for an infection." c. "I previously had several ESWL procedures performed." d. "I take over-the-counter naproxen (Aleve) twice a day for joint pain."

d. "I take over-the-counter naproxen (Aleve) twice a day for joint pain." Because a high risk for bleeding during ESWL has been noted, clients should not take nonsteroidal anti-inflammatory drugs before this procedure; the ESWL will have to be rescheduled for this client. Blood in the client's urine should be reported to the health care provider, but will not require rescheduling of the procedure because blood is frequently present in the client's urine when kidney stones are present. A diminished amount of blood would not eliminate the need for the procedure. The client's taking cephalexin (Keflex) and the fact that the client has had several previous ESWL procedures should be reported, but will not require rescheduling of the procedure.

The nurse is teaching a client who is scheduled for a neobladder and a Kock pouch. Which client statement indicates a correct understanding of these procedures? a. "If I restrict my oral intake of fluids, the adjustment will be easier." b. "I must go to the restroom more often because my urine will be excreted through my anus." c. "I need to wear loose-fitting pants so the urine can flow into my ostomy bag." d. "I will have to drain my pouch with a catheter."

d. "I will have to drain my pouch with a catheter." For the client with a neobladder and a Kock pouch, urine is collected in a pouch and is drained with the use of a catheter. Fluids should not be restricted. A neobladder does not require the use of an ostomy bag.

The nurse is teaching the importance of a low purine diet to a client admitted with urolithiasis consisting of uric acid. Which statement by the client indicates that teaching was effective? a. "I am so relieved that I can continue eating my fried fish meals every week." b. "I will quit growing rhubarb in my garden since I'm not supposed to eat it anymore." c. "My wife will be happy to know that I can keep enjoying her liver and onions recipe." d. "I will no longer be able to have red wine with my dinner."

d. "I will no longer be able to have red wine with my dinner." Nutrition therapy depends on the type of stone formed. When stones consist of uric acid (urate), the client should decrease intake of purine sources such as organ meats, poultry, fish, gravies, red wines, and sardines. Reduction of urinary purine content may help prevent these stones from forming. Avoiding oxalate sources such as spinach, black tea, and rhubarb is appropriate when the stones consist of calcium oxalate.

A client with thyroid cancer has just received 131I ablative therapy. Which statement by the client indicates a need for further teaching? a. "I cannot share my toothpaste with anyone." b. "I must flush the toilet three times after I use it." c. "I need to wash my clothes separately from everyone else's clothes." d. "I'm ready to hold my newborn grandson now."

d. "I'm ready to hold my newborn grandson now." Clients undergoing 131I therapy should avoid close contact with pregnant women, infants, and young children for 1 week after treatment. Clients should remain at least 1 meter (39 inches, or roughly 3 feet) away, and limit exposure to less than 1 hour per day. Some radioactivity will remain in the client's salivary glands for up to 1 week after treatment. Care should be taken to avoid exposing others to the saliva. Flushing the toilet three times after use will ensure that all urine has been diluted and removed. Clothing needs to be washed separately and the washing machine then needs to be run empty for a full cycle before it is used to wash the clothing of others.

A client is recovering from an above-the-knee amputation resulting from peripheral vascular disease. Which statement indicates that the client is coping well after the procedure? a. "My spouse will be the only person to change my dressing." b. "I can't believe that this has happened to me. I can't stand to look at it." c. "I do not want any visitors while I'm in the hospital." d. "It will take me some time to get used to this."

d. "It will take me some time to get used to this." Acknowledging that it will take time to get used to the amputation indicates that the client is expressing acceptance and effective coping. Stating that the spouse will change the dressing indicates the client does not want to participate in self-care. Expressing disbelief and disgust over the amputation indicates the client is unwilling to address what has happened. The client who does not want to receive visitors is having difficulty coping with the change in body image.

The nurse is performing discharge teaching for a client after a nephrectomy for renal cell carcinoma. Which statement by the client indicates that teaching has been effective? a. "Because renal cell carcinoma usually affects both kidneys, I'll need to be watched closely." b. "I'll eventually require some type of renal replacement therapy." c. "I'll need to decrease my fluid intake to prevent stress to my remaining kidney." d. "My remaining kidney will provide me with normal kidney function now."

d. "My remaining kidney will provide me with normal kidney function now." After a nephrectomy, the second kidney is expected to eventually provide adequate kidney function, but this may take days or weeks. Renal cell carcinoma typically only affects one kidney. Renal replacement therapy is not the typical treatment for renal cell carcinoma. Fluids should be maintained to flush the remaining kidney.

An older adult woman confides to the nurse, "I am so embarrassed about buying adult diapers for myself." How does the nurse respond? a. "Don't worry about it. You need them." b. "Shop at night, when stores are less crowded." c. "Tell everyone that they are for your husband." d. "That is tough. What do you think might help?"

d. "That is tough. What do you think might help?" Stating that the situation is tough acknowledges the client's concerns, and asking the client to think about what might help assists the client to think of methods to solve her problem. Telling the client not to worry is dismissive of the client's concerns. Telling the client to shop at night does not empower the client, and it reaffirms the client's embarrassment. Suggesting to the client that she tell everyone they are for her husband also does not empower the client; rather, it suggests to the client that telling untruths is acceptable.

A client with thrombocytopenia is being discharged. What information does the nurse incorporate into the teaching plan for this client? a. "Avoid large crowds." b. "Drink at least 2 liters of fluid per day." c. "Elevate your lower extremities when sitting." d. "Use a soft-bristled toothbrush."

d. "Use a soft-bristled toothbrush." Using a soft-bristled toothbrush reduces the risk for bleeding in the client with thrombocytopenia. Avoiding large crowds reduces the risk for infection, but is not specific to the client with thrombocytopenia. Increased fluid intake reduces the risk for dehydration, but is not specific to the client with thrombocytopenia. Elevating extremities reduces the risk for dependent edema, but is not specific to the client with thrombocytopenia.

During discharge teaching for a client with kidney disease, what does the nurse teach the client to do? a. "Drink 2 liters of fluid and urinate at the same time every day." b. "Eat breakfast and go to bed at the same time every day." c. "Check your blood sugar and do a urine dipstick test." d. "Weigh yourself and take your blood pressure."

d. "Weigh yourself and take your blood pressure." Regular weight assessment monitors fluid restriction control, while blood pressure control is necessary to reduce cardiovascular complications and slow the progression of kidney dysfunction. Fluid intake and urination, and breakfast time and bedtime, do not need to be at the same time each day. Clients with diabetes, not kidney disease, should regularly check their blood sugar and perform a urine dipstick test.

A client is being treated for kidney failure. Which statement by the nurse encourages the client to express his or her feelings and concerns about the risk for death and the disruption of lifestyle? a. "All of this is new. What can't you do?" b. "Are you afraid of dying?" c. "How are you doing this morning?" d. "What concerns do you have about your kidney disease?"

d. "What concerns do you have about your kidney disease?" Asking the client about any concerns is open-ended and specific to the client's concerns. Asking the client to explain what he or she can't do implies inadequacy on the client's part. Asking the client if he or she is afraid of dying is too direct and would likely cause the client to be anxious. Asking the client how he or she is doing is too general and does not encourage the client to share thoughts on a specific topic.

A client with multiple myeloma reports bone pain that is unrelieved by analgesics. How does the nurse respond to this client's problem? a. "Ask your doctor to prescribe more medication." b. "It is too soon for additional medication to be given." c. "I'll turn on some soothing classical music for you." d. "Would you like to try some relaxation techniques?"

d. "Would you like to try some relaxation techniques?" Because most clients with multiple myeloma have local or generalized bone pain, analgesics and alternative approaches for pain management, such as relaxation techniques, are used for pain relief. This also offers the client a choice. Before prescribing additional medication, other avenues should be explored to relieve this client's pain. Even if it is too soon to give additional medication, telling that to the client is not helpful because it dismisses the client's pain concerns. Although music therapy can be helpful, this response does not give the client a choice.

The nurse is teaching a client about pelvic muscle exercises. What information does the nurse include? a. "For the best effect, perform all of your exercises while you are seated on the toilet." b. "Limit your exercises to 5 minutes twice a day, or you will injure yourself." c. "Results should be visible to you within 72 hours." d. "You know that you are exercising correct muscles if you can stop urine flow in midstream."

d. "You know that you are exercising correct muscles if you can stop urine flow in midstream." When the client can start and stop the urine stream, the pelvic muscles are being used. Pelvic muscle exercises can be performed anywhere and should be performed more often than 5 minutes twice daily. Noticeable results take several weeks.

A client is taking methimazole (Tapazole) for hyperthyroidism and would like to know how soon this medication will begin working. What is the nurse's best response? a. "You should see effects of this medication immediately." b. "You should see effects of this medication within 1 week." c. "You should see full effects from this medication within 1 to 2 days." d. "You should see some effects of this medication within 2 weeks."

d. "You should see some effects of this medication within 2 weeks." Methimazole is an iodine preparation that decreases blood flow through the thyroid gland. This action reduces the production and release of thyroid hormone. The client should see some effects within 2 weeks; however, it may take several more weeks before metabolism returns to normal. Although onset of action is 30 to 40 minutes after an oral dose, the client will not see effects immediately. Effects will take longer than 1 week to become apparent when methimazole is used. Methimazole needs to be taken every 8 hours for an extended period of time. Levels of triiodothyronine (T3) and thyroxine (T4) will be monitored and dosages adjusted as levels fall.

A client with anemia asks, "Why am I feeling tired all the time?" How does the nurse respond? a. "How many hours are you sleeping at night?" b. "You are not getting enough iron." c. "You need to rest more when you are sick." d. "Your cells are delivering less oxygen than you need."

d. "Your cells are delivering less oxygen than you need." The single most common symptom of anemia is fatigue, which occurs because oxygen delivery to cells is less than is required to meet normal oxygen needs. Although assessment of sleep and rest is good, it does not address the cause related to the diagnosis. While it may be true that the client isn't getting enough iron, it does not relate to the client's fatigue. The statement about the client needing rest because of being sick is simply not true.

The health care provider requests phenazopyridine (Pyridium) for a client with cystitis. What does the nurse tell the client about the drug? a. "It will act as an antibacterial drug." b. "This drug will treat your infection, not the symptoms of it." c. "You need to take the drug on an empty stomach." d. "Your urine will turn red or orange while on the drug."

d. "Your urine will turn red or orange while on the drug." Phenazopyridine will turn the client's urine red or orange. Clients should be warned about this effect of the drug because it will be alarming to them if they are not informed, and care should be taken because it will stain undergarments. Phenazopyridine reduces bladder pain and burning by exerting a local analgesic/anesthetic effect on the mucosa of the urinary tract. It does treat the symptoms of bladder infection; it has no antibacterial action. Phenazopyridine should be taken with a meal or immediately after eating.

In reviewing the health care provider admission requests for a client admitted in a hyperglycemic-hyperosmolar state, which request is inconsistent with this diagnosis? a. 20 mEq KCl for each liter of IV fluid b. IV regular insulin at 2 units/hr c. IV normal saline at 100 mL/hr d. 1 ampule NaHCO3 IV now

d. 1 ampule NaHCO3 IV now NaHCO3 is given for the acid-base imbalance of diabetic ketoacidosis, not the hyperglycemic-hyperosmolar state, which presents with hyperglycemia and absence of ketosis/acidosis. KCl 20 mEq for each liter of IV fluid will correct hypokalemia from diuresis. IV regular insulin at 2 units/hr will correct hyperglycemia. IV normal saline at 100 mL/hr will correct dehydration.

Which client does the nurse manager on the medical unit assign to an experienced LPN/LVN? a. A 42-year-old with painless hematuria who needs an admission assessment b. A 46-year-old scheduled for cystectomy who needs help in selecting a stoma site c. A 48-year-old receiving intravesical chemotherapy for bladder cancer d. A 55-year-old with incontinence who has intermittent catheterization prescribed

d. A 55-year-old with incontinence who has intermittent catheterization prescribed Insertion of catheters is within the education and legal scope of practice for LPN/LVNs. Admission assessments and intravesical chemotherapy should be done by an RN. Preoperative preparation for cystectomy and stoma site selection should be done by an RN and either a Certified Wound, Ostomy, and Continence Nurse or an enterostomal therapy nurse.

The nurse is starting the shift by making rounds. Which client does the nurse decide to assess first? a. A 42-year-old with anemia who is reporting shortness of breath when ambulating down the hallway b. A 47-year-old who recently had a Rumpel-Leede test and is requesting a nurse to "look at the bruises on my arm" c. A 52-year-old who has just had a bone marrow aspiration and is requesting pain medication d. A 59-year-old who has a nosebleed and is receiving heparin to treat a pulmonary embolism

d. A 59-year-old who has a nosebleed and is receiving heparin to treat a pulmonary embolism The client with the nosebleed may be experiencing the bleeding as a result of excessive anticoagulation and should be assessed for the severity of the situation first. The client with anemia and the client who had a Rumpel-Leede test are more stable, and can be assessed later. Making clients wait for pain medication is not desirable, but in this scenario, the client who is bleeding is the higher priority. The client waiting for pain medication should be next on the nurse's "to do" list.

The nurse assesses the client with which hematologic problem first? a. A 32-year-old with pernicious anemia who needs a vitamin B12 injection b. A 40-year-old with iron deficiency anemia who needs a Z-track iron injection c. A 67-year-old with acute myelocytic leukemia with petechiae on both legs d. An 81-year-old with thrombocytopenia and an increase in abdominal girth

d. An 81-year-old with thrombocytopenia and an increase in abdominal girth An increase in abdominal girth in a client with thrombocytopenia indicates possible hemorrhage; this warrants further assessment immediately. The 32-year-old with pernicious anemia, the 40-year-old with iron deficiency anemia, and the 67-year-old with acute myelocytic leukemia do not indicate any acute complications, so the nurse can assess them after assessing the client with thrombocytopenia.

A client with type 2 diabetes who is taking metformin (Glucophage) is seen in the diabetic clinic. The fasting blood glucose is 108 mg/dL, and the glycosylated hemoglobin (HbA1C) is 8.2%. Which action does the nurse plan to take next? a. Instruct the client to continue with the current diet and metformin use. b. Discuss the need to check blood glucose several times every day. c. Talk about the possibility of adding rapid-acting insulin to the regimen. d. Ask the client about current dietary intake and medication use.

d. Ask the client about current dietary intake and medication use. The nurse's first action should be to assess whether the client is adherent to the currently prescribed diet and medications. The client's current diet and medication use have not been successful in keeping glucose in the desired range. Checking blood glucose more frequently and/or using rapid-acting insulin may be appropriate, but this will depend on the assessment data. The HbA1C indicates that the client's average glucose level is not in the desired range, but discussing the need to check blood glucose several times every day assumes that the client is not compliant with the therapy and glucose monitoring. The nurse should not assume that adding insulin, which must be prescribed by the provider, is the answer without assessing the underlying reason for the treatment failure.

The nurse prepares to perform a neurovascular assessment on a client with closed multiple fractures of the right humerus. Which technique does the nurse use? a. Inspect the abdomen for tenderness and bowel sounds. b. Auscultate lung sounds. c. Assess the level of consciousness and ability to follow commands. d. Assess sensation of the right upper extremity.

d. Assess sensation of the right upper extremity. Assessing sensation of the right upper extremity is part of a focused neurovascular assessment for the client with multiple fractures of the right humerus. Inspecting the abdomen and auscultating lung sounds of the client with multiple fractures are not part of a focused neurovascular assessment. Because the client does not have a head injury, assessing the client's level of consciousness and ability to follow commands is not part of a focused neurovascular assessment.

The nurse reviews the vital signs of a client diagnosed with Graves' disease and sees that the client's temperature is up to 99.6° F. After notifying the health care provider, what does the nurse do next? a. Administers acetaminophen b. Alerts the Rapid Response Team c. Asks any visitors to leave d. Assesses the client's cardiac status completely

d. Assesses the client's cardiac status completely If the client's temperature has increased by even 1°, the nurse's first action is to notify the provider. Continuous cardiac monitoring should be the next step. Administering a nonsalicylate antipyretic such as acetaminophen is appropriate, but is not a priority action for this client. Alerting the Rapid Response Team is not needed at this time. Asking visitors to leave would not be the next action, and if visitors are providing comfort to the client, this would be contraindicated.

Which nursing action can the home health nurse delegate to a home health aide who is making daily visits to a client with newly diagnosed type 2 diabetes? a. Assist the client's spouse in choosing appropriate dietary items. b. Evaluate the client's use of a home blood glucose monitor. c. Inspect the extremities for evidence of poor circulation. d. Assist the client with washing the feet and applying moisturizing lotion.

d. Assist the client with washing the feet and applying moisturizing lotion. Assisting with personal hygiene is included in the role of home health aides. Assisting with dietary choices, evaluating the effectiveness of teaching, and performing assessments are complex actions that should be implemented by licensed nurses.

Which nursing action does the nurse on the orthopedic unit plan to delegate to unlicensed assistive personnel (UAP)? a. Remove the wound drain for a client who had an open reduction of a hip fracture 3 days ago. b. Assess for bruising on a client who is receiving warfarin (Coumadin) to prevent deep vein thrombosis. c. Teach a client with a right ankle fracture how to use crutches when transferring and ambulating. d. Check the vital signs for a client who was admitted after a total knee replacement 3 hours ago

d. Check the vital signs for a client who was admitted after a total knee replacement 3 hours ago Vital sign assessment is a skill that is within the role of the UAP. Removing a wound drain, assessment, and client teaching are nursing actions that require broader education and are within the scope of practice of licensed nursing staff.

Which technique does the nurse use to obtain a sterile urine specimen from a client with a Foley catheter? a. Disconnect the Foley catheter from the drainage tube and collect urine directly from the Foley. b. Remove the existing catheter and obtain a sample during the process of inserting a new Foley. c. Use a sterile syringe to withdraw urine from the urine collection bag. d. Clamp the tubing, attach a syringe to the specimen, and withdraw at least 5 mL of urine.

d. Clamp the tubing, attach a syringe to the specimen, and withdraw at least 5 mL of urine. Clamping the tubing, attaching a syringe to the specimen, and withdrawing at least 5 mL of urine is the correct technique for obtaining a sterile urine specimen from the client with a Foley catheter. Disconnecting the Foley catheter from the drainage tube and collecting urine directly from the Foley increases the risk for microbe exposure. A Foley catheter should not be removed to get a urine sample. Microbes may be in the urine collection bag from standing urine, so using a sterile syringe to withdraw urine from the urine collection bag is not the proper technique to obtain a urine sample.

The RN has just received change-of-shift report on the medical-surgical unit. Which client will need to be assessed first? a. Client with Hashimoto's thyroiditis and a large goiter b. Client with hypothyroidism and an apical pulse of 51 beats/min c. Client with parathyroid adenoma and flank pain due to a kidney stone d. Client who had a parathyroidectomy yesterday and has muscle twitching

d. Client who had a parathyroidectomy yesterday and has muscle twitching A client who is 1 day postoperative for parathyroidectomy and has muscle twitching is showing signs of hypocalcemia and is at risk for seizures. Rapid assessment and intervention are needed. Clients with Hashimoto's thyroiditis are usually stable; this client does not need to be assessed first. Although an apical pulse of 51 is considered bradycardia, a low heart rate is a symptom of hypothyroidism. A client with a kidney stone will be uncomfortable and should be asked about pain medication as soon as possible, but this client does not need to be assessed first.

The charge nurse is making client assignments for the medical-surgical unit. Which client will be best to assign to an RN who has floated from the pediatric unit? a. Client in Addisonian crisis who is receiving IV hydrocortisone b. Client admitted with syndrome of inappropriate antidiuretic hormone (SIADH) secondary to lung cancer c. Client being discharged after a unilateral adrenalectomy to remove an adrenal tumor d. Client with Cushing's syndrome who has elevated blood glucose and requires frequent administration of insulin

d. Client with Cushing's syndrome who has elevated blood glucose and requires frequent administration of insulin An RN who works with pediatric clients would be familiar with glucose monitoring and insulin administration. A client in Addisonian crisis would best be monitored by an RN from the medical-surgical floor. Although the float RN could complete the admission history, the client with SIADH secondary to lung cancer might require teaching and orientation to the unit that a nurse more familiar with that area would be better able to provide. Discharge teaching specific to adrenalectomy should be provided by the RN who is regularly assigned to the medical-surgical floor and is more familiar with taking care of postoperative adult clients with endocrine disorders.

The nurse has these client assignments. Which client does the nurse encourage to consume 2 to 3 liters of fluid each day? a. Client with chronic kidney disease b. Client with heart failure c .Client with complete bowel obstruction d. Client with hyperparathyroidism

d. Client with hyperparathyroidism A major feature of hyperparathyroidism is hypercalcemia, which predisposes a client to kidney stones; this client should remain hydrated. A client with chronic kidney disease should not consume 2 to 3 liters of water because the kidneys are not functioning properly, and this could lead to fluid retention. People with heart failure typically have fluid volume excess. A client with complete bowel obstruction may experience vomiting and should be NPO.

The nurse receives report on a 52-year-old client with type 2 diabetes: Lungs clear Glucose 179 mg/dL Regular insulin 8 units if blood glucose 250 to 275 mg/dL and cold to touch Right great toe mottled Hemoglobin A1c 6.9% Regular insulin 10 units if glucose 275 to 300 mg/dL Client states wears eyeglasses to read Which complication of diabetes does the nurse report to the provider? a. Poor glucose control b. Visual changes c. Respiratory distress d. Decreased peripheral perfusion

d. Decreased peripheral perfusion A cold, mottled toe may indicate arterial occlusion secondary to arterial occlusive disease or embolization; this must be reported to avoid potential gangrene and amputation. Although one glucose reading is elevated, the hemoglobin A1c indicates successful glucose control over the past 3 months. After the age of 40, reading glasses may be needed due to difficulty in accommodating to close objects. Lungs are clear and no evidence of distress is noted.

What does the nurse teach a client to do to decrease the risk for urinary tract infection (UTI)? a. Limit fluid intake. b. Increase caffeine consumption. c. Limit sugar intake. d. Drink about 3 liters of fluid daily.

d. Drink about 3 liters of fluid daily. Drinking about 3 liters of fluid daily, if another medical problem does not require fluid restriction, helps prevent dehydration and UTIs. Fluids flush the system and should not be limited. Increased caffeine intake and limiting sugar intake will not prevent UTIs.

The nurse is assessing the nutritional status of a client with anemia. How does the nurse obtain information about the client's diet? a. Asks the client to rate his or her diet on a scale of 1 (poor) to 10 (excellent) b. Determines who prepares the client's meals and plans an interview with him or her c. From a prepared list, finds out the client's food preferences d. Has the client write down everything he or she has eaten for the past week

d. Has the client write down everything he or she has eaten for the past week Having the client provide a list of items eaten in the past week is the most accurate way to find out what the client likes and dislikes, as well as what the client has been eating. It will provide information about "junk" food intake, as well as protein, vitamin, and mineral intake. Rating scales are good for subjective data collection about some conditions such as pain, but the subjectivity of a response such as this does not provide the nurse with specific data needed to assess a diet. Interviewing the food preparer is time-consuming and poses several problems, such as whether a number of people are preparing meals, or if the client goes "out" for meals. Determining food preferences from a prepared list provides information about what the client enjoys eating, not necessarily what the client has been eating; for instance, the client may like steak but may be unable to afford it.

An 82-year-old client with anemia is requested to receive 2 units of whole blood. Which assessment findings cause the nurse to discontinue the transfusion because it is unsafe for the client? (Select all that apply.) a. Capillary refill less than 3 seconds b. Decreased pallor c. Flattened superficial veins d. Hypertension Correct e. Hypotension Correct f. Rapid, bounding pulse Correct

d. Hypertension e. Hypotension f. Rapid, bounding pulse In an older adult receiving a transfusion, hypertension is a sign of overload, low blood pressure is a sign of a transfusion reaction, and a rapid and bounding pulse is a sign of fluid overload. In this scenario, 2 units, or about a liter of fluid, could be problematic. Capillary refill time that is less than 3 seconds is considered to be normal and would not pose a problem. Increased (not decreased) pallor and cyanosis are signs of a transfusion reaction, while swollen (not flattened) superficial veins are present in fluid overload in older adult clients receiving transfusions.`

What effect can starting a dose of levothyroxine sodium (Synthroid) too high or increasing a dose too rapidly have on a client? a. Bradycardia and decreased level of consciousness b. Decreased respiratory rate c. Hypotension and shock d. Hypertension and heart failure

d. Hypertension and heart failure Hypertension and heart failure are possible if the levothyroxine sodium dose is started too high or raised too rapidly, because levothyroxine would essentially put the client into a hyperthyroid state. The client would be tachycardic, not bradycardic. The client may have an increased respiratory rate. Shock may develop, but only as a late effect and as the result of "pump failure."

A client who has been newly diagnosed with leukemia is admitted to the hospital. Avoiding which potential problem takes priority in the client's nursing care plan? a. Fluid overload (overhydration) b. Hemorrhage c. Hypoxia d. Infection

d. Infection The main objective in caring for a newly diagnosed client with leukemia is protection from infection. Fluid overload, hemorrhage, and hypoxia are not priority problems for the client with leukemia.

Which staff member does the charge nurse assign to a client who has benign prostatic hyperplasia and hydronephrosis and needs an indwelling catheter inserted? a. RN float nurse who has 10 years of experience with pediatric clients b. LPN/LVN who has worked in the hospital's kidney dialysis unit until recently c. RN without recent experience who has just completed an RN refresher course d. LPN/LVN with 5 years of experience in an outpatient urology surgery center

d. LPN/LVN with 5 years of experience in an outpatient urology surgery center Catheterization of a client with an enlarged prostate, a skill within the scope of practice of the LPN/LVN, would be performed frequently in a urology center. The pediatric nurse would have little exposure to prostatic obstruction and adult catheterization. Dialysis clients do not typically have catheters inserted, so the LPN/LVN from the kidney dialysis unit would not be the best staff member to assign to the client. The nurse who has been out of practice for several years is not the best candidate to insert a catheter in a client with an enlarged prostate.

Which assessment finding represents a positive response to erythropoietin (Epogen, Procrit) therapy? a. Hematocrit of 26.7% b. Potassium within normal range c. Absence of spontaneous fractures d. Less fatigue

d. Less fatigue Treatment of anemia with erythropoietin will result in increased hemoglobin and hematocrit (H&H) and decreased shortness of breath and fatigue. A hematocrit value of 26.7% is low; erythropoietin should restore the hematocrit to at least 36% to be effective. Erythropoietin causes more red blood cells to be produced, increasing H&H, not potassium. Calcium supplements and phosphate binders prevent renal osteodystrophy; erythropoietin treats anemia.

The nurse is conducting a musculoskeletal history in an older adult client who requires a caregiver to perform all activities of daily living (ADLs). Which level of functioning does the nurse record in the client's history using Gordon's Functional Health Patterns? a. Level 0 b. Level II c. Level III d. Level IV

d. Level IV Level IV indicates that the client is dependent and does not participate in ADLs such as dressing him- or herself. Level 0 indicates a client who is able to perform full self-care. Level II indicates a client who requires assistance or supervision of another person without assistive equipment or devices. Level III indicates that the client requires the assistance or supervision of another person, as well as assistive equipment or devices.

Which goal for a client with diabetes will best help to prevent diabetic nephropathy? a. Heed the urge to void. b. Avoid carbohydrates in the diet. c. Take insulin at the same time every day. d. Maintain glycosylated hemoglobin (HbA1c).

d. Maintain glycosylated hemoglobin (HbA1c). Maintaining long-term control of blood glucose will help prevent the progression of diabetic nephropathy. Voiding when the client has the urge prevents the backflow of urine and infection. The diabetic diet is composed of carbohydrates, proteins, and fats. Although taking insulin at the same time each day may indirectly help control blood glucose, it is not the best option.

The ambulatory surgery postanesthesia care unit (PACU) nurse has just received report about clients who had arthroscopic surgery. Which client will the nurse plan to assess first? a. Young adult client who has been in the PACU for 30 minutes after left knee arthroscopy under local anesthesia b. Adult client who had a synovial biopsy of the right knee under local anesthesia and has been in the PACU for 20 minutes c. Adult client who has multiple right knee incisions for repair of torn cartilage and arrived in the PACU an hour ago d. Middle-aged adult client who returned to the PACU 25 minutes ago after left knee arthroscopic surgery under epidural anesthesia

d. Middle-aged adult client who returned to the PACU 25 minutes ago after left knee arthroscopic surgery under epidural anesthesia After epidural anesthesia, frequent assessments for the return of sensation and movement of the leg will be important. The client who had knee arthroscopic surgery under epidural anesthesia is at greatest risk for complications and should be assessed first. The clients who had local anesthesia for knee arthroscopy, the client who had a synovial biopsy of the right knee, and the client who had multiple right knee incisions are all at less risk for developing complications.

The nurse plans to refer a client diagnosed with osteoporosis to which community resource? a. American Bone Society b. CanSurmount c. I Can Cope d. National Osteoporosis Foundation

d. National Osteoporosis Foundation Clients with musculoskeletal problems should be referred to appropriate community resources, such as the National Osteoporosis Foundation, for help and support for their diagnosis. There is no organization known as the American Bone Society. CanSurmount is a cancer support group geared toward client and family education. I Can Cope is also a support group for clients with cancer.

When caring for a client with hemorrhage secondary to kidney trauma, the nurse provides volume expansion. Which element does the nurse anticipate will be used? a. Fresh-frozen plasma b. Platelet infusions c. 5% dextrose in water d. Normal saline solution (NSS)

d. Normal saline solution (NSS) Isotonic solutions and crystalloid solutions are administered for volume expansion; 0.9% sodium chloride (NSS) and 5% dextrose in 0.45% sodium chloride may be used. Clotting factors, contained in fresh-frozen plasma, are given for bleeding, not for volume expansion. Platelet infusions are administered for deficiency of platelets. A solution hypotonic to the client's blood, 5% dextrose, is administered for nutrition or hypernatremia, not for volume expansion.

A client is scheduled for a cystoscopy later this morning. The consent form is not signed, and the client has not had any preoperative medication. The nurse notes that the provider visited the client the day before. What action does the nurse take? a.Asks the client to sign the informed consent b.Cancels the procedure c.Asks the client's spouse to sign the form d.Notifies the department and the provider

d. Notifies the department and the provider The client may be asked to sign the consent form in the department; notifying both the provider and the department ensures communication across the continuum of care, with less likelihood of omission of information. The provider gives the client a complete description of and reasons for the procedure and explains complications; the nurse reinforces this information. The procedure should not be cancelled without an attempt to correct the situation. The client has not received sedation, so nothing suggests that the client is not competent to consent.

Which would be an appropriate task to delegate to unlicensed assistive personnel (UAP) working on a medical-surgical unit? a. Administering erythropoietin to a client with myelodysplastic syndrome b. Assessing skin integrity on an anemic client who fell during ambulation c. Assisting a client with folic acid deficiency in making diet choices d. Obtaining vital signs on a client receiving a blood transfusion

d. Obtaining vital signs on a client receiving a blood transfusion Obtaining vital signs on a client is within the scope of practice for UAP. Administering medication, assessing clients, and assisting with prescribed diet choices are complex actions that should be done by licensed nurses.

The charge nurse in the hospital-based day surgery center is making client assignments for the staff. Which client is most appropriate to assign to a nurse who has floated from the general surgical unit? a. Young adult who has just been admitted for surgery after sustaining an ankle fracture b. Adult who needs teaching about quadriceps-setting exercises after knee arthroscopy c. Middle-aged adult who will require a pneumatic tourniquet applied before knee surgery `d. Older adult who has undergone arthroscopic surgery of the shoulder under local anesthesia

d. Older adult who has undergone arthroscopic surgery of the shoulder under local anesthesia Arthroscopic surgery and local anesthesia have low complication rates and could be monitored by the float nurse, who would be expected to know how to assess neurovascular status. The young, newly admitted client requires assessment that will be best performed by nurses with more experience in day surgery. Client teaching for the adult client who has had arthroscopic knee surgery is best completed by nurses with more experience in day surgery. The middle-aged adult who needs a pneumatic tourniquet requires an intervention that is best performed by nurses with more experience in day surgery.

A client has had a sequestrectomy of the right fibula for osteomyelitis 1 day ago. Which assessment finding requires the nurse to immediately contact the surgeon? a. Swelling of the right lower extremity b. 1+ to 2+ bilateral palpable pedal pulses c. Pain of right lower extremity on movement d. Paresis of right lower extremity

d. Paresis of right lower extremity Paresis indicates a neurovascular compromise that must be reported immediately to the surgeon. The client undergoing a sequestrectomy experiences increased swelling after the procedure; the affected extremity should be elevated to increase venous return and thus control swelling. Palpable pulses of 1+ to 2+ bilaterally are a sign of adequate blood flow. Pain on movement of the right lower extremity is an expected finding.

An older adult client has multiple tibia and fibula fractures of the left lower extremity after a motor vehicle crash. Which pain medication does the nurse anticipate will be requested for this client? a. Cyclobenzaprine (Flexeril) b. Ibuprofen (Advil) c. Meperidine (Demerol) d. Patient-controlled analgesia (PCA) with morphine

d. Patient-controlled analgesia (PCA) with morphine Morphine is an opioid narcotic analgesic; given through PCA, it is the most appropriate mode of pain management for this type of acute pain associated with multiple injuries. Muscle relaxants such as cyclobenzaprine are effective for treating pain related to muscle spasms, but they are not adequate for this type of acute pain. Ibuprofen is a nonsteroidal anti-inflammatory drug that is used to treat mild to moderate pain; bone pain is very acute, so ibuprofen would not be sufficient. Meperidine should never be used for older adults because it has toxic metabolites that can cause seizures.

Which nursing intervention or practice is most effective in helping to prevent urinary tract infection (UTI) in hospitalized clients? a. Encouraging them to drink fluids b. Irrigating all catheters daily with sterile saline c. Recommending that catheters be placed in all clients d. Periodically re-evaluating the need for indwelling catheters

d. Periodically re-evaluating the need for indwelling catheters Studies have shown that re-evaluating the need for indwelling catheters in clients is the most effective way to prevent UTIs in the hospital setting. Encouraging fluids, although it is a valuable practice for clients with catheters, will not necessarily prevent the occurrence of UTIs in the hospital setting. In some clients, their conditions do not permit an increase in fluids, such as those with congestive heart failure and kidney failure. Irrigating catheters daily is contraindicated; any time a closed system is opened, bacteria may be introduced. Placing catheters in all clients is unnecessary and unrealistic. This practice would place more clients at risk for the development of UTI.

The nurse is reviewing the medication history for a client scheduled for a left total hip replacement. The nurse plans to contact the health care provider if the client is taking which medication? a. Acetaminophen (Tylenol) for pain relief b. Bupropion (Wellbutrin) for smoking cessation c. Magnesium hydroxide (Milk of Magnesia) to treat heartburn d. Prednisone (Deltasone) to treat asthma

d. Prednisone (Deltasone) to treat asthma Long-term steroid use is strongly associated with osteoporosis and will increase the risk for poor wound healing and prolonged recovery after the hip replacement. Taking acetaminophen for pain relief, bupropion for smoking cessation, or magnesium hydroxide to treat heartburn will not influence the potential success of the surgery.

Which problem excludes a client hoping to receive a kidney transplant from undergoing the procedure? a. History of hiatal hernia b. Presence of diabetes and glycosylated hemoglobin of 6.8% c. History of basal cell carcinoma on the nose 5 years ago d. Presence of tuberculosis

d. Presence of tuberculosis Long-standing pulmonary disease and chronic infection typically exclude clients from transplantation; these conditions worsen with the immune suppressants that are required to prevent rejection. A client with a history of hiatal hernia is not exempt from undergoing a kidney transplant. Good control of diabetes is a positive point, and would not exclude the client from transplantation. Basal cell carcinoma is considered curable and occurred 5 years ago, consistent with appropriate candidates for transplantation.

A client had a computed tomography (CT) scan with contrast dye 8 hours ago. Which nursing intervention is the priority for this client? a. Maintaining bedrest b. Medicating for pain c. Monitoring for hematuria d. Promoting fluid intake

d. Promoting fluid intake The nurse should ensure adequate hydration by urging the client to take oral fluid or by giving IV fluids. Hydration reduces the risk for kidney damage. Bedrest is not indicated for the client who has undergone a CT scan with contrast dye. CT with contrast dye is not a painful procedure, so pain medication is not indicated. The client who has undergone CT with contrast dye is not at risk for hematuria.

When preparing a client for nephrostomy tube insertion, it is essential for the nurse to monitor which factor before the procedure? a. Blood urea nitrogen (BUN) and creatinine b. Hemoglobin and hematocrit (H&H) c. Intake and output (I&O) d. Prothrombin time (PT) and international normalized ratio (INR)

d. Prothrombin time (PT) and international normalized ratio (INR) The procedure will be cancelled or delayed if coagulopathy in the form of prolonged PT/INR exists because dangerous bleeding may result. Nephrostomy tubes are placed to prevent and treat kidney damage; monitoring BUN and creatinine is important, but is not essential before this procedure. H&H is monitored to detect anemia and blood loss; this would not occur before the procedure. This client should be on I&O during the entire hospitalization; it is not necessary only before the procedure, but throughout the admission.

The nurse is caring for a client who is in sickle cell crisis. What action does the nurse perform first? a. Apply cool compresses to the client's forehead. b. Encourage the client's use of two methods of birth control. c. Increase food sources of iron in the client's diet. d. Provide pain medications as needed.

d. Provide pain medications as needed. Analgesics are needed to treat sickle cell pain. Warm soaks or compresses can help reduce pain perception. Cool compresses do not help the client in sickle cell crisis. Birth control is not the priority for this client. Increasing iron in the diet is not pertinent for the client in sickle cell crisis.

A client with pyelonephritis has been prescribed urinary antiseptic medication. What purpose does this medication serve? a. Decreases bacterial count b. Destroys white blood cells c. Enhances the action of antibiotics d. Provides comfort

d. Provides comfort Urinary antiseptic drugs such as nitrofurantoin (Macrodantin) are prescribed to provide comfort for clients with pyelonephritis. Antibiotics, not antiseptics, are used to decrease bacterial count and treat pyelonephritis infection; the action of antibiotics is not enhanced with antiseptics. White blood cells, along with antibiotics, fight infection.

A client who is 6 months pregnant comes to the prenatal clinic with a suspected urinary tract infection (UTI). What action does the nurse take with this client? a. Discharges the client to her home for strict bedrest for the duration of the pregnancy b. Instructs the client to drink a minimum of 3 liters of fluids, especially water, every day to "flush out" bacteria c. Recommends that the client refrain from having sexual intercourse until after she has delivered her baby d. Refers the client to the clinic nurse practitioner for immediate follow-up

d. Refers the client to the clinic nurse practitioner for immediate follow-up Pregnant women with UTIs require prompt and aggressive treatment because simple cystitis can lead to acute pyelonephritis. This in turn can cause preterm labor with adverse effects for the fetus. It is unsafe for the client to be sent home without analysis of the symptoms that she has. Her problem needs to be investigated without delay. Although drinking increased amounts of fluids is helpful, it will not cure an infection. Having sexual intercourse (or not having it) is not related to the client's problem. The client's symptoms need follow-up with a health care provider.

A client with a possible adrenal gland tumor is admitted for testing and treatment. Which nursing action is most appropriate for the charge nurse to delegate to the nursing assistant? a. Assess skin turgor and mucous membranes for hydration status. b. Discuss the dietary restrictions needed for 24-hour urine testing. c. Plan ways to control the environment that will avoid stimulating the client. d. Remind the client to avoid drinking coffee and changing position suddenly.

d. Remind the client to avoid drinking coffee and changing position suddenly. Drinking caffeinated beverages and changing position suddenly are not safe for a client with a potential adrenal gland tumor because of the effects of catecholamines. Reminding the client about previous instructions is an appropriate role for a nursing assistant who may observe the client doing potentially risky activities. Client assessment, client teaching, and environment planning are higher-level skills that require the experience and responsibility of the RN, and are not within the scope of practice of the nursing assistant.

Which action does the nurse delegate to unlicensed assistive personnel (UAP)? a. Drawing a partial thromboplastin time from a saline lock on a client with a pulmonary embolism b. Performing a capillary fragility test to check vascular hemostatic function on a client with liver failure c. Referring a client with a daily alcohol consumption of 12 beers for counseling d. Reporting any bleeding noted when catheter care is given to a client with a history of hemophilia

d. Reporting any bleeding noted when catheter care is given to a client with a history of hemophilia Reporting findings during routine care is expected and required of unlicensed staff members. Drawing a partial thromboplastin time, performing a capillary fragility test, and referring a client for alcohol counseling are more complex and should be done by licensed nursing staff.

A client is brought to the emergency department via ambulance after a motor vehicle crash. What condition does the nurse assess for first? a. Bleeding b. Head injury c. Pain d. Respiratory distress

d. Respiratory distress The client should first be assessed for respiratory distress, and any oxygen interventions instituted accordingly. Bleeding is the second assessment priority, head injury is the third assessment priority, and pain is the fourth assessment priority in this case.

The nurse is reviewing the laboratory test results for a client admitted with a possible pituitary disorder. Which information has the most immediate implication for the client's care? a. Blood glucose 125 mg/dL b. Blood urea nitrogen (BUN) 40 mg/dL c. Serum potassium 5.2 mEq/L d. Serum sodium 110 mEq/L

d. Serum sodium 110 mEq/L The normal range for serum sodium is 135 to 145 mEq/L; a result of 110 mEq/L is considered hyponatremia and is extremely dangerous. The client is at risk for increased intracranial pressure, seizures, and death. The RN must act rapidly because this situation requires immediate intervention. The normal range for fasting blood glucose is 60 to 110 mg/dL; 125 mg/dL is high, but is not considered dangerous. The normal range for BUN is 7 to 20 mg/dL; 40 mg/dL is high. An elevated BUN can be an indication of kidney failure, dehydration, fever, increased protein intake, and shock, so the client should have a creatinine drawn for a more complete picture of kidney function. The normal range for serum potassium is 3.5 to 5.2 mEq/L; 5.2 mEq/L is high normal.

What is the most important environmental risk for developing leukemia? a. Direct contact with others with leukemia b. Family history c. Living near high-voltage power lines d. Smoking cigarettes

d. Smoking cigarettes According to the American Cancer Society (ACS), the only proven lifestyle-related risk factor for leukemia is cigarette smoking. Leukemia is not contagious. Genetics is a strong indicator, but it is not an environmental risk factor. According to the ACS, living near high-voltage power lines is not a proven risk factor for leukemia.

A client who is receiving a blood transfusion suddenly exclaims to the nurse, "I don't feel right!" What does the nurse do next? a. Call the Rapid Response Team. b. Obtain vital signs and continue to monitor. c. Slow the infusion rate of the transfusion. d. Stop the transfusion.

d. Stop the transfusion. The client may be experiencing a transfusion reaction; the nurse should stop the transfusion immediately. Calling the Rapid Response Team or obtaining vital signs is not the first thing that should be done. The nurse should not slow the infusion rate, but should stop it altogether.

The nurse is caring for clients on a renal/kidney medical-surgical unit. Which drug, requested by the health care provider for a client with a urinary tract infection (UTI), does the nurse question? a. Bactrim b. Cipro c. Noroxin d. Tegretol

d. Tegretol Drug alerts state that confusion is frequent (sound alike and look alike) between the drugs Tequin (gatifloxacin) and Tegretol (carbamazepine). The former is used for UTI, and the latter is prescribed as an oral anticonvulsant. Bactrim (trimethoprim/sulfamethoxazole), Cipro (ciprofloxacin), and Noroxin (norfloxacin) are drugs used to treat UTI.

When caring for a client who receives peritoneal dialysis (PD), which finding does the nurse report to the provider immediately? a. Pulse oximetry reading of 95% b. Sinus bradycardia, rate of 58 beats/min c. Blood pressure of 148/90 mm Hg d. Temperature of 101.2° F (38.4° C)

d. Temperature of 101.2° F (38.4° C) Peritonitis is the major complication of PD, caused by intra-abdominal catheter site contamination; meticulous aseptic technique must be used when caring for PD equipment. A pulse oximetry reading of 95% is a normal saturation. Although a heart rate of 58 beats/min is slightly bradycardic, the provider can be informed upon visiting the client. Clients with kidney failure tend to have slightly higher blood pressures due to fluid retention; this is not as serious as a fever.

A client is referred to a home health agency after a transsphenoidal hypophysectomy. Which action does the RN case manager delegate to the home health aide who will see the client daily? a. Document symptoms of incisional infection or meningitis. b. Give over-the-counter laxatives if the client is constipated. c. Set up medications as prescribed for the day. d. Test any nasal drainage for the presence of glucose.

d. Test any nasal drainage for the presence of glucose. Cerebrospinal fluid (CSF) will test positive using a glucose "dipstick." Nasal drainage that is positive for glucose after a transsphenoidal hypophysectomy would indicate a CSF leak that would require immediate notification of the health care provider. Home health aides can be taught the correct technique to perform this procedure. Assessing for symptoms of infection and documenting them in the record, medication administration, and setting up medication are not within the scope of practice of the home health aide.

A client is referred to a home health agency after being hospitalized with overflow incontinence and a urinary tract infection. Which nursing action can the home health RN delegate to the home health aide (unlicensed assistive personnel [UAP])? a. Assisting the client in developing a schedule for when to take prescribed antibiotics b. Inserting a straight catheter as necessary if the client is unable to empty the bladder c. Teaching the client how to use the Credé maneuver to empty the bladder more fully d. Using a bladder scanner (with training) to check residual bladder volume after the client voids

d. Using a bladder scanner (with training) to check residual bladder volume after the client voids Use of a bladder scanner is noninvasive and can be accomplished by a home health aide (UAP) who has been trained and evaluated in this skill. Assisting the client in developing a schedule for when to take prescribed antibiotics, inserting a straight catheter, and teaching the client to use the Credé maneuver all require more education and are in the legal scope of practice of the LPN/LVN or RN.

A client is scheduled for a bone marrow aspiration. What does the nurse do before taking the client to the treatment room for the biopsy? a. Clean the biopsy site with an antiseptic or povidone-iodine (Betadine). b. Hold the client's hand and ask about concerns. c. Review the client's platelet (thrombocyte) count. d. Verify that the client has given informed consent.

d. Verify that the client has given informed consent. Verifying informed consent must be done before the procedure can be performed. A signed permit must be on the client's chart. Cleaning the biopsy site is done before the procedure, but this is not done until consent is verified; it will be done just before the procedure is performed. Holding the client's hand and offering verbal support may be done during the procedure, but the procedure cannot be completed until the consent is signed. Reviewing the client's platelet count is not imperative.

The nurse is mentoring a recent graduate RN about administering blood and blood products. What does the nurse include in the data? a. Obtain the client's initial set of vital signs (VS) within the first 10 minutes of the infusion. b. Remain with the client who is receiving the blood for the first 5 minutes of the infusion. c. Use a 22-gauge needle to obtain venous access when starting the infusion. d. Verify with another RN all of the data on blood products.

d. Verify with another RN all of the data on blood products. All data are checked by two RNs. Human error is the most common cause of ABO incompatibilities in administering blood and blood products. Initial VS should be recorded before the start of infusion of blood, not after it has begun. The nurse remains with the client for the first 15 to 30 minutes (not 5) of the infusion. This is the period when any transfusion reactions are likely to happen. A 20-gauge needle (or a central line catheter) is used; the 22-gauge needle is too small.

The nurse is transfusing a unit of whole blood to a client when the health care provider requests the following: "Furosemide (Lasix) 20 mg IV push." What does the nurse do? a. Add furosemide to the normal saline that is infusing with the blood. b. Administer furosemide to the client intramuscularly (IM). c. Piggyback furosemide into the infusing blood. d. Wait until the transfusion has been completed to administer furosemide.

d. Wait until the transfusion has been completed to administer furosemide. Completing the transfusion before administering furosemide is the best course of action in this scenario. Drugs are not to be administered with infusing blood products; they can interact with the blood, causing risks for the client. Stopping the infusing blood to administer the drug and then restarting it is also not the best decision. Changing the admission route is not a nursing decision.

Which is the best way to decrease the risk for osteoporosis in a client who has just been determined to be at high risk for the disease? a. Increase nutritional intake of calcium. b. Engage in high-impact exercise, such as running. c. Increase nutritional intake of phosphorus. d. Walk for 30 minutes three times a week.

d. Walk for 30 minutes three times a week. Walking for 30 minutes three to five times a week is the single most effective exercise for osteoporosis prevention. Walking is a safe way to promote weight bearing and muscle strength. A variety of nutrients are needed to maintain bone health, so the promotion of a single nutrient will not prevent or treat osteoporosis. High-impact exercise and overtraining, such as running, may cause vertebral compression fractures and should be avoided. Calcium loss occurs at a more rapid rate when intake of phosphorus is high; people who drink large amounts of carbonated beverages each day (over 40 ounces) are at high risk for calcium loss and subsequent osteoporosis, regardless of age or gender.

The nurse is caring for a client with hypercortisolism. The nurse begins to feel the onset of a cold but still has 4 hours left in the shift. What does the nurse do? a. Asks another nurse to care for the client b. Monitors the client for cold-like symptoms c. Refuses to care for the client d. Wears a facemask when caring for the client

d. Wears a facemask when caring for the client A client with hypercortisolism will be immune-suppressed. Anyone with a suspected upper respiratory infection who must enter the client's room must wear a mask to prevent the spread of infection. Although asking another nurse to care for the client might be an option in some facilities, it is not generally realistic or practical. The nurse, not the client, feels the onset of the cold, so monitoring the client for cold-like symptoms is part of good client care for a client with hypercortisolism. Refusing to care for the client after starting care would be considered abandonment.

A client with Cushing's disease says that she has lost 1 pound. What does the nurse do next? a. Auscultates the lungs for crackles b. Checks urine for specific gravity c. Forces fluids d. Weighs the client

d. Weighs the client Fluid retention with weight gain is more of a problem than weight loss in clients with Cushing's disease. Weighing the client with Cushing's disease is part of the nurse's assessment. Crackles in the lungs indicate possible fluid retention, which would cause weight gain, not weight loss. Urine specific gravity will help assess hydration status, but this would not be the next step in the client's assessment. Forcing fluids is not appropriate because usually excess water and sodium reabsorption cause fluid retention in the client with Cushing's disease.

The nurse is caring for a client with neutropenia. Which clinical manifestation indicates that an infection is present or should be ruled out? a. Coughing and deep breathing b. Evidence of pus c. Fever of 102° F or higher d. Wheezes or crackles

d. Wheezes or crackles Wheezes or crackles in the neutropenic client may be the first symptom of infection in the lungs. Coughing and deep breathing are not indications of infection, but can help prevent it. The client with leukopenia, not neutropenia, may have a severe infection without pus or with only a low-grade fever.

A client recently admitted with new-onset type 2 diabetes will be discharged with a self-monitoring blood glucose machine. When is the best time for the nurse to explain to the client the proper use of the machine? a. Day of discharge b. On admission c. When the client states readiness d. While performing the test in the hospital

d. While performing the test in the hospital Teaching the client about the operation of the machine while performing the test in the hospital is the best way for the client to learn. The teaching can be reinforced before discharge. Instructing the client on the day of admission or the day of discharge would be overwhelming to the client because of all of the other activities taking place on those days. The client may never feel ready to learn this daunting task; the nurse must be more proactive.

One of the nurse's roles is talking to adult clients about urinary and sexual hygiene. Which words does the nurse use when referring to the client's reproductive body parts? a. Children's terms that are easily understood b. Slang words and terms that are heard "socially" c. Technical and medical terminology d. Words that the client uses

d. Words that the client uses The nurse should use the terms with which the client is most familiar, so there is no chance for the client to misunderstand information. Using the client's language ensures the comfort level for the client. The use of children's terms is demeaning to adult clients. The use of slang terms is unprofessional. Technical terms should not be used because the client may not know what they mean.

How does the drug desmopressin (DDAVP) decrease urine output in a client with diabetes insipidus (DI)? a. Blocks reabsorption of sodium b. Increases blood pressure c. Increases cardiac output d. Works as an antidiuretic hormone (ADH) in the kidneys

d. Works as an antidiuretic hormone (ADH) in the kidneys Desmopressin is a synthetic form of ADH that binds to kidney receptors and enhances reabsorption of water, thus reducing urine output. Desmopressin does not have any effect on sodium reabsorption. It may cause a slight increase or a transient decrease in blood pressure, but this does not affect urine output. Desmopressin does not increase cardiac output.

The nurse is teaching a client with newly diagnosed anemia about conserving energy. What does the nurse tell the client? (Select all that apply.) a. "Allow others to perform your care during periods of extreme fatigue." b. "Drink small quantities of protein shakes and nutritional supplements daily." c. "Perform a complete bath daily to reduce your chance of getting an infection." d. "Provide yourself with four to six small, easy-to-eat meals daily." e. "Perform your care activities in groups to conserve your energy." f. "Stop activity when shortness of breath or palpitations are present."

a. "Allow others to perform your care during periods of extreme fatigue." b. "Drink small quantities of protein shakes and nutritional supplements daily." d. "Provide yourself with four to six small, easy-to-eat meals daily." f. "Stop activity when shortness of breath or palpitations are present." It is critical to have others help the anemic client who is extremely tired. Although it may be difficult for him or her to ask for help, this practice should be stressed to the client. Drinking small protein or nutritional supplements will help rebuild the client's nutritional status. Having four to six small meals daily is preferred over three large meals; this practice conserves the body's expenditure of energy used in digestion and assimilation of nutrients. Stopping activities when strain on the cardiac or respiratory system is noted is critical. A complete bath should be performed only every other day; on days in between, the client can be taught to take a "mini" sponge bath, which will conserve energy and still be safe in preventing the risks for infection. Care activities should be spaced every hour or so rather than in groups to conserve energy; the time just before and after meals should be avoided.

A client has returned from a captopril renal scan. Which teaching does the nurse provide when the client returns? a. "Arise slowly and call for assistance when ambulating." b. "I must measure your intake and output." c. "We must save your urine because it is radioactive." d. "I must attach you to this cardiac monitor."

a. "Arise slowly and call for assistance when ambulating." Captopril can cause severe hypotension during and after the procedure, so the client should be warned to avoid rapid position changes and about the risk for falling as a result of orthostatic (positional) hypotension. Intake and output measurement is not necessary after this procedure, unless it had been requested previously. A small amount of radionuclide is used in a renal scan; the urine is not radioactive, although the nurse should practice Standard Precautions, as always, and wear gloves. Cardiac monitoring is not needed, although the nurse should monitor for hypotension secondary to captopril.

A distant family member arrives to visit a female client recently diagnosed with leukemia. The family member asks the nurse, "What should I say to her?" Which responses does the nurse suggest? (Select all that apply.) a. "Ask her how she is feeling." b. "Ask her if she needs anything." c. "Tell her to be brave and to not cry." d. "Talk to her as you normally would when you haven't seen her for a long time." e. "Tell her what you know about leukemia."

a. "Ask her how she is feeling." b. "Ask her if she needs anything." d. "Talk to her as you normally would when you haven't seen her for a long time." Asking the client how she is feeling is a broad general opening and would be nonthreatening to the client. Asking if she needs anything is a therapeutic communication of offering self and would be considered to be therapeutic and helpful to the client. The family member should talk to her as she normally would when she hasn't seen her in a long time. There is no need to act differently with the client. If she wants to offer her feelings, keeping a normal atmosphere facilitates that option. Acting as if things are "different" because she has cancer takes the control of the situation from the client. Telling her to be brave and not to cry is callous and unfeeling; if the client is feeling vulnerable and depressed, telling her to "be brave" shuts off any opportunity for her to express her feelings. There is no need to inform the client about her disease, unless she asks about it. Opening the conversation with discussion about leukemia should be the client's prerogative.

A client with a urinary tract infection is prescribed trimethoprim/sulfamethoxazole (Bactrim). What information does the nurse provide to this client about taking this drug? (Select all that apply.) a. "Be certain to wear sunscreen and protective clothing." b. "Drink at least 3 liters of fluids every day." c. "Take this drug with 8 ounces of water." d. "Try to urinate frequently to keep your bladder empty." e. "You will need to take all of this drug to get the benefits."

a. "Be certain to wear sunscreen and protective clothing." b. "Drink at least 3 liters of fluids every day." c. "Take this drug with 8 ounces of water." e. "You will need to take all of this drug to get the benefits." Wearing sunscreen and protective clothing is important while taking trimethoprim/sulfamethoxazole. Increased sensitivity to the sun can lead to severe sunburn. Sulfamethoxazole can form crystals that precipitate in the kidney tubules; fluid intake prevents this complication. Clients should be cautioned to take all of the drug that is prescribed for them, even if their symptoms improve or disappear soon. Emptying the bladder is important, but not keeping it empty. The client should be advised to urinate every 3 to 4 hours or more often if he or she feels the urge.

A client with chronic kidney disease asks the nurse about the relationship between the disease and high blood pressure. What is the nurse's best response? a. "Because the kidneys cannot get rid of fluid, blood pressure goes up." b. "The damaged kidneys no longer release a hormone that prevents high blood pressure." c. "The waste products in the blood interfere with other mechanisms that control blood pressure." d. "This is a compensatory mechanism that increases blood flow through the kidneys in an effort to get rid of some of the waste products."

a. "Because the kidneys cannot get rid of fluid, blood pressure goes up." In chronic kidney disease, fluid levels increase in the circulatory system. The statements asserting that damaged kidneys no longer release a hormone to prevent high blood pressure, waste products in the blood interfere with other mechanisms controlling blood pressure, and high blood pressure is a compensatory mechanism that increases blood flow through the kidneys in attempt excrete waste products are not accurate regarding the relationship between chronic kidney disease and high blood pressure.


Conjuntos de estudio relacionados

CSCS- Testing and Data Evaluation

View Set

Ellentétpárok - Adjektive Gegenteil

View Set

Massage Practice | HLTH3821 | Quiz 1 testtest

View Set

Economics of Strategy: Chapter 10: Information & Valuation CreationWa

View Set

Study Guide SOL 6 Part 2: Roman Empire

View Set